Está en la página 1de 117

Isquemia de la úvula marcha carente de coordinación como un ebrio

ataxia troncal

isquemia del lóbulo anterior del trastornos de la postura e hipertonia extensora


cerebelo

irritación de la parte rostral del flexión de una extremidad ipsilateral


interpósito

isquemia del N. dentado músculos hipotónicos que se fatigan fácil, reflejos


osteotendinosos pendulares

Núcleos vesiculados, cuerpos comofilos golgi II capa granulosa

C. purkinje pasan por la sustancia gris y granulosa

núcleo rodeado de un citoplasma delgado, es una C. granulosa


célula inmunorreactiva al glutamato

gémulas C. purkinje

El emboliforme y globoso Proyectan sus fibras principalmente ⅔ caudales del N. rojo

núcleo dentado se proyecta a la oliva principal

núcleo interpósito posterior dirige sus fibras al núcleo olivar accesorio medial

proyecciones eferentes del no salen a través del pedúnculo cerebeloso superior


nucleo del techo

Núcleo centromediano Tercio caudal del tálamo entre NMD y NVP

núcleo parafascicular Rodea la parte dorsomedial del fascículo retroflexo

nucleos intralaminares celulas fusiforme y de color oscuro

NPC Lámina medular interna

miembro superior culmen

miembro inferior L. central

estimulación táctil General potencial de acción bilateral en los lobulillos paramedianos

Cara lobulillo simple

L. Anciforme Localizado entre el surco posterosuperior y el lobulillo delgado

Cruz I,II L. Anciforme

Lingula Estructura más rostral del surco primario

L. Anterior Rostral al surco primario


Relaciona los conocimientos del dolor referido:

Riñón neuralgia intercostal de la 12 costilla hasta el ombligo

Esofago entre las escápulas

Estómago hombro derecho

Pulmones trapecio derecho

Relaciona los conocimientos del ritmo circadiano

21H00 Comienzo de la secreción de melatonina

18H30 mayor elevación de la presión sanguínea

16H00 temperatura corporal más alta

Relaciona los contenidos anatómicos


F. trepadoras procedentes Tienen glutamato como neurotransmisor
del complejo olivar inferior

F. trepadoras conexión excitadora de tipo todo o nada con las células de purkinje

Glomérulo contiene fibras presinápticas y postsinápticas en relación compleja


cerebeloso

F. musgosas pierden la mielina en la capa granulosa

Empareja los contenidos segun corresponda:

Fibras simpaticas preganglionares de la N. esplacnico mayor


glandula suprarrenal

tronco vagal anterior Izquierdo

fibras simpaticas postganglionares esfinter Plexo hipogastrico


interno anal

esfinter vesical Plexo hipogastrico

1. Femenino de 42 años consulta por cuadro de 5 días de evolución caracterizado por alteración
fluctuante del estado de vigilia, excitación psicomotriz con agresividad , presenta además signo de
Babinski bilateral, pérdida de la sensibilidad de todo el cuerpo del cuello hacia abajo,disminución de
fluidez del lenguaje y alteraciones visuales y parálisis vertical de la mirada. La RM cerebral presenta
un área hipertensa en sectores paramediales de ambos tálamos que se extiende a mesencéfalo.
¿Qué arteria está afectada por el paciente?
RESPUESTA: percherón

2. Paciente masculino de 55 años que presenta trombosis de la carótida interna, produzco de un


trauma de la región anterior del cuello, como consecuencia de una golpiza, el examen físico el
paciente se encuentra con una PA de 140/95, glucemia de 92 y FC de 120, al examen neurológico
se constató obnubilación, hemiplejia izquierda y una estrabismo de carácter convergente en el ojo
derecho, con estos datos, ¿en qué lugar del recorrido de la arteria carótida interna se
encuentra la lesión?.Respuesta: porción cavernosa.

3. Femenino 76 años ingresa por un cuadro clínico de tres horas caracterizado por disartria,
ataxia, alteraciones en la marcha y disminución del estado de consciencia. Al examen se
encontró: lateralidad motora dada por hemiparesia braquiocrural izquierda, mirada conjugada
con desviación derecha, hiperreflexia. presentó crisis convulsivas tónico clónicas
generalizadas. Se solicitó una ecocardiografía en la que se encontró una tumoración auricular
izquierda. Empleando sus conocimientos de neuroanatomía, indique ¿que arteria puede estar
afectada?

Respuesta: Arteria cerebral media derecha

Femenino 64 años de edad con antecedentes de hipertensión arterial no controlada acude a por
presentar movimientos bruscos involuntarios incesantes en el miembro superior izquierdo y pie
ipsilateral que se mantiene durante todo el día. Estos movimientos son observables en la noche, sin
predominio de reposo o actividad y sin limitación del movimiento voluntario. El cuadro es compatible
con una hemicorea vascular. En la RM cerebral observamos una imagen hiperintensa en región
talámica posterolateral derecha.

¿Qué arteria se puede encontrar afectada? Arteria coroidal posterior

4. Masculino 58 años de edad con antecedente de hipertensión arterial acude a emergencias por
presentar movimientos bruscos involuntarios incesantes en el miembro superior izquierdo y pie
ipsilateral que se mantienen durante todo el día. Estos movimientos son observables en la
noche, sin predominio de reposo o actividad y sin limitación del movimiento voluntario. el
cuadro es compatible con una hemicorea vascular: ¿Dónde esperaría encontrar la lesión en
una resonancia magnética cerebral?
Respuesta: Región talámica posterolateral derecha

5. ¿Cuál de las siguientes estructuras se localiza en la parte ventral del tercer ventrículo en las
inmediaciones del receso infundibular extendiéndose a la eminencia media? Respuesta: N.
Anciforme
6. En los dermatomas T5-T9 que visera encontramos: Píloro
7. ¿cual de los siguientes enunciados es INCORRECTO?
Respuesta: Las fibras estriadonigras se originan en una población de neuronas no espinosas
diferente a la estriadopalidales, pero comparten los mismos neurotrasmisores
Opciones correctas:
● las grandes neuronas del globo pálido pertenecen a una sola población neural
● las fibras eferentes del globo pálido se dividen en cuatro haces
● las fibras estriadopalidales inmunorreactivas a encefalina se localizan en GPL
● las fibras subtalamo palidales ejercen un efecto excitador sobre el globo pálido

8. Paciente masculino de 44 años, se presenta a consulta neurológica refiriendo que desde


hace una semana y sin ninguna razón aparte, ha perdido toda la sensibilidad el lado
derecho del cuerpo, refiere también tener poca sensibilidad en su rostro y que la comida
no le está sentando tan bien como de costumbre, bajo este cuadro el neurólogo a cargo
del caso decide ordenar la RM cerebral con la cual se diagnostica un infarto talámico,
utilizando sus conocimiento de neuroanatomía, responda dónde está la lesión
Respuesta: Tálamo izquierdo, núcleo VPL y VPM

9. ¿La irrigación de la paratiroides superior izquierda está a cargo de? A. tiroidea superior
izquierda
10. La glándula tiroides está situada en una vaina fascial que pertenece al sistema de las
fascias del cuello. Esta vaina está constituida de la siguiente manera:
Anteriormente por la hoja profunda de la lámina pretraqueal de la fascia cervical,
posteriormente por la vaina visceral y sus extensiones
11. En la relación a las venas tiroideas superiores, todas SON INCORRECTAS MENOS:
Drenan en el tronco tirolingofaringo facial

12. Paciente masculino de 45 años, acude a la emergencia refiriendo un dolor de cabeza y


fallece súbitamente, en la autopsia se observa un daño en la zona señalada del encéfalo,
usando su conocimiento de neuroanatomía, responda cuál de las siguientes
afirmaciones es CORRECTA:

La arteria que irriga a esta zona también irriga al área 41 de


brodman

13. En relación al Asa límbica la respuesta correcta es: Corteza cerebral …. estriado ventral….
pálido ventral…. DMmc
14. cual de los siguientes núcleos talámicos cumple una función descrita de la siguiente
manera: “respuestas asociadas al comportamiento. que implican decisiones basadas en
la predicción y en los incentivos”: Núcleo mediodorsal del tálamo
15. Seleccione la opción correcta en relación con la estructura señalada con la flecha negra.

Drena la sangre hasta el seno transverso

16. Quien le da su inervación autónoma a las glándulas sublinguales: VII par, nervio lingual,
fibras parasimpáticas

17. Una mujer de 25 años consultó a su medico porque experimentaba episodios de cambio de
color acompañado de dolor en los dedos cuarto y quinto de ambas manos. Al principio sus
dedos se volvían blancos por la exposición al frío y luego adquieren un color azul oscuro. El
cambio de color se limitaba a la mitad distal de cada dedo y se acompañaba de un dolor agudo.
El único tratamiento que le aliviaba el dolor consiste en mantener las manos cerca de una
estufa caliente o entrar en una habitación cálida. la paciente le comentó al médico que había
notado que sus dedos estaban húmedos de sudor durante algunos de los episodios. utilice sus
conocimientos de neuroanatomía y formule el diagnóstico
Respuesta: síndrome de Raynaud

18. ¿Cuál de las siguientes estructuras es reconocida por su propiedad de presentar


calcificaciones?
Respuesta: Glándula pineal

19. ¿Que tipo de fibras vegetativas son las que inervan las glándulas sudoríparas y los músculos
piloerectores?
Respuesta: Fibras simpáticas colinergicas

20. ¿Cuál de los siguientes núcleos se encuentra en la adherencia intertalamico?

Respuesta: núcleos Reuniens

21. Recibe sus principales aferencias de dos sitios: el segmento palidal lateral y la corteza
motora las proyecciones corticales probablemente representen colaterales de fibras
corticofugas destinadas a otras sustancias: Núcleo subtalámico

22. Las siguientes estructuras forman parte del sistema límbico, EXCEPTO: Locus niger

Si forman parte del sistema límbico: Área 32,24 de Brodmann, hipocampo, núcleo
accumbens, corteza del cíngulo

23. cual de las siguientes afirmaciones es correcta:


Respuesta: La arteria radicular de Adamkiewicz surge en la mayoria de ocasiones en zonas
lumbares altas del lado izquierdo

24. escoja la alternativa correcta


Respuesta: la ereccion masculina es una funcion parasimpatica, la eyaculacion es una funcion
simpatica

25. Las fibras poscomisurales de la estría terminal terminan en ?


Respuesta: Area preoptica medial

26. En relación al asa cognoscitiva, seleccione cuál de las siguientes secuencias


corresponde a la vía
Respuesta: Corteza cerebral, nucleo caudado, GPi MNr, Tálamo, Corteza

27. Paciente femenino de 32 años, se presenta a la emergencia con un cuadro de dificultad


para el habla y movimiento, bajo este cuadro el interno de la guardia decide
interconsulta a neurología, se decide emplear un estudio imagenologico y con este se le
diagnostica un infarto de la PICA izquierda, usando sus conocimientos de
neuroanatomía, responda cual de las siguientes afirmaciones es correcta.
Respuesta: Las PICA no irriga a las piramides bulbares

28. En relación con el complejo amigdalino todo son correctos EXCEPTO


Respuesta: Las fibras que se originan en la mitad rostral del hipotálamo pasan a todos los
nucleos amigdalinos

29. Las proyecciones amigdalocorticales a que parte de la corteza específicamente llegan?


Respuesta: Cíngulo lóbulo parietal
30. Paciente masculino de 35 años de edad sin antecedentes de enfermedad, acude a consulta por
deterioro progresivo de la memoria reciente y ligeramente de la pasada desde hace 1 año, no
refiere trastornos el sueño ni eventos estresantes dice tener una vida sana con buena
alimentación y actividad, solo refiere que hace dos años nadando en el mar casi se ahoga y
tuvo que ser asistido con respiración boca a boca, por 10 minutos. El examen físico neurológico
y general se encuentra dentro de los parámetros normales, en los exámenes de laboratorio lo
único destacable es una hemoglobina rozando el limite inferior. ¿Qué área del cerebro está
afectada?
Respuesta: Hipocampo

31. Adolescente de 17 años que acude por presentar evidente retraso de la pubertad, no presenta
caracteres sexuales secundarios y los genitales tiene aspecto infantil, además de ello su voz es
muy aguda, por lo que está sufriendo bullying en la escuela.
Según sus conocimientos escriba en qué parte del SN está la lesión

Respuesta: Eje hipotálamo hipofisario

32. En relación con los núcleos límbicos y de proyección inespecifica cuál de los siguientes núcleos
tiene como aferencias el fascículo mamilotalámico?
Respuesta: NA

33. Las fibras postganglionares simpaticas llegan a la glandula parotida por:


Respuesta: nervio auriculotemporal

34. cual de las siguientes vías constituyen una vía aferente y eferente del hipotálamo?
Respuesta: Haz prosencefálico medial

35. escoja la alternativa correcta:


Respuesta: La arteria espinal posterior procede de la arteria vertebral

36. Núcleo localizado en la parte más ventral del tercer ventriculo, cerca de la entrada al receso
infundibular y se extiende a la eminencia media.
Respuesta: N. Aciforme

37. qué neurotransmisor utilizan los núcleos ventral lateral, medial y posterior del talamo para
conectarse con la porción sensoriomotora del neoestriado?
Respuesta: Glutamato
38. Con respecto a la irrigación del páncreas, menciona de donde proviene la arteria
pancreatoduodenal inferior.
Respuesta: arteria mesenterica superior
39. que estructura conecta a los nucleos del trigono habenular con los nucleos interpedunculares?
Respuesta: Epitalamo
40. Cual de los siguientes vasos deberia estar infartado para afectar VA,VL,DM,AV?
Respuesta: Tuberotalamico
41. La invervacion autonoma del pancreas es a cargo de?
Respuesta: Plexo celiaco
42. cual de los siguientes no es una caracteristica comun entre la atetosis, la core y el balismo?
Respuesta: Se manifiestan con marcada rigidez
43. Paciente masculino de 25 años previamente sano que presenta un cuadro de 7 meses de
evolución caracterizado por taquicardia, hipertensión arterial, debilidad muscular proximal,
estrías cutáneas abdominales, fascie de luna llena, joroba en la espalda, visión borrosa,
alteraciones neurológicas, osteomusculares, cutáneas y auditivas. Se le realizan pruebas
diagnósticas que orientan hacia una posible causa hipofisiaria y se decide realizar una
resonancia magnética cerebral que revela una masa de 6 mm en adenohipófisis que deforma
ligeramente la estructura de la glándula hipofisaria. Mencione patología que presenta el
paciente.
Respuesta: Enfermedad de Cushing

44. Una mujer de 48 años visita a su médico de cabecera con la queja de orinar con frecuencia. La historia
revela que este síndrome apareció hace unos 3 meses; en ese momento la mujer recuerda que tuvo un
dolor de cabeza repentino que respondió a los medicamentos de venta libre. la mujer también dice que
tiene SED todo el tiempo y tiene problemas para dormir debido a sus frecuentes viajes al baño por la
noche. las pruebas de laboratorio revelan niveles normales de glucosa, la resonancia magnética muestra
una pequeña lesión dentro de su hipotálamo, es más probable que esta lesión se localice en cuál de los
siguientes núcleos hipotalámicos?

RESPUESTA: B

45. Un hombre de 43 años es llevado al Servicio de Urgencias. La historia proporcionada por la


familia es conocida por el personal de urgencias, el hombre tiene un desorden de abuso de
alcohol que padece déficits característicos de su problema, además se le ha diagnosticado
diabetes insípida debido a una lesión en el hipotálamo. basado en el diagnóstico de su
diabetes. ¿Cuál es el lugar más probable de la lesión hipotalámica?

RESPUESTA: B

46. Paciente con cuadrantanopsia homónima, hemihipoestesia, afasia transcortical, hemiparesia y


coreoatetosis. ¿Qué arteria estaría afectando su aporte de sangre?
Respuesta: Coroidea posterior

47. ¿qué arteria sería la responsable de brindar riego sanguíneo a?Segmento posterior de la
cápsula interna,GP, amígdala,tracto óptico, tálamo lateral, lóbulo temporal medial.
Respuesta: Coroidea anterior

48. Un paciente con un infarto de la arteria tuberotalámica derecha acude con: parálisis
facial para los movimientos emocionales, hemiparesia contralateral, defectos del
campo visual, deterioro del intelecto y temblor del miembro superior contralateral ¿Cuál
de los siguientes NO corresponde al cuadro clínico del paciente?
R// TEMBLOR DEL MIEMBRO SUPERIOR CONTRALATERAL

49. en relacion al asa motora, seleccione cual de las siguientes secuencias corresponde a
la via directa.
Respuesta: Corteza cerebral,putamen,GPi MNr, nucleo pedunculopontino,talamo,
corteza
50. Relaciona los contenidos anatómicos
F. trepadoras procedentes del complejo olivar inferior: tienen glutamato como
neurotransmisor
F. trepadoras: conexión excitadora de tipo todo o nada con las células de purkinje
Glomérulo cerebeloso: contiene fibras presinápticas y postsinápticas en relacion
compleja
F. musgosas: pierden la mielina en la capa granulosa
51. relaciona las siguientes partes anatomicas
cruz I,II: L. Anciforme
L anciforme: localizado entre el surco posterior y el lobulillo delgado
L. Anterior: Rostral al surco primario
Língula: Estructura más rostral al surco primario
52. Relaciona las diferentes lesiones
Isquemia del lóbulo anterior del cerebelo: Trastornos de la postura e hipertonía extensora
Isquemia de la úvula: Marcha carente de coordinación como un ebrio ataxia troncal
Isquemia del N. dentado: Músculos hipotónicos que se fatigan fácil, reflejos osteotendinosos
pendulares
Irritación de la parte rostral del interpósito: Flexión de una extremidad ipsilateral
53. Relaciona las distintas aferencias y fisiológicas
Alta actividad de la N. acetil transferasa: Oscuridad
Núcleo habenular medial: Sustancia innominada, Área tegmental ventral
Bajos niveles de actividad de la hidroxinol metiltransferasa: Lesión bilateral del núcleo
supraquiasmático
Núcleo habenular lateral: Núcleo del Rafe mesencefálico
54. Relaciona las proyecciones y características de los núcleos
Núcleo centromediano: Tercio caudal del tálamo entre NMD y NVP
Núcleo parafascicular: Rodea la parte dorsomedial del fascículo retroflexo.
Núcleos intralaminares: Células fusiformes y de color oscuro
NPC: Lámina medular interna
55. Relaciona los contenidos anatómicos
C. Purkinje: Pasan por la sustancia gris y la granulosa
Núcleos vesiculados, cuerpos cromófilos: Golgi II capa granulosa
Gémulas: C. Purkinje
Núcleo rodeado de un citoplasma delgado, en una célula inmunorreactiva al glutamato: C.
granulosa
56. Relaciona los contenidos anatomofisiológicos del cerebelo
Miembro superior: Culmen
Miembro inferior: L. Central
Estimulación táctil: Genera potencia de acción bilateral en los lobulillos paramedianos
Cara: Lobulillo simple
57. Relaciona
Núcleos anteriores del tálamo: Cuerpo calloso
Núcleos de la línea media: Sustancia gris periventricular
NMD: Magnocelular, parvocelular, paralaminar
Núcleo mamilar lateral: Calota mesencefálica
58. Relaciona
F. Trepadoras: conexión excitadora de tipo todo o nada con las células de Purkinje
Glomérulo cerebeloso: contiene fibras presinápticas y postsinápticas en relación compleja
F. musgosas: pierden la mielina en la capa granulosa
F. Trepadoras procedentes del complejo olivar inferior: Tienen glutamato como neurotransmisor
BANCO SEGUNDO PARCIAL
NO SAPOS HPTAS
MACRO

Núcleos (AV, AD,AM) : Tienen escasas sustancia cromófila y rodeadas por una
cápsula de mielina fina
N. especificos de relevo: Vias ascendentes
Grupo nuclear anterior: Parte rostral del tálamo
N. asociacion: Áreas 18, 19
Núcleo mamilar lateral: Calota mesencefálica
NMD: magnonuclear, parvocelular, paralaminar
Núcleos de la línea media: Sustancia gris periventricular
Núcleos anteriores del tálamo: cuerpo calloso
Célula de purkinje: Pasan por la sustancia gris y granulosa
Núcleo rodeado de un citoplasma delgado, es una célula inmunorreactiva al
glutamato: Célula granulosa
Gémulas: Célula Purkinje
Nucleo vesiculados, cuerpos cromofilos: Golgi II capa granulosa
Núcleo habenular lateral: Sustancia innominada, área tegmental ventral
Núcleo habenular medial: Núcleo del Rafe mesencefálico
Bajos niveles de actividad de la hidroxinol metiltransferasa: Lesión bilateral del
núcleo supraquiasmático
Alta actividad de la N. acetil transferasa: Oscuridad
Isquemia de la Úvula: Marcha carente de coordinación como un ebrio ataxia troncal
Irritación de la parte rostral del interpósito: Flexión de una extremidad ipsilateral
Isquemia del N. Dentado: Músculos hipotónicos que se fatigan fácil, reflejos
osteotendinosos pendulares
Isquemia del lóbulo anterior del cerebelo: Trastornos de la postura e hipertonía
extensora
F. Musgosas: Pierden la mielina en la capa granulosa
F. Trepadoras: Conexión excitadora de tipo todo o nada con las células de Purkinje
Glomérulo cerebeloso: Contiene fibras presinápticas y postsinápticas en relación
compleja
F. Trepadoras procedentes del complejo olivar inferior: Tienen glutamato como
neurotransmisor
Grupo nuclear anterior: rostral al tálamo
Área 18-19: N de asociación
Núcleos AV AD AM: escasa sustancia cromófilas y rodeada por una cápsula de
mielina fina
Específicos de relevo: vias ascendentes
Glomérulo cerebeloso: contiene fibras presinápticas y postsinápticas en relación
compleja
Núcleo interpósito posterior: Dirige sus fibras al núcleo olivar accesorio medial
El emboliforme y globoso → Proyectan sus fibras principalmente 2/3 caudales del
Núcleo rojo
Proyecciones eferentes del núcleos del techo → No salen a través del pedúnculo
cerebeloso superior
L. anciforme → localizado entre el surco posterosuperior y el lobulillo delgado
Língula → estructura más rostral al surco primario
L. anterior → Rostral al surco primario
Cruz I, II → L. anciforme
Alta actividad de la N. acetil transferasa → oscuridad
Núcleo habenular lateral → sustancia innominada, área tegmental ventral
Bajos niveles de actividad de la hidroxinol metiltransferasa → lesión bilateral del
núcleo supraquiasmático
Núcleos habenular medial → núcleo del rafe mesencefálico
Núcleos específicos de relevo → Vías ascendentes
Grupo nuclear anterior → parte rostral del tálamo
NPC → Lámina medular interna
Núcleos intralaminares → células fusiformes y de color oscuro
Núcleos centromediano → Tercio caudal del tálamo entre NMD y NVP
Núcleos parafascicular → Rodea la parte dorsomedial del fascículo retroflexo
Miembro inferior → L. central
Estimulación táctil → Genera potencial de acción bilateral en los lobulillos
paramedianos
Miembro superior → Culmen
Cara → Lobulillo simple
Núcleos de la línea media → sustancia gris periventricular
Núcleo mamilar lateral → Calota mesencefálica
Núcleos anteriores del tálamo → Cuerpo calloso
NMD → Magnocelular, parvocelular, paralaminar
Núcleos vesiculados, cuerpos cromófilos → Golgi II capa granulosa
Núcleo rodeado de un citoplasma delgado, es una célula inmunorreactiva al
glutamato → C, granulosa
C. Purkinje → pasan por la sustancia gris y la granulosa
Gémulas → C. Purkinje
CASOS CLÍNICOS

Paciente masculino de 55 años que presenta una trombosis de la carótida interna,


producto de un trauma de la región anterior del cuello, como consecuencia de una
golpiza, al examen físico del paciente se encuentra con una PA de 140/95, glucemia
de 92 y FC de 120, al examen neurológico se constató obnubilación, hemiplejía
izquierda, y una estrabismo de carácter convergente en el ojo derecho, con estos
datos, en qué lugar del recorrido de la arteria carótida interna se encuentra la lesión.
RESPUESTA: Porción Cavernosa

Femenino 64 años de edad con antecedente de hipertensión arterial no contralada


acude a por presentar movimientos bruscos involuntarios incesantes en el miembro
superior izquierdo y pie ipsilateral que se mantienen durante todo el día. Estos
movimientos son observables en la noche, sin predominio de reposo o actividad y
sin limitación del movimiento voluntario. EL cuadro es compatible con una hemicorea
vascular. En la RM cerebral observamos una imagen hiperintensa en región talámica
posterolateral derecha.
Respuesta: art coroidea post

Femenino 76 años ingresa por un cuadro clínico de tres horas caracterizado por
disartria, ataxia, alteraciones en la marcha y disminución del estado de conciencia.
Al examen se encontró: lateralidad motora dada por hemiparesia braquiocrural
izquierda, mirada conjugada con desviación derecha, hiperreflexia. Presentó crisis
convulsivas tónico clónicas generalizadas. Se solicitó una ecocardiografía en la que
se encontró una tumoración auricular izquierda. Empleando sus conocimientos de
neuroanatomía, indique ¿Qué arteria puede estar afectada?
RESPUESTA: Arteria cerebral media derecha.

● Paciente masculino de 44 años , se presenta a consulta neurológica refiriendo que


desde hace una semana y sin ninguna razón aparente , ha perdido toda la
sensibilidad el lado derecho del cuerpo , refiere también tener poca sensibilidad en
su rostro, bajo este cuadro el neurólogo a cargo del caso decide ordenar una RM
cerebral con la cual se diagnostica un infarto talámico , utilizando sus conocimientos
de neuroanatomía , responda dónde está La lesión.
Respuesta: Tálamo izquierdo, núcleos VPL y VPM

● Paciente con pérdida pansensorial pura izquierda con hemiparesia ipsilateral,


defectos del campo visual, con movimientos coreiformes. En relación con sus
conocimientos del sistema vascular, ¿qué síndrome tiene el paciente?
Respuesta: Síndrome de Déjerine Roussy
Femenino de 42 años consulta por cuadro de 5 días de evolución caracterizado por
alteración fluctuante del estado de vigilia, excitación psicomotriz con agresividad
presenta además signo de babbinski bilateral, pérdida de la sensibilidad de todo el
cuerpo del cuello hacia abajo, disminución de la fluidez del lenguaje y alteraciones
visuales y parálisis vertical de la mirada. La RM cerebral presenta un área hipertensa
en sectores paramediales de ambos tálamos que se extiende a mesencéfalo. Qué
arteria esta afecta en el paciente ?
Respuesta: Percheron

Masculino 58 años con antecedentes de hipertensión arterial acude a emergencias


por presentar movimientos bruscos involuntarios incesantes en el miembro superior
izquierdo y pie ipsilateral que se mantienen durante todo el dia. Estos movimientos
son observables en la noche sin predominio de reposo o actividad y sin limitación del
movimiento voluntario. El cuadro es compatible con una hemicorea vascular:
¿Donde esperaria encontrar la lesion en una resonancia magnetica?
Región talámica posterolateral derecha

CUADROS RIKOS
Preguntas y casos de n euroanatomía T segund o p arcial
¿Cuál de los siguientes vasos acompaña en una parte de su recorrido al VII par craneal? Laberíntica

¿La superficie inferolateral y medial del lóbulo temporal, así como las superficies lateral y medial del
lóbulo occipital están a cargo de que vasos? La arteria cerebral posterior ramas corticales

Arteria que bordea los pedúnculos cerebrales, así como la pineal, velo medular superior entre otros.
Arteria Cerebelosa Superior

Las siguientes estructuras forman parte del sistema límbica, excepto: Locus Niger (hipocampo,
corteza del cíngulo, área 32, 24 de brodman, fórnix, núcleo accumbens)

Paciente de 55 años desempleado ingeniero industrial con hábitos de tabaquismo y alcoholismo que
empeoraron con el desempleo es traído por la hermana que refiere que a pesar de ser un hombre
muy serio y de poco hablar desde la juventud, hace unos meses está muy mentiroso, además se
desorienta y confunde de vez en cuando, el examen presenta amnesia para ciertos momentos de la
vida recientemente ocurridos, ¿Según sus conocimientos neuroanatómicos en qué lugar es la lesión
del SN? C uerpos mamilares, áreas adyacentes y tálamo

VA SCULAR

1. Un distinguido neurocirujano, durante una conferencia sobre accidentes cerebrovasculares, hizo la


siguiente afirmación: "En general hay acuerdo en cuanto a que no hay anastomosis de importancia
clínica entre las arterias terminales dentro de la sustancia encefálica, pero existen muchas anastomosis
importantes entre las grandes arterias, dentro y fuera del cráneo, y éstas pueden desempeñar un papel
importante en la determinación de la extensión del daño encefálico en la enfermedad
cerebrovascular". Comente esta afirmación y mencione los sitios donde ocurren anastomosis arteriales
importantes.

Respuesta:
• Polígono de Willis
• Ramas de las arterias cerebrales sobre la superficie de los hemisferios cerebrales y los hemisferios
cerebelosos
• Ramas de las arterias carótidas interna y externa
- Origen en la arteria carótida común.
- Entre las ramas de la arteria oftálmica dentro de la órbita y las arterias facial y maxilar.
- Entre las ramas meníngeas de la arteria carótida interna y la arteria meníngea media.

2. Durante el examen de una angiografía carotídea se observó que el medio de contraste había llenado
las arterias cerebrales anterior y media pero no la arteria cerebral posterior. Siguiendo
cuidadosamente el medio de contraste, se observó que entraba en la arteria comunicante posterior,
pero no se extendía más allá. Explique este fenómeno en una persona normal.

Respuesta:
3. Un hombre de 45 años fue internado en el hospital tres días después de haber perdido la conciencia
en su casa. Fue hallado por un amigo en el piso de la sala de estar en estado de inconsciencia parcial.
En el examen físico se halló hemianopsia homónima derecha, aunque el examen cuidadoso de los
campos visuales mostró que las regiones maculares eran normales. También había hemianestesia e
hemianalgesia derechas, aunque el paciente refería un dolor quemante intenso en la pierna derecha.
Durante las primeras 24 horas en el hospital, el paciente mostró una hemiparesia derecha de tipo
fláccido, que desapareció en 2 días. ¿Cuál es su diagnóstico? Describa en forma específica las ramas
de la arteria que están afectadas.

Respuesta:
Hemianopsia homónima derecha: oclusión de las ramas corticales de la ACP izquierda debido a isquemia
del área visual primaria en la cisura calcarina.
Síndrome talámico o D éjerine Roussy: Hemianestesia derecha, dolor quemante intenso en la pierna
derecha. Obstrucción de una de las ramas centrales de la ACP izquierda que irriga los núcleos sensitivos
del tálamo izquierdo.
Núcleos sensitivos del tálamo: CGL (Núcleo geniculado lateral) – VP (núcleo ventral posterior) – CGM
(núcleo geniculado medial).
Hemiparesia derecha leve y pasajera: oclusión transitoria de una rama de la ACP izquierda hacia el
pedúnculo cerebral izquierdo.

4. Durante el curso de la necropsia de un paciente que había fallecido recientemente por Una
enfermedad cerebrovascular, el patólogo comentó que, en la aterosclerosis de las arterias cerebrales,
las placas ateromatosas tienden a aparecer donde las arterias principales se dividen o donde las
arterias se curvan súbitamente. Se cree que en estos sitios los cambios del flujo de presión pueden ser
un factor en la causalidad del proceso patológico. Usando su conocimiento de anatomía, mencione
tantos sitios como pueda donde las arterias cerebrales principales se dividen o cambian bruscamente
de curso.

Respuesta:
• Seno carotideo de la arteria carótida interna en la bifurcación de la arteria carótida común o después
de esta.
• La primera bifurcación principal de la arteria cerebral media.
• Donde las arterias vertebrales se unen para formar la arteria basilar.
• Donde la arteria cerebral anterior describe una curva hacia arriba y atrás sobre la rodilla del cuerpo
calloso.
• Donde la ACP pasa alrededor de la cara lateral del pedúnculo cerebral.

Respuesta del Castroso

• En la región del extremo medial del surco cerebral lateral donde la arteria carótida
interna gira hacia atrás hasta la región del extremo medial del surco cerebral lateral.
Aquí se divide en las arterias cerebrales anterior y media.
• En la parte posterior del cuerpo calloso donde la ACA se curva hacia atrás del cuerpo
calloso y se anastomosan en la ACP.
• En el extremo inferior del puente, sitio donde se van a unir las dos arterias vertebrales
para formar la arteria basilar.
5. Habiendo examinado con todo cuidado a un paciente con una enfermedad cerebrovascular, el médico
se reunió con la familia para considerar la naturaleza de la patología, el curso de tratamiento y el
pronóstico. La hija le preguntó al médico qué quería decir accidente cerebrovascular y cuáles eran las
causas comunes. Los familiares también le preguntaron por qué varían tanto los hallazgos clínicos de
un paciente a otro. Usando su conocimiento de la anatomía y la fisiología del flujo sanguíneo cerebral,
explique por qué los pacientes con enfermedad cerebrovascular presentan esa variedad de síntomas.

Respuesta: Los síntomas y signos dependen de la causa de la interrupción del flujo sanguíneo cerebral y del
tamaño de la arteria afectada.
Por ejemplo: la embolia cerebral o la hemorragia cerebral es un evento súbito, mientras que el desarrollo de
aterosclerosis en un paciente con hipertensión es un proceso lento que puede empeorar repentinamente
cuando ocurre trombosis en el sitio de la placa ateromatosa. La hemiplejía es el signo más frecuente, pero
pueden desarrollarse muchos defectos sensitivos adicionales, según cual se la arteria bloqueada. Son ejemplos
la hemianestesia, la hemianopsia, la disfasia y la disartria.

6. El signo clásico de la enfermedad cerebrovascular es la hemiplejía; sin embargo, sabemos que la


mayoría de los pacientes también presentan déficit sensitivo de diferentes tipos. Usando su
conocimiento de la distribución anatómica de las arterias cerebrales, explique los principales tipos de
déficit sensitivos que pueden hallarse en estos pacientes.

Respuesta:
Oclusión de la arteria cerebral media o de sus ramas:
• Páralisis de los músculos lisos del lado contrario del cuerpo.
• Hemianestesia contralateral; isquemia del surco posterior
• Hemianopsia homónima: isquemia de la radiación óptica

Oclusión de la arteria cerebral anterior o de sus ramas:


• Pérdida sensitiva contralateral en la pierna, el pie y los dedos de los pies, debido a la isquemia de la
zona de la pierna de la corteza cerebral.

Oclusión de la arteria cerebral posterior o de sus ramas:


• Hemianopsia homónima contralateral; isquemia de la zona visual primaria en la región del surco
calcarino.

Oclusión de las ramas del tálamo:


• Hemianestesia contralateral.
• Dolor grave en las mismas áreas.

7. Durante la explicación de los signos y síntomas de una mujer de 70 años que había sido internada en
el hospital para el tratamiento de una enfermedad cerebrovascular, una estudiante de medicina de
cuarto año comentó que estaba sorprendida porque muchos de los signos y síntomas eran bilaterales
en esta paciente. Dijo que los tres pacientes previos que había examinado habían presentado sólo
signos y síntomas unilaterales. Usando su conocimiento de neuroanatomía, explique por qué algunos
pacientes presentan signos y síntomas bilaterales mientras que en otros el síndrome es claramente
unilateral.

Respuesta:
La arteria carótida interna y las arterias basilares resultan igualmente afectadas por la enfermedad.
• La arteria carótida interna irriga predominantemente un hemisferio cerebral a través de la arteria
cerebral anterior y de las ramas de la cerebral media.

Oclusión de la arteria carótida interna:


· Hemiplejía
· Hemianestesia
· Hemianopsia contralaterales
· Afasia
· Agnosia

Dependiendo de que el hemisferio dominante esté afectado.


• La arteria basilar contribuye a la irrigación de sangre de ambos lados del cerebro a través de las
dos arterias cerebrales posteriores y de muchas ramas de ambos lados del cuerpo. (Sx.
Enclaustramiento)
8. Los neurólogos hablan con frecuencia de hemisferio dominante y si una enfermedad cerebrovascular
afecta ese hemisferio puede esperarse que el paciente tenga afasia sensitivomotora global. Explique
este fenómeno.
Respuesta:
El hemisferio dominante posee la función del lenguaje.
Afecte a la arteria cerebral media del lado izquierdo será, por tanto, más grave que en el lado derecho,
dado que afectará a la zona cortical del habla y producirá una afasia motora y sensitiva completa. En
las personas que tienen un hemisferio derecho dominante, ocurre al revés.

9. Explique por qué los pacientes con trombosis de la arteria cerebral media a menudo se presentan con
hemianopsia homónima, así como con hemiplejía y hemianestesia.
Respuesta:

La arteria cerebral media, además de lugar a:


• ramas corticales
• ramas centrales, irrigan parte de la rama posterior de la cápsula interna y de la radiación
óptica.

Oclusión de las ramas centrales


• Origina una hemianopsia homónima contralateral.

10. Durante el curso de neurobiología, el profesor de neuroanatomía destacó la importancia de conocer


la estructura y la irrigación de la cápsula interna. Explicó la disposición de los tractos ascendentes y
descendentes dentro de la cápsula y mostró cómo se concentraban en un área pequeña entre el
tálamo y el núcleo caudado medialmente y el núcleo lenticular lateralmente. Resulta claro que una
interrupción de la irrigación de esta área vital produce defectos neurológicos diseminados. ¿Cuál es la
irrigación de la cápsula interna?
Respuesta:
La cápsula interna. Está irrigada por las ramas estriadas central y lateral de la arteria cerebral media y
por las ramas centrales de la arteria cerebral anterior.
Oclusión de su aporte sanguíneo produce un déficit neurológico extenso.

11. Un hombre de 36 años concurrió a su médico y refirió que en tres ocasiones durante los últimos seis
meses había perdido el conocimiento en el trabajo. Durante un interrogatorio cuidadoso, el paciente
dijo que en cada una de ellas había perdido del conocimiento estando sentado en su escritorio
mientras entrevistaba personal de oficina; agregó que las personas entrevistadas se sentaban en una
silla a la derecha de su escritorio. Dijo que inmediatamente antes de cada desmayo se sentía mareado,
luego perdía el conocimiento y se recuperaba en unos momentos. La tarde anterior había tenido un
episodio similar de mareos cuando giró la cabeza rápidamente hacia la derecha para hablar con un
amigo en la calle. El médico observó que el paciente usaba un cuello duro bastante ajustado. Cuando
el médico comentó esto, el paciente dijo que siempre usaba este tipo de cuello para trabajar. No se
observaron signos físicos anormales en el examen. Usando su conocimiento de anatomía y fisiología,
¿qué diagnóstico haría?

Respuesta 1:
• Este paciente presenta síntomas de síndrome Del seno carotídeo.
• La distensión de las paredes arteriales produce una reducción refleja de la frecuencia cardíaca
y caída de la presión arterial. Esto se produce como consecuencia de un aumento en el número
de impulsos nerviosos que pasan a través del nervio del seno, una rama del nervio glosofaríngeo
• La hipersensibilidad del reflejo o la presión externa pueden hacer que la presión arterial caiga
bruscamente y produzca isquemia cerebral y pérdida de consciencia

12. Un hombre de 45 años, director de una compañía, se levantó para hacer su discurso luego de la cena
anual, cuando súbitamente sintió un dolor "muy intenso y opresivo" sobre el esternón. Sintiéndose
mareado y débil, se dejó caer sobre su silla. Unos momentos más tarde perdió el conocimiento. Uno
de los concurrentes, que había recibido cierto entrenamiento en reanimación cardiopulmonar
mientras estaba en las fuerzas armadas, corrió hacia el paciente y observó que había dejado de
respirar. Rápidamente comenzó la reanimación boca a boca y el masaje cardíaco y continuó hasta que
llegó el personal de la ambulancia para llevar el paciente al hospital. El médico de terapia intensiva
refirió luego al paciente que la prontitud y la competencia de ese concurrente a la cena le habían
salvado la vida. Usando sus conocimientos de neurofisiología, diga durante cuánto tiempo puede
sobrevivir el tejido encefálico cuando hay paro cardíaco completo y la respiración ha cesado.

RESPUESTA:
• 5-10 sg se cortó el flujo sanguíneo al cerebro (pérdida de conciencia)
• 1 m ya no habrá función neurológica
• 4 m cambios irreversible, ya no hay solución (muerte celular)
• Trombosis coronaria es la causa más frecuente

13. Un hombre de 62 años con antecedentes de hipertensión consultó a su médico porque el día anterior
había perdido transitoriamente la visión del ojo derecho. Explicó que la pérdida de la visión era parcial
y que había durado alrededor de media hora. En un interrogatorio cuidadoso, el paciente admitió
haber experimentado episodios similares de ceguera en el mismo ojo en los últimos seis meses, pero
que sólo habían durado pocos minutos. El paciente también mencionó que había días en que no podía
recordar los nombres de personas y cosas. Recientemente también había experimentado cefaleas
intensas en el lado derecho. Al ser interrogado acerca de sus actividades, dijo que no podía caminar
tan bien como solía y algunas veces sentía la pierna izquierda débil y entumecida. Mientras realizaba
un examen físico cuidadoso, el médico auscultó con su estetoscopio un soplo sistólico evidente en el
lado derecho del cuello. Dado que el paciente tenía enfermedad vascular encefálica , ¿qué arteria es
probable que esté comprometida en la enfermedad? ¿Qué investigaciones clínicas especiales
efectuaría efectuarse para confirmar el diagnóstico?

Respuesta:
• La arteria carótida interna derecha se ve afectada
• consecuencia tendremos pérdida de memoria y cefaleas derechas
• Las mediciones oftalmodinamométricas pueden mostrar una disminución las arterias retinianas debido
a una disminución de la presión de la arteria oftálmica derecha
• el angiograma confirma la presencia de una estenosis critica de la arteria carótida interna en su origen

14. Un hombre de 39 años fue internado debido a que había sufrido una cefalea generalizada intensa de
aparición brusca mientras trabajaba en el jardín. Diez minutos más tarde cayó al suelo en estado de
inconsciencia. Después de haber sido llevado adentro de la casa y recostado, recuperó la conciencia,
pero parecía confuso. Refería cefalea intensa y rigidez de nuca. El examen físico mostró cierta rigidez
de nuca, pero nada más. Un examen neurológico cuidadoso tres días más tarde mostró cierta pérdida
de tono en los músculos de la pierna izquierda. Usando su conocimiento de anatomía, haga el
diagnóstico. ¿Cuál es el motivo de la rigidez de nuca?

Respuesta:
• paciente tenía un aneurisma congénito de la arteria comunicante anterior.
• El inicio brusco de una cefalea grave, que a menudo es tan intensa que el paciente siente como si se
hubiese sido golpeado en la cabeza, es característico de la rotura de un aneurisma congénito en el
espacio subaracnoideo.
• La rigidez de nuca; irritación meníngea producida por la presencia de sangre en el espacio
subaracnoideo.
• Este paciente no tenía evidencia de una presión previa sobre el nervio óptico que diera lugar a un
defecto de visión unilateral, lo que a veces se produce cuando el aneurisma se halla situado en la parte
anterior del circuito arterial cerebral.
• La pérdida de tono de los músculos de la pierna izquierda es difícil de explicar, aunque se puede deber
al daño que produce la hemorragia brusca en el espacio subaracnoideo en el hemisferio cerebral
derecho.

15. Un hombre de 26 años, luego de salir de un bar donde había bebido algunas copas, caminaba por la
calzada a la 1 de la mañana y fue golpeado por un vehículo que pasaba. Afortunadamente, el automóvil
circulaba lentamente y la cabeza del paciente recibió un golpe al sesgo. Una hora más tarde, un policía
encontró al paciente inconsciente en la vereda. En el examen físico en el hospital local se halló que
había recuperado la conciencia durante pocos minutos, pero enseguida había recaído en estado de
inconsciencia. La pupila derecha estaba dilatada y había hipotonía muscular de la pierna izquierda.
Había signo de Babinski positivo del lado izquierdo. El examen del cuero cabelludo mostró un
hematoma importante sobre la sien. derecha y una radiografía de perfil de cráneo mostró una fractura
en el ángulo anteroinferior del hueso parietal. La TC mostró un área densa que se extendía de adelante
hacia atrás a lo largo de la tabla interna del hueso parietal derecho. ¿Cuál es el diagnóstico?
Supongamos que el equipo para realizar una TC no se encontrara disponible y que se decidiera -
efectuar una punción lumbar; esta prueba puso de manifiesto una presión elevada del líquido
cefalorraquídeo y el líquido tenía una tinción sanguinolenta muy leve. Explique estos hallazgos
adicionales.

Respuesta:
• hemorragia extradural del lado derecho, debida a la fractura de la parte anterior del hueso parietal,
que ha roto la división anterior de la arteria meníngea media derecho
• encontrado inconsciente y de que después recuperó la consciencia durante un tiempo para luego
volver a quedar inconsciente es un hallazgo característico.
• El traumatismo inicial generalmente es responsable de una pérdida inicial de consciencia.
• La recaída en el estado de inconsciencia se debe a un gran coágulo de sangre a baja presión que
comprimía la capa meníngea de la duramadre. Esto es responsable de la pupila dilatada en el lado
derecho debido a la presión indirecta en el nervio oculomotor derecho.
• La presión sobre el surco precentral derecho produce hemiplejía y debilidad de la pierna izquierda;
también produce un signo de Babinski positivo en el lado izquierdo.
• La presencia de un coágulo también era la responsable de la elevación de la presión del líquido
cefalorraquídeo.
• La ligera tinción de sangre del líquido obtenido mediante una punción lumbar se debía a una
pequeña extravasación de sangre desde el espacio extradural en el espacio subaracnoideo en el
lugar de la fractura.

16. Una mujer de 50 años consultó a su médico a causa de cefaleas, somnolencia y confusión mental. Al
ser interrogada, la paciente recordó claramente haberse golpeado la cabeza contra la puerta de un
anuario al agacharse tres semanas antes. La TC mostró la presencia de una gran lesión ocupante sobre
el lóbulo frontal derecho del encéfalo. ¿Cuál es el diagnóstico posible?

Respuesta:
• tenía un hematoma subdural crónico tras un traumatismo en la cabeza producido 3 semanas
antes.
• Se produjo porque se rasgó una de las venas cerebrales superiores en el punto de entrada en el
seno sagital superior.
• La sangre se acumulaba a baja presión entre la duramadre y la aracnoides. Las cefaleas, la
somnolencia y la confusión se debían a un aumento de la presión intracraneal.

17. Un hombre de 55 años con antecedentes de hipertensión se desmayó en la calle cuando se dirigía a su
trabajo. Refirió una cefalea intensa súbita. Después de 5 minutos, su cara comenzó a "caer" del lado
derecho y la palabra se hizo arrastrada. Al ingresar en el hospital, se observó que el brazo y la pierna
derechos estaban más débiles que los izquierdos y los músculos se encontraban hipotónicos. Los ojos
estaban desviados hacia la izquierda. Más tarde el brazo y la pierna derechos mostraron una parálisis
total y no eran sensibles al pinchazo. Había signo de Babinsiki positivo del lado derecho. Dos horas
después el paciente entro en coma profundo y tenía las pupilas fijas y dilatadas. Más tarde. las
respiraciones se tornaron profundas e irregulares y el paciente falleció 6 horas después. Usando Su
conocimiento de neuroanatomía, haga el diagnóstico.

Respuesta:
• todos estos signos son diagnósticos de un accidente cerebrovascular que afecta al hemisferio cerebral
izquierdo.
• En la necropsia se observó que las ramas centrales perforantes de la arteria cerebral media izquierda
se hallaban extensamente afectadas por aterosclerosis.
• Una de estas arterias se había roto, dando lugar a un gran hematoma en el núcleo lenticular y en la
cápsula interna izquierda.
• Las pupilas fijas dilatadas, la irregularidad de la respiración, y finalmente la muerte, se debieron al
aumento de la presión intracraneal dentro del hemisferio, produciendo una compresión hacia abajo
que afectaba al tallo cerebral.

18. ¿Cuál es la irrigación de la médula espinal? ¿Qué áreas de la médula espinal están irrigadas por la
arteria espinal anterior? ¿Qué regiones de la médula espinal son mas susceptibles a la isquemia?

Respuesta:
• recibe su aporte sanguíneo de tres arterias pequeñas: las dos arterias espinales posteriores y la espinal
anterior.
• La arteria espinal anterior irriga los dos tercios anteriores de la médula espinal.
• Los segmentos torácicos superior e inferior tienen un aporte relativamente escaso de sangr e porque
la arteria espinal anterior en esta región puede ser muy pequeña; por tanto, son más susceptibles a la
isquemia.

1. Las siguientes afirmaciones se refieren al aporte de sangre al cerebro:

(b ) E l circu ito a rterial cerebral está formado p or la a rteria cerebral a n terior, la arteria carótida
in t erna, la a rteria cerebral p o sterior, la a rteria b a silar, y la s a rterias co mu nicantes a nterior y
p o sterior.

2. Las zonas de la corteza cerebral enumeradas a continuación reciben su aporte de sangre como se
indica:

(a ) E l surco p recentral (área d e la cara) está irrigado p or la arteria cerebral med ia.

3. Las arterias enumeradas a continuación proceden de las arterias principales como se indica:

(d ) L a arteria espinal p o sterior p rocede d e la arteria vertebral.


4. Las venas enumeradas a continuación drenan en los senos venosos indicados:

(e) E l sen o sagital inferior d rena en el seno recto.

5. Las siguientes afirmaciones se refieren al torrente sanguíneo cerebral:

(d ) U n o d e los vaso dilatadores más p otentes d e los vasos sanguíneos cerebrales es el d ióxido d e
ca rbo no.

6. Las siguientes afirmaciones se refieren a la isquemia cerebral:


(c) E l d año cerebral irreversible comienza t ras el cese d el t orrente sangu íneo d u rante alrededor d e 4
min .

7. Las siguientes afirmaciones se refieren a la irrigación de la médula espinal:

(a ) Las arterias espinales p o steriores irrigan el tercio p osterior d e la méd ula espinal.

8. Un hombre de 58 años, mientras cenaba, se quejó bruscamente de cefalea. Momentos después,


cayó hacia delante y quedó inconsciente. Al ingresar en el hospital, el médico que lo examinó encontró
los siguientes signos, excepto:

(e) E staban au sentes los reflejos a bdominales d erech os, y había u na respuesta p o sitiva d e B abin ski en
el la do izq uierd o.

9. Un hombre de 58 años, mientras cenaba, se quejó bruscamente de cefalea. Momentos después,


cayó hacia delante y quedó inconsciente. Tres días después, el paciente recuperó la consciencia, y los
signos que aparecieron fueron los siguientes, excepto:
(c) L as p artes su perior e inferior d el lado d erecho d e su cara estaban p aralizadas.

10. Un hombre de 58 años, mientras cenaba, se quejó bruscamente de cefalea. Momentos después,
cayó hacia delante y quedó inconsciente. Durante las siguientes 2 semanas, se desarrollaron los
siguientes signos, excepto:

L o s mú sculos d el lado izquierdo mo straban h ipotonía.

11. El neurólogo encargado del paciente interpretó los hallazgos como sigue. Todas sus
interpretaciones fueron correctas, excepto:

(b ) L a p rofun didad d el coma n o tiene relació n con la extensión d e la o b strucción a rterial.


12. Los siguientes signos físicos y los datos anatómicos conocidos sugerían la afectación de la arteria
cerebral media, excepto:

(c) Las ramas centrales d e la arteria cerebral med ia n o irrigan el n úcleo lenticular, el n úcleo caudado n i
la cápsula interna.
13. Un hombre de 60 años ingresó en el servicio de urgencias quejándose del inicio brusco de un dolor
insoportable y lacerante localizado en la parte de atrás del tórax y en la espalda. Tras una exploración
física y pruebas radiológicas, se estableció el diagnóstico de disección de la aorta descendente. En
pocas horas, el paciente comenzó a experimentar un dolor «en cinturón» que afectaba al cuarto
dermatoma torácico en ambos lados. Más adelante, se observó que tenía una termoanestesia
bilateral y una analgesia por debajo del nivel del cuarto dermatoma torácico. El sentido de la posición,
la vibración y el tacto superficial se mantuvieron normales. Se desarrolló rápidamente una parálisis
espástica completa en ambas piernas.

El inicio brusco del dolor «en cinturón» en este paciente se debía con mayor probabilidad a:

(b ) B loqueo d e los o rígenes d e las a rterias interco stales p o steriores q ue dan lugar a las a rterias
esp inales segmentarias p or la d isecció n aórtica.
14. El desarrollo de la termoanestesia y de la analgesia bilateral por debajo del nivel del cuarto
segmento torácico de la médula y el posterior desarrollo de la paraplejía podría estar causado por:

(c) A usencia d e circulación en la a rteria esp inal a nterior.

L IMB ICO

1. Cuando se comentaban las bases neurológicas de las emociones en una mesa redonda, un
neurólogo pidió a una estudiante de tercer curso de medicina que explicara el síndrome de Klüver-
Bucy. ¿Qué hubiera respondido usted a esta pregunta? ¿Se ha presentado alguna vez el síndrome
de Klüver-Bucy en seres humanos?

R e: El síndrome d e Klüver-B ucy se refiere a los signos y síntomas q ue se o bservan en monos después de
la extracción b ilateral d el ló b ulo t emporal. L os mo no s se vu elven más d óciles e in sensibles, y no
mu estran sign os d e miedo n i d e d isgusto. Tienen u n a umento d el apetito y también de la actividad
sexual, a menudo con perversiones. N o son capaces d e reconocer los o bjetos q ue ven. Las personas en
la s q u e se h a d estruido el á rea amigdalina n o suelen d emostrar este síndrome, si bien se ha
d escrito en pacientes después de la extracción bilateral d e áreas extensas de los ló bulos temporales.

2. Una mujer de 23 años con antecedentes de crisis epilépticas desde hacía 4 años acudió a una
consulta con su neurólogo. Un amigo de la paciente describió con viveza una de sus crisis. Unos
segundos antes de que comenzaran las convulsiones, la paciente se quejaba de un olor
desagradable, similar al que se encontraría en una vaquería. Después, la paciente emitía un grito
estridente mientras caía al suelo inconsciente. Inmediatamente, todo su cuerpo participaba en
movimientos generalizados tonicoclónicos. Está claro que esta paciente presentaba una forma
generalizada de crisis epilépticas. A partir de sus conocimientos de neuroanatomía, indique qué
lóbulo del encéfalo estaba afectado inicialmente en la descarga epiléptica

R e: E l aura o lfatoria q ue p recedía a las convu lsiones g eneralizadas en u na crisis ep iléptica indicaría la
a fectación inicial d el ló bulo t emporal d e la corteza cerebral
3. Un hombre de 54 años falleció en el hospital por un tumor cerebral. Siempre había sido
intelectualmente muy brillante, y podía recordar con facilidad los episodios de su infancia. En los
últimos 6 meses, su familia observó que tenía dificultades para recordar dónde había dejado sus
cosas, por ejemplo, su pipa. También tenía dificultades para recordar los episodios nuevos, e
inmediatamente antes de su muerte ni tan solo podía recordar que su hermano le había visitado
el día anterior. A partir de sus conocimientos de neuroanatomía, indique qué parte del encéfalo
estaba siendo afectada por un tumor en expansión altamente invasivo.

R e: E n la a u t opsia se d emostró la in vasió n d el h ip ocampo, el fó rnix y lo s cuerpos mamilares en


a mb os h emisferios cerebrales. Parece ser q u e el h ipocampo p a rticipa en el almacenamiento y la
cla sificación d e la in formación a ferente relacionada con la memoria reciente.

4. Las siguientes afirmaciones se refieren a la formación reticular:

(d ) E n su p arte superior, la formación reticular sirve d e estación d e relevo h acia la corteza cerebral.

5. Las siguientes afirmaciones se refieren a las funciones de la formación reticular:


(a ) In fluye en la a ctividad d e las motoneuronas α y γ.

6. Las siguientes afirmaciones se refieren a las funciones de la formación reticular:

Pu ede co n trolar la s eferencias p a rasimpáticas y simpáticas med iante su s fa scículos


reticulomedu lares y reticuloespinales.

7. Anatómicamente, las siguientes estructuras forman colectivamente el sistema límbico:

L a s circunvo luciones subcallosa, d el cíngu lo y d el parahip ocampo, la formación d el h ipocampo, el


cu erpo a migdalino, los cuerpos mamilares y lo s n úcleos anteriores d el tálamo .

8. Las siguientes afirmaciones se refieren a las conexiones eferentes del hipocampo:

(b ) Tienen u n trayecto a través d el fórnix

9. Las siguientes afirmaciones se refieren a las funciones del sistema límbico:

E l h ipocampo está relacio nado co n la memoria reciente.

Preguntas leccio nes:

1. ¿Cuál de los siguientes enunciados no es correcto en relación a las colaterales de la A.C.I?

R E : la a rteria comunicante p o sterior (ACP) se o rigina d istal a la b ifurcación terminal d e la carotida


in t erna, forma p arte d el p olígono d e Willis p uede tener ramas (A. centrales p osteromediales).

R e: la arteria cerebral media n o irriga la corteza motora correspondiente al miembro in ferior, p ero si
la p o rción sensitiva d el mismo miembro in ferior.

2. Si una paciente debe ser intervenida por un carcinoma de tiroides y se remueve el aparato
ganglionar de la glándula, selecciona por donde sería el abordaje quirúrgico?

R e: R aíz lateral media

3. Es de característica nerviosa y fija la glándula al plexo celiaco, seleccionar cual de las siguientes
estructuras corresponde a la característica anterior, perteneciente a la suprarrenal

R e: H o ja p o steromedial

4. ¿Cuál de las siguientes estructuras no es irrigada por una rama de la arteria cerebral posterior
(ACP) o por la propia ACP?

R e: p lexos coroideos d e los ventrículos laterales p arte anterior.

5 . Que vasos no drena la sangre al plexo tiroideo:


R e: Venas t iroides su periores y medias

6. El síndrome de Dejerine Roussy se produce por el compromiso de la irrigación de cual de estas


arterias?
R e: Gen icu lotalamica

7. Cuales son los síntomas de la oclusión de la arteria carótida anterior:


R e: H emiplejia y H emiparesia co ntralateral, H emianopsia h o mónima contralateral.

8. La oclusión de la arteria cerebral posterior genera:

R e: H emianopsia h omónima co ntralateral (sin a fectación d e la macu la)

9. Si un paciente presenta parálisis bilateral, en especial en los miembros inferiores y una alteración
en la sensibilidad que se asemeja a una lesión de la medula espinal. ¿La obstrucción dónde
estaría?

R e: en ambas a rterias cerebrales anteriores

10. Paciente es llevado a urgencias por presentar un accidente cerebrovascular y en la evaluación


presenta lo siguiente: parálisis del hipogloso ipsilateral, hemiplejia contralateral, perdida de la
propiocepción y vibración contralateral.

R e: se en cuentra a fectada la a rteria espinal a n terior, t iene síndrome B ulbar Medial y están
a fectadas las p irámides b ulbares.

11. Unir conceptos:


V. Terminal: cisura cerebral t ransversa

V b a sal: Vena d e L abbe

VC M superficial: su perficie a nterior d el encéfalo

VC M p ofunda: cintilla o ptica

12. Respecto al triangulo de Silvio, cuales son los enunciados correctos:

R e: se forma en la región insular y la rama p o sterior d e la A CM (arteria cerebral media) forma el


p u n to d e S ilvio cuando emerge p or la cisura lateral.
13. Paciente con trastornos en la fisiología del sueno posterior a una lesión isquémica del tejido
cerebral, se examina y se miden valores de melatonina bajos en sangre. ¿Qué vaso estaría
ocluido?

R e: arteria central p osteromedial


14. Ramas de la arteria vertebral: arteria espinal anterior, arteria espinal posterior, arteria cerebelosa
posteroinferior, meníngea posterior.
E xcepto: arteria cerebelosa superior.
A U TONOMO

1 . U n h o mbre d e 3 5 año s estaba b ajando d e la parte d e atrás d e u n camión cuando éste empezó a
mo verse. Con u n p ie y a en el suelo agarró u na b arandilla d el camión con su mano d erecha y se
co lgó de la misma. El camión continuó moviéndo se a lo largo d e una manzana antes de p arar.
Mientras tanto, el h o mbre h abía sido arrastrad o a lo largo d e la calle colgando d el camión. Fu e
a t endido en el servicio d e urgencias en estado d e shock, con cortes y abrasiones en sus piernas. En
la exploración cu idado sa d e su b razo d erecho se a p reció u n a p arálisis d e lo s siguientes
mú sculo s: flexor cubital d el carpo, flexor p ro fundo d e los d ed os, interóseos p almar y dorsal, y
mú sculo s tenar e hipotenar. También existía pérdida de sensibilidad d e la cara medial del brazo, del
a n tebrazo y d e la ma no . E l reflejo tend inoso p rofundo d el b ícep s b raq uial estaba
co n servado, p ero el tricipital se hallaba a u sente. Tamb ién se a p reció q ue la p u p ila d el ojo
d erech o estaba co ntraída, y existía u na caída d el p á rpado su p erior d erecho . E l g lobo ocular
d erech o parecía ser menos prominente que el izquierdo. La piel de la mejilla derecha se notaba más
ca liente y seca, y est aba má s co lorada q u e la mejilla iz q uierda. U t ilizando sus
co n ocimientos d e n euroanatomía, explique lo s h allazgos clín icos.

R E : Como resultado de quedarse colgando por la mano derecha de un camión en movimiento, este
hombre ha sufrido una grave lesión por tracción de las raíces octava cervical y primera torácica del
plexo braquial. Los diferentes músculos paralizados del antebrazo y de la mano, junto con la pérdida
sensitiva son característicos de la parálisis de Klumpke. En este caso, la tracción del primer nervio
torácico fue tan importante como para desgarrar la rama comunicante blanca, separándolo del
ganglio simpático cervical. Esta lesión ha seccionado las fibras simpáticas preganglionares destinadas a
la cara lateral de la cabeza y del cuello, causando un síndrome de Horner derecho (tipo
preganglionar). Ello se ejemplifica por: a) la contracción de la pupila, b) la caída del párpado superior y
c) el enoftalmos. La vasodilatación arteriolar, debida a la pérdida de las fibras simpáticas
vasoconstrictoras, era la responsable de la mejilla roja y caliente en el lado derecho. La sequedad de
la piel de la misma también era debida a la pérdida de la inervación simpática secretomotora
destinada a las glándulas sudoríparas.

2 . E l p ed iatra recibe en la con sulta a u n n iñ o d e 3 a ñ o s co n el a ntecedente d e estreñ imiento


cró nico y d istensión a b d ominal d esde el p eríod o d e la ctancia. L a ma dre d ice q u e el
estreñ imiento ha id o empeoran do de manera progresiva. No responde a lo s laxantes, y requiere la
a p licación de u n en ema u n a vez a la semana p ara a liviar la d istensión a bd ominal. E n la
exp loración física, el a bdomen d el n iño se encuentra claramente d istendido, presentando u na
ma sa p alpable d e consistencia p astosa a lo largo d el curso d el co lon d escendente, en la fosa
ilíaca izq uierda. L a exploración anal muestra u n recto n o d ilatado y ausencia d e heces. Desp ués
d e u na repetida irrigación co lónica con solución salina, se administró un enema d e bario seguid o d e
u n a radiografía. É sta d emostró u n co lon d escendente marcadamente d istendido, con u n
ca mbio abrupto en el d iámetro luminal en el p unto que el colon descen dente se sigue del colo n
sig moides. F ue interesante co nstatar q ue el n iño n o p udo vaciar el bario d el colon. Utilizando
su s co nocimientos acerca d e la inervación autónoma d el colo n, ¿cuál es el d iagnóstico? ¿Cómo
t rataría a este p aciente?

RE: Este niño de 3 años tenía la enfermedad de Hirschprung, una anomalía congénita en la que existe un
fallo en el desarrollo del plexo mientérico (plexo de Auerbach) en la parte distal del colon. La parte
proximal del colon es normal, pero se distiende de manera progresiva debido a la acumulación de heces.
En este paciente pudo demostrarse después de la intervención que el colon sigmoides bajo carecía de
células ganglionares parasimpáticas. Así, este segmento del intestino no tenía peristaltismo efectivo, y
bloqueaba el paso de las heces. Una vez que el diagnóstico fue confirmado por la biopsia del segmento
distal del intestino, el tratamiento consistió en la exéresis quirúrgica del segmento aganglionar del
intestino.
3 . U n a mujer d e 2 5 años, muy n erviosa, acude a su médico p or experimentar ataques de palidez
d o loro sa d el cuarto y q u into d ed o d e a mbas manos. E xp licaba q ue sus síntomas habían
emp ezado h acía 2 a ños, d urante el in viern o, y q u e a fectaban p rimero a su mano derecha y, en
crisis siguientes, t a mbién a la iz q uierda. In icialmente, los ded os se vo lvían b lancos con la
exp osició n al frío , a dquiriendo luego u n color azulado. El cambio d e color se limitaba a la mitad
d istal d e cada d edo e ib a acompañado d e u n d olor agudo . El ú nico tratamiento que aliviaba el
d o lor era ma ntener la s manos so b re la estufa o p ermanecer en u n a hab itación caliente. A
med ida q ue el d olor d esaparecía, sus d edos se volvían rojo s e inflamad os. Le d ijo a su médico que
h a b ía n otad o q u e su s d ed os se h u medecían d e su d o r d urante a lgu nas d e las crisis.
E stablezca el diagnóstico utilizando sus conocimientos d e neuroanato mía. ¿Cuál es la in erva ció n
a u tón oma d e lo s vasos sanguíneos d e la extremidad superior? ¿Cómo se debería tratar a esta
p a ciente?

RE: Esta paciente presentaba la clásica historia de la enfermedad de Raynaud. Esta enfermedad es
mucho más frecuente en las mujeres que en los hombres, especialmente en aquellas con una
predisposición nerviosa. La palidez inicial de los dedos es debida al espasmo de las arteriolas digitales.
La cianosis que sigue es debida a la dilatación capilar ocasionada por la acumulación de metabolitos.
Dado que no existe torrente sanguíneo a través de los capilares, la hemoglobina desoxigenada se
acumula en su interior. Es durante este período de cianosis prolongada cuando la paciente presenta
un dolor intenso. Al exponer los dedos al calor, el vasoespasmo desaparece y la sangre oxigenada
fluye de vuelta a los capilares dilatados. Ahora se produce una hiperemia reactiva y un incremento en
la formación de líquido tisular, responsable de la inflamación de los dedos afectados. La sudoración
de los dedos durante la crisis probablemente es debida al exceso de actividad simpática, que puede
ser responsable, en parte, del vasoespasmo arteriolar

4 . U n a paciente o besa, de 45 año s, madre d e seis niños, fu e explorada por su médico por presentar
sín tomas su gestivos d e p atología d e la vesícu la b iliar. R efería p resentar ep isodios d e d o lor
có lico intenso p or d ebajo d el margen co stal d erecho y q ue, con frecuencia, se reflejaba a la
esp alda, a n ivel d e la p unta d el á ngu lo escapular derecho. E l médico se d irigió a u n estu diante d e
med icina y le d ijo: «Anota q ue la p aciente refiere d o lor irradiad o a la espalda». ¿ Qué q uería d ecir
co n esta a firmación? E xp lique el fen ómeno d el d o lor irradiado a la esp alda y ,
o ca sionalmente, al h ombro d erecho en los casos d e afectación d e la vesícula b iliar.

RE: La paciente ha presentado un cólico biliar. El dolor visceral está originado por el conducto cístico o
el colédoco, y es debido al estiramiento o al espasmo de la musculatura lisa de su pared. Las fibras
dolorosas aferentes pasan a través de los ganglios celíacos o ascienden vehiculizados por el nervio
esplácnico mayor para penetrar en los segmentos torácicos entre el quinto y el noveno de la médula
espinal. El dolor era referido a los dermatomas torácicos entre quinto y noveno en el lado derecho,
es decir, la piel correspondiente a la zona suprayacente e inferior de la escápula derecha.

5 . L a exploració n d e u n p aciente co n n eurosífilis mo stró q u e la p u p ila d e su o jo izquierdo era


p eq ueña y fija, sin reacción a la luz, p ero q ue se contraía cuan do se le ped ía q ue enfocase sobre un
o b jeto cercano . ¿ C uál es la in ervación d el iris? U t ilizan do su s co n o cimientos d e
n eu roanatomía, establezca d ó nde cree q u e ra dica la lesió n n eurológica q u e explica estos
t rastornos.
RE: Este paciente tiene una pupila de Argyll Robertson, que es una pupila pequeña y fija que no
reacciona a la luz, pero que se contrae con la acomodación. Esta situación suele deberse a una lesión
sifilítica. La inervación del iris se describe en la página 409. La lesión neurológica en este paciente
interrumpía las fibras que tienen un trayecto desde el núcleo pretectal hasta los núcleos
parasimpáticos del nervio oculomotor en ambos lados.

6 . U n h o mbre d e 3 6 año s fue atendid o en el servicio d e u rgencias p or u na h erida p o r p royectil d e


a rma d e fuego en la p arte b aja d e la espalda. La radio logía reveló q ue la b ala estaba a lojada en el
ca nal vertebral, a n ivel d e la tercera vértebra lumbar. La exp loración n eurológica completa
d emostró sign os y síntomas q ue in dicaban u na lesión completa d e la co la d e caballo. ¿Cuál es el
n ervio a utó nomo q ue inerva la vejiga? ¿Ten drá este paciente algún t ipo d e interferencia con la
fu n ció n vesical?

RE: La vejiga urinaria es inervada por las fibras simpáticas procedentes de los segmentos lumbares
primero y segundo, y por las fibras parasimpáticas del segundo, tercero y cuarto segmentos sacros de
la médula espinal. En este paciente, la cola de caballo había sido seccionada a nivel de la tercera
vértebra lumbar. Ello significa que las fibras simpáticas ganglionares que descienden por las raíces
anteriores del primer y segundo nervio lumbar estaban intactas, puesto que abandonaban el canal
vertebral para formar los nervios raquídeos correspondientes a un nivel por encima de la entrada de
la bala. Sin embargo, las fibras parasimpáticas preganglionares habían sido seccionadas en su trayecto
descendente en el canal vertebral en el interior de las raíces anteriores del segundo, tercer y cuarto
nervios sacros. Por ello, el paciente tenía una vejiga autónoma sin ningún control externo reflejo. La
vejiga se rellena hasta su capacidad y luego se orina por rebosamiento. La micción puede activarse
por una fuerte compresión de los músculos abdominales por el paciente, o asistida por presión
manual sobre su pared abdominal anterior en la región suprapúbica.

7 . E n u n a exploració n médica rutinaria, se d etectó la p resencia d e h ipertensión esencial en u n


h o mbre d e raza n egra d e 4 0 a ñ os. L as d eterminaciones d e p resión a rterial eran d e 1 8 0 d e
sistólica y 100 de d iastólica (mmHg). ¿Cómo puede tratarse médicamente a este paciente? ¿Cuál es
la a cción d e los d iferentes tipo s d e fármacos q ue se u tilizan h abitualmente en el tratamiento d e la
h ip ertensión?

RE: La causa precisa de la hipertensión es desconocida. Sin embargo, el objetivo del tratamiento es
disminuir la presión sanguínea y mantenerla, en la medida de lo posible, dentro de los límites de la
normalidad, antes de que se desarrollen complicaciones, como la hemorragia cerebral, e insuficiencia
renal o cardíaca. La mejor manera de conseguirlo en los pacientes con hipertensión leve es reducir el
volumen plasmático mediante el uso de diuréticos. Los fármacos β-bloqueadores se utilizan
extensamente. Reducen la frecuencia y la fuerza de la contracción del músculo cardíaco.

8. ¿ Qué sustancias n eurotransmisoras se lib eran en la s sigu ientes terminacio nes nerviosas: a)
simpática p reganglio nar, b ) p a rasimpática p reganglionar, c) p a rasimpática p o sganglionar, d)
fib ras simpáticas p o sganglionares p a ra el mú sculo ca rd íaco y e) fib ras simpáticas
p o sganglionares p ara las g lánd ulas sudoríparas d e la mano
RE: a) Acetilcolina, b) acetilcolina, c) acetilcolina, d) noradrenalina y e) acetilcolina.
9 . L a s siguientes afirmacio nes t ienen q ue ver co n el sistema n ervioso a utón omo:
RE: (a) El sistema nervioso entérico está formado por el plexo submucoso de Meissner y el mientérico de
Auerbach.

1 0 . L a s siguientes afirmacio nes t ienen q ue ver con el sistema n ervioso a utónomo:

RE: (a) Una pupila de Argyll Robertson indica que el reflejo de acomodación para la visión cercana es
normal, pero que se ha perdido el reflejo fotomotor.

1 1 . L a s siguientes afirmacio nes generales tienen q ue ver con el sistema n ervio so a utón omo:

RE: (d) El dolor que se origina en el tubo gastrointestinal se irradia a la línea media.

1 2 . L a s siguientes afirmacio nes respecto al síndrome d e H orner:

RE: El paciente presenta una vasodilatación de las arteriolas de la piel de la cara.

1 3 . L a eferencia simpática:

RE: Se limita a los segmentos D1-L2 de la médula espinal.

1 4 . L a n oradrenalina se secreta en las t erminaciones de:

RE: Las fibras simpáticas posganglionares.

1 5 . L a inervación p arasimpática q ue controla la g lánd ula p arótida surge:

RE: Del nervio glosofaríngeo.

1 6 . ¿ Cuál d e las siguientes a firmaciones d escribe la p a rte p arasimpática d el sistema n ervioso


a u tónomo?

RE: Los efectos son locales y delimitados, puesto que las neuronas preganglionares establecen sinapsis
con un número limitado de neuronas posganglionares.
1 7 . L o s fármaco s a nticolinesterásico s actúan a n ivel d e las sinapsis mediante:

RE: El bloqueo de la destrucción de la acetilcolina.

1 8 . L a atropina tiene el siguiente efecto en el sistema nervioso a utón omo:

RE:(c) Bloquea la acción de la acetilcolina en los lugares efectores en el sistema parasimpático.

1 9 . L a eferencia parasimpática en la médula espinal se p roduce en los n iveles:

RE: S2, S3 y S4.

2 0 . L a s siguientes afirmaciones h acen referencia a la inervación a utónoma de la vejiga u rinaria:

RE: Las fibras aferentes de la vejiga alcanzan la médula espinal en el segmento lumbar segundo y en
los segmentos sacros segundo, tercero y cuarto.

2 1 . L a s siguientes afirmacio nes h acen referencia a la inervación a utónoma del corazón:


RE:Los nervios simpáticos motivan la aceleración cardíaca e incrementan la fuerza de la
contracción del corazón.
Unir:

1 4 . Glándula submandibu lar (b ) Ganglio submandibular

1 5 . Glándula lagrimal (c) Ganglio pterigo palatin o

1 6 . Glándulas n asales (c) Ganglio p terigopalatino

1 7 . Glándula parotídea (a) Ganglio ótico

1 8 . Glándula sub lingual (b ) Ganglio submandibular

U n ir:

1 9 . Ganglio cervical superior (a ) E levador d el p árpado (sólo músculo liso)

2 0 . Ganglio ciliar (c) C onstrictor d e la p upila

2 1 . Ganglio celíaco (e) N inguno d e lo s a nteriores su perior

2 2 . Ganglio mesentérico inferior (d ) C olon d escendente

2 3 . Ganglio mesentérico superior (b ) A pénd ice vermiforme

U n ir:

2 4 . N ervio facial (c) N úcleo lagrimal

2 5 . N ervio o cu lomotor (b ) N úcleo d e E dinger-Westphal

2 6 . N ervio g loso faríngeo (a) N úcleo salival inferior

2 7 . N ervio h ipoglo so (d ) N ingu no d e los anteriores

GA NGLIOS O N UCLEOS B ASALES

1. U n a niña d e 10 a ños es visitada p or u n n eurólogo debido a la aparición gradual de movimientos


in vo luntarios. A l p rincipio, los mo vimientos fu ero n co n siderad os p o r su s p a d res co mo una
in q u ietud g eneral, p ero más tarde empezaron a p roducirse muecas faciales y movimientos en
sa cudidas d e lo s b razos y la s p iernas. E n este mo mento, la n iña p resenta d ificultades para
efectuar mo vimiento s n o rmales d e lo s b razos, y el h echo d e ca minar se h a vuelto
p ro gresivamente má s d ificultoso . L o s mo vimientos anómalos parecen emp eorar en las
extremidades su periores, y so n más exagerados en el lad o derecho del cuerpo. Los movimientos
emp eoran cuan do la n iñ a se h a lla excitada, p ero d esaparecen co mpletamente cu ando está
d o rmida. L a p aciente ha sido tratada recientemente p o r u na fiebre reumática. ¿Existe alguna
p o sible co n exión en t re lo s sín to mas d e la n iñ a y lo s g a nglios b asales d e lo s hemisferios
cerebrales?

R E : E sta n iña p resenta u na corea d e S ydenham (v. pág. 3 22). E ste cuadro se produce, en la mayoría de
lo s ca so s, en n iñ as en tre lo s 5 y lo s 1 5 a ñ os d e ed a d. S e ca racteriza p o r la presencia de
mo vimientos rápidos, irregulares e in voluntarios que carecen de u n a finalidad. La enfermedad tiene
relación con la fiebre reumática y , hab itualmente, se pro duce u na recuperació n completa.
2 . U n h o mbre d e 4 0 a ños q ue p resenta mo vimientos in voluntarios ráp idos y en sacudidas q ue
a fectan a las extremidades su periores e inferiores visita a su médico. El cuadro se inició h ace
a p roximadamente 6 meses, y h a empeorad o progresivamente. E l paciente afirma q ue está muy
p reocupado p or su salud , p o rque su p adre h abía p resentado síntomas similares h acía 3 0 años y
h a b ía muerto en u na institució n d e salud mental. S u mujer le d ice al médico q ue él también h a
su frido ep isodios d e d epresión extrema, y q u e h a o b servado q u e p resenta p eríod os d e
irritabilidad y co nd ucta impulsiva. E l méd ico efectúa el d iagn óstico d e corea d e H u ntingto n.
E mp leando sus conocimientos d e n euroanatomía, exp lique cómo a fecta esta en fermedad a los
n ú cleos basales.

RE: La corea de Huntington es una enfermedad hereditaria progresiva que suele aparecer entre los
30 y los 45 años. Los movimientos involuntarios suelen ser más rápidos y en sacudidas que los que se
observan en los pacientes con corea de Sydenham. Los cambios mentales progresivos dan lugar a
demencia y muerte. Existe una degeneración progresiva de las neuronas secretoras de GABA, las
secretoras de sustancia P y las de acetilcolina de la vía estriatonígrica. Esto da lugar a que las neuronas
secretoras de dopamina de la sustancia negra resulten hiperactivas; de esta forma, la vía nigroestriada
inhibe el núcleo caudado y el putamen. Esto causa los movimientos involuntarios. Se produce la
atrofia del núcleo caudado y del putamen.

3. U n h ombre d e 6 1 a ños p resenta d e forma b rusca movimientos incoordinados d el tronco y brazo


d erech o. L a extremidad superior d erecha sale p ro yectada b ruscamente, d e forma vigorosa y sin
o b jetivo alguno, g o lpeando a cualq uiera q ue se en cuentra en su ca min o. El p aciente se está
recu perando d e u na h emiplejía d el lad o d erecho, secundaria a una hemorragia cerebral. ¿Cuál es
el n o mbre q ue se da a este sign o clínico? ¿ Afecta este cuadro a lo s ganglios b asales?

RE: El signo clínico se conoce como hemibalismo. El inicio brusco suele estar causado por afectación
vascular debida a hemorragia u oclusión. Sí, el hemibalismo afecta a los núcleos basales; es el
resultado de la destrucción del núcleo subtalámico contralateral o de sus conexiones neuronales,
causando los movimientos violentos e incoordinados de los músculos axiales y proximales de la
extremidad.

4 . L a s siguientes frases se refieren a los ganglios basales:


RE: La cabeza del núcleo caudado está conectada con el putamen.
5 . L a s siguientes frases se refieren a los n ú cleo s (ganglios) basales:
RE: El núcleo amigdalino está conectado con el núcleo caudado.
6 . L a s siguientes frases se refieren a los ganglios basales:
RE: La cola del núcleo caudado se encuentra en el techo del ventrículo lateral.
7 . L a s siguientes frases se refieren a l n úcleo caudado:
RE: La cola acaba anteriormente en el núcleo amigdalino.
8 . L a s siguientes frases se refieren a las fibras corticoestriatales a ferentes al cuerpo estriado:
RE: Todas las partes de la corteza cerebral envían fibras al núcleo caudado y al putamen.
9 . L a s siguientes frases se refieren a las fibras n igroestriadas:
RE: Las neuronas de la sustancia negra envían axones al putamen.
1 0 . L a s siguientes frases se refieren a las fibras eferentes d el cuerpo estriado:
RE: Algunas de las fibras estriatopalidales tienen GABA como neurotransmisor.
1 1 . L a s siguientes frases se refieren a las funciones d e los ganglios basales:
RE: Las actividades del globo pálido preceden a las actividades de la corteza motora que se
refieren a los movimientos aislados de manos y pies.

D IENCEFALO: TALAMO H IPOTALAMO

1 . U n h o mbre d e 4 5 año s q ue h abía d esarrollado sú bitamente d ebilidad d e la p ierna izq uierda 1 2 h


a n tes, fue ingresado a la clín ica. E n la exploración se a preció q ue ten ía parálisis d e la p ierna
iz q uierda, co n d ebilidad d e los músculo s d el b razo izqu ierdo. L os múscu los d e las extremidades
a fectadas mo straban u n t o n o in crementado, y existía u n a exageración d e lo s reflejos
t en dinoso s en el lad o izquierdo d el cu erpo . Tenía tamb ién u na considerable p érdida sensitiva en
el la d o iz quierdo d el cuerpo q u e implicaba t anto a la sen sibilidad su p erficial co mo a la
p ro fun da. Durante la exploración , el p aciente p resentaba movimientos involu ntarios en forma d e
sa cudidas d e la p ierna izquierda. Cuando se le p edía q ue to case la p unta d e su n ariz co n el
ín d ice izquierdo, mo straba u n considerable temb lor intencional. La misma p rueba co n el b razo
d erech o n o mostraba nada anómalo. Tres d ías d espués, el paciente empezó a q uejarse d e u n
d o lor terrible en la p ierna izqu ierda. E l d olor empezaba de manera espo ntánea o se iniciaba p or u n
ligero toq ue d e las sábanas d e la cama. ¿Cuál es su d iagn óstico? ¿Cómo explica los variado s
sig nos y síntomas?
RE: Este hombre sufrió una trombosis de la rama talamogeniculada de la arteria cerebral posterior
derecha. Ello resultó en una lesión degenerativa en el interior del tálamo derecho, causando una
alteración de las sensaciones superficial y profunda del lado izquierdo del cuerpo. La hemiparesia
contralateral, implicando a la pierna izquierda y al brazo izquierdo con un incremento del tono
muscular, estaba producida por edema en el brazo posterior de la cápsula interna derecha,
situada en la proximidad, lo cual ocasionaba el bloqueo de las fibras corticoespinales. A medida
que se resolvía el edema, la parálisis y la espasticidad mejoraron. Los movimientos
coreoatetósicos de la pierna izquierda y el temblor intencional del brazo izquierdo eran debidos,
probablemente, a la lesión del tálamo derecho o de las fibras nerviosas dentotalámicas derechas.
El dolor insoportable que sufría en la pierna izquierda era debido a la lesión en el tálamo derecho.
2 . L a s siguientes afirmacio nes h acen referencia a l tálamo:
Todos los tipos de información sensitiva, con la excepción del olfato, alcanzan los núcleos
talámicos a través de las fibras aferentes.
3 . L a s siguientes informacio nes h acen referencia a l tálamo:
La vía nerviosa cerebelorubro-talamo-cortico-pontocerebelosa es importante en el movimiento
voluntario.
4 . L a s siguientes afirmacio nes h acen referencia a los n úcleos talámicos:

RE: Las proyecciones del núcleo ventral posterolateral ascienden a la circunvolución poscentral a
través del brazo posterior de la cápsula interna.

5 . L a s siguientes afirmaciones h acen referencia a l cuerpo geniculado medial:


RE: El cuerpo geniculado medial recibe información auditiva de ambos oídos, pero
predominantemente del oído opuesto.
6 . L a s siguientes afirmacio nes h acen referencia a l cuerpo gen icu lado lateral:

RE: Cada cuerpo geniculado lateral recibe información visual del campo opuesto de visión.
1: fisura longitudinal

2: rodilla del cuerpo calloso

3:cabeza del cuerpo caudado

4:tercer ventrículo

5:talamo

6:núcleo lenticular

7:cuerno anterior del ventrículo lateral

8:septum pellucidum

1 . U n jo ven d e 1 7 a ñ o s fue in gresado en la sa la d e medicina g eneral p a ra o bservació n. El


d iagnó stico d e sospecha era d e síndrome d e F röhlich. Presentaba u na h istoria de 3 meses de
cefaleas importantes. Más recientemente hab ía tenid o accesos d e vómito s y hacía u na semana
q u e n otó p rob lemas visuales. E l p aciente d ecía q ue tenía d ificultades para ver o b jetos en el
ca mpo lateral d e ambo s o jos. S us p adres estaban p reocupados p orque estaba engordand o, d e
ma nera especial a n ivel d e la p arte inferior d el tro nco. E n la exploración física se apreciaba q ue el
ch ico medía 188 cm y p resentaba u na o besidad tro ncular excesiva. Lo s testículos y el p ene eran
p eq ueños, y n o existía vello axilar n i p ú bico. U na radiografía lateral d el cráneo mostró u n
a g randamiento d e la silla turca, con erosión d el d orso d e la silla. L a exploración d e lo s campos
o cu lares co nfirmó q ue el p aciente p resentaba hemianopsia b itemporal p arcial. U tilizan do sus
co n ocimientos d e n euroanatomía, explique lo s síntomas y signo s d e este paciente
RE: El chico presentaba un síndrome de Fröhlich secundario a un adenoma cromófobo del lóbulo
anterior de la hipófisis. Esta lesión ocupante de espacio había ido erosionando gradualmente la
silla turca del cráneo y había comprimido el quiasma óptico, produciendo hemianopsia
bitemporal. El tamaño del tumor estaba causando un in|cremento de la presión intracraneal que
era responsable de las cefaleas y de los ataques de vómito. La presión sobre el hipotálamo
interfería con su función y daba lugar a la característica acumulación de grasa en el tronco,
especialmente en la parte inferior del abdomen. El hipogonadismo y la ausencia de características
sexuales secundarias podrían ser debidas a la presión ejercida por el tumor sobre los núcleos
hipotalámicos, con la consecuente pérdida del control sobre el núcleo anterior de la hipófisis.
También podría deberse al efecto directo del tumor presionando sobre las células vecinas del
lóbulo anterior de la hipófisis.
2. U n a mujer d e 4 0 año s se vio implicada en u n accidente d e a utomóvil en el q ue su frió graves
lesio nes craneales. Después d e u na recuperación lenta, p ero sin complicaciones, fue dada d e
a lta d el h osp ital sin n ingún signo n i síntoma residual. S eis meses después, la p aciente empezó a
q u ejarse d e micció n n octurna frecuente, con emisión d e grandes cantidades d e o rina clara.
Ta mbién explicaba q ue siempre tenía sed, y q ue b ebía alrededor d e 1 0 vasos de agua en u na
ma ñana. Utilizan do sus conocimientos d e n euroanatomía y d e n eurofisio logía, ¿cree q ue existe
a lguna conexión entre los síntomas u rinarios y el a ccidente d e a utomóvil?
RE: Sí, existe una conexión entre el accidente y los síntomas urinarios. Esta paciente presentaba
diabetes insípida ocasionada por la lesión traumática, ya sea del lóbulo posterior de la hipófisis o
del núcleo supraóptico del hipotálamo. En cualquier caso, la producción de vasopresina se
encontraba inhibida. Debe destacarse que una lesión del lóbulo posterior de la hipófisis no suele
dar lugar a diabetes insípida, puesto que la vasopresina producida por las neuronas del núcleo
supraóptico escapa directamente al interior del torrente sanguíneo.
3. ¿ C ree u sted p o sible q ue u n p aciente con h idrocefalia p ueda p resentar u n mal funcio namiento
h ip otalámico? En caso afirmativo, explique la conexión.
RE: Sí, es posible. La hidrocefalia ocasionada por el bloqueo de los tres orificios en el techo del
cuarto ventrículo o por el bloqueo del acueducto cerebral da lugar a un incremento de la presión
en el tercer ventrículo, con presión sobre el hipotálamo. Esta presión, que se centra en el suelo y
en la parte baja de las paredes laterales del tercer ventrículo, si es lo suficientemente grande,
puede ocasionar fácilmente un mal funcionamiento del hipotálamo.
4. E xp lique q u é significado t ienen lo s t érminos tracto h ip otálamo-hipofisario y sist ema p o rta
h ip o fisario.
RE: El tracto hipotálamo-hipofisario se describe en la página 388, y el sistema porta hipofisario se
describe también en la misma página. Hay que recordar que el hipotálamo ejerce su control sobre
las funciones metabólica y visceral a través de la hipófisis cerebral y del sistema nervioso
autónomo.
5. L a s siguientes afirmacio nes h acen referencia a l h ip otálamo:
RE: Los núcleos del hipotálamo se dividen por un plano imaginario formado por los pilares del
fórnix y el fascículo mamilotalámico en un grupo medial y uno lateral.
6. L a s siguientes afirmacio nes h acen referencia a l h ip otálamo:

RE: La barrera hematoencefálica se halla ausente en la eminencia media del hipotálamo, lo cual
permite que las neuronas controlen directamente los contenidos químicos del plasma.

7 . L a s siguientes a firmacio nes h a cen referencia a las fib ras aferentes q u e co munican a l
h ip otálamo:
RE: El hipotálamo recibe muchas fibras aferentes desde las vísceras a través de la formación
reticular.
8 . L a s siguientes afirmacio nes h acen referencia a l h ip otálamo:
RE: Las células nerviosas del hipotálamo produce hormonas liberadoras e inhibidoras que
controlan la producción de diferentes hormonas en el lóbulo anterior de la hipófisis.
9 . L a s siguientes afirmacio nes implican las actividades funcionales d el h ipotálamo:
RE: El hipotálamo regula los cambios físicos asociados con la emoción, como el incremento de la
frecuencia cardíaca y la palidez o rubor de la piel.
1 0 . L a s siguientes afirmacio nes h acen referencia a l tracto h ipotálamo -hipo fisario:
RE: Las células nerviosas de los núcleos supraóptico y paraventricular producen las hormonas
vasopresina y oxitocina
1 1 . L a s siguientes afirmacio nes h acen referencia a l sistema p orta h ipofisario:

RE: Conduce las hormonas liberadoras e inhibidoras para las células secretoras del lóbulo anterior de
la hipófisis

Preguntas d e lecciones:

1 . E n u nciado in correcto:

RE: las fibras estriadonigras se originan en una población de neuronas no espinosas diferente a las
estriadopalidades, pero comparten los mismos neurotransmisores.

2 . L a s fibras p ostganglionares simpáticas llegan a la g lándu la p arótida p or:

RE: nervio auriculotemporal

3 . ¿ E n relación a los n úcleo s talámico s y d e p royección inespecífica cual d e lo s siguientes n úcleos


t iene co mo aferencias el fascícu lo mamilotalámico?

RE:NA

4 . ¿ C uál d e los sigu iente n úcleos t iene como a ferencia el área gustativa p ontina?

RE:NVPM

5 . L a ctante q ue es t raído d e la guard ia en b razos d e la madre. E l p aciente egreso d e la sala de la


p ed iatría hace u n mes con el d iagnostico d e E nfermedad d e H irschspru ng. ¿ Qué parte del tubo
d ig estivo especifica se encuentra enormemente d istendida en el n iñ o?
RE: parte proximal del colon.
6 . Pa ciente d e 9 0 ano s q ue acude p or d o lores d e cabeza d e mayor inten sidad en la n oche, se le
realiza u na a ngiografía cerebral q u e reporta aneurisma d e gran t amaño en la u n ión d e la
co municante y la cerebral p o sterior. C uand o lo examinen entre o tros se encuentra d ilatación de
la p u p ila ip silateral co n p érdida d el reflejo fo t omotor. ¿ Qu e fibras a u tóno mas están
a fectadas p o r esta lesión?

RE:fibras parasimpáticas preganglionares.

7 . Ped iátrico d e 7 anos q ue acude referido p or p resentar cambio s d e la voz que se hace mas grave de
la vo z además refiere la mama esta con aparición d e b ellos en la región genital y la barba. ¿Dónde
está localizada la lesió n?

RE: destrucción de la pineal (tumor pineal glándula pineal).


8. A d o lescente d e 1 7 a nos q ue acu de p or p resentar evidente retraso de la pubertad, no presenta
ca racteres sexuales secundarios y los genitales tiene aspecto infantil, además de ello su voz es muy
a g uda, p or lo q ue está su friendo b u llying en la secundaria
Lesión: glándula pineal (pinealoma parenquimatoso).
9 . ¿ cuál d e lo s siguientes n úcleos tiene como eferencia la corteza p remotora?
RE: NVA
1 0 . S elecciona la inervación simpática d e las g lán dulas sudoríparas d e la cabeza:

RE:T1-T3 NIML

1 1 . L a gandula tiroides esta situada en u na vaina fascial q ue p ertenece a l sistema de las fascias d el
cu ello. Esta vaina esta con stituida d e la siguiente manera:

RE: anteriormente por la hoja profunda de la lámina pretraqueal de la fascia cervical, posteriormente
por la vaina visceral y sus extensiones.

1 2 . ¿ Qué estructura d el S N g enera como resp uesta a su est imulació n la liberación d e g randes
ca ntidades d e ACTH y h ormona gonadotrópica co n respuesta lactogena?

RE: complejo nuclear amigdalino (amigdala).

1 3 . E n relació n a las venas t iroideas superiores, todas so n incorrectas menos:

Correctas: drenan en el tronco tirolingotraqueal facial, drenan directamente en la vena yugular


interna

Incorrectas: Drenan en la vena subclavia, drena en las venas braquicefalicas.

1 4 . U n ir:

TRONCO VAGAL ANTERIOR-IZQUIERDO

FIBRAS SIMPATICAS POSTGANGLIONARES ESFINTER INTERNOANAL-PLEXO HIPOGASTRICO

ESFINTER VESICAL- PLEXO HIPOGASTRICO

FIBRAS SIMPATICAS PREGNAGLIONARES DE LA GLANDULA SUPREARRENAL-NERVIO ESPLANICO


MAYOR

1 5 . ¿ Cuál d e las siguientes n o es u na característica común entre la atetosis, la corea y el balismo?

RE: se manifiestan con marcada rigidez.

1 6 . U n ir:

N E UR OTRANSMIS OR E N L A MA YORIA D E L A S TE RMINACIONES POS GANGLIONARES S IMPATICAS-


NORADRENALINA

A GENTE QUE B LOQUEA L OS R ECEPTORES DE L AS CELULAS E FECTORAS S IMPATICAS -PROPRANOL

A GENTE B L OQUEANTE S OB RE L OS R E CEPTORES D E L A S C E LULAS E FECTORAS PA RAS IMPATICAS -


ATROPINA
A GENTE QUE IR RITA L A ACTIVIDAD AUTONOMA PARASIPATICA-METACOLINA

A GENTE QUE IR RITA L A ACTIVIDAD AUTONOMA S IMPATICA-FENILEFRINA.

1 7 . ¿ Cuál d e los sigu ientes n úcleos t iene como aferencia el b razo d el colículo inferior?

RE:NGM

1 8 . H o mbre d e 2 7 anos co n a ntecedentes d e d iabetes e h ipertensión arterial, además, dentro de su


fa milia existiero n casos d e fallecimientos p o r cardiopatías y u n ca so a islado de colesterolemia.
Meses atrás sufrió u n a ccidente d e transito q ue lo llevo a un cuadro de hemiplejia braquiocrural
iz q uierda con parálisis facial p eriférica d erecha p or lo q ue fue intervenido quirugicamente y fue
d a d o d e a lta u n a semana después d e la p ractica q u irúrgica a l no presentarsintomologia alguna.
L u ego d e tres meses sufrió u na caída que p rovoco una h erida p or debajo de la oreja, al cabo de dos
semanas, acu de a realizarse chequeos d e rutina, sin embargo, comenzó a llorar en el lado derecho
d e la cara.
RE: lesión de las fibras parasimpáticas posganglionares de los nervios auriculotemporal-auricula
mayor y facial derechos.
1 9 . ¿ Cual d e las siguientes estructuras se localiza en la parte ventral d el tercer ventrículo en las
in mediaciones d el receso infund ibular extend iéndose a la eminencia media?

RE: núcleo arciforme

2 0 . S eleccione u na:
a. La arteria carótida interna da una colateral para irrigar la adenohipófisis
b . U n in farto d e la arteria cerebral med ia ca usaría sordera cortical
c. La arteria radicular de Adamkiewitzc se encontrará comúnmente en zonas lumbares del lado
izquierdo
d. Un infarto de la arteria cerebral anterior nos podría causar un déficit motor en los miembros
inferiores
e. La arteria cerebral posterior es rama de la arteria basilar
21. Pa ciente ma sculino d e 2 5 años p reviamente sa no q ue p resenta u n cu adro d e 7 meses de
evo lución ca racterizado p or t a quicardia, h ipertensió n a rterial, d eb ilidad mu scular proximal,
estrías cu t áneas a b do minales, fa scie d e lu n a llena, jo roba en la esp alda, visió n borrosa,
a lteraciones n eu rológicas, o st eomusculares, cu táneas y a u d itivas. S e le realizan pruebas
d iagnó sticas q ue o rientan h acia u na p osib le causa h ip o fisiaria y se d ecide realizar una
resonancia magnética cerebral q u e revela u na masa d e 6 mm en a d enohipó fisis q ue deforma
ligeramente la estructura d e la g lándu la h ip o fisiaria. Mencione p a tología q u e p resenta el
p a ciente.
RE: Enfermedad de Cushing
2 1 . Pa ciente masculino d e 3 5 a ños d e edad sin antecedentes d e enfermedad, acude a consulta por
d eterioro p rogresivo d e la memoria reciente y ligeramente d e la p asada d esde hace 1 año , no
refiere trastorn os d el su eño n i eventos estresantes d ice t en er u n a vid a sa n a con buena
a limentació n y actividad, solo refiere que hace dos años nadando en el mar casi se ahoga y tuvo que
ser asistido con respiració n b oca a b oca, p or 1 0 minutos. Al examen físico neurológico y general
se en cuentra d entro d e lo s p arámetros n ormales, en los exámenes d e laboratorio lo
ú n ico d estacable es u n a h emoglob ina rozand o el límite in ferior. Qu e á rea d el cerebro está
a fectada?
R E : CA1ami

2 2 . ¿ La inervació n autónoma d el p áncreas es a cargo d e?


R E : Plexo celiaco
2 3 . Pa ciente d e 5 5 an os q ue p resenta u na tromb osis de la carótida interna, producto de un trauma de
la región anterior del cuello, co mo consecuencia de u n a golpiza, el examen físico del paciente
p resenta u n a PA d e 1 4 0/95, g lucemia d e 9 2 y FC d e 1 2 0 , el examen n eurologico se constato
o b n u lacion, h emiplejia izq uierda, y u n a estrabismo d e carácter convergente en el ojo derecho.
¿ Qu é lugar d el recorrido d e la carótida interna tiene la lesión?

R E : p orción cavernosa o intracavernosa.

2 4 . Pa ciente masculino d e 4 5 a ñ os, a cude a emergencias refiere a u n d o lor de cabeza y fallece


sú b itamente, en la a utop sia se o b serva u n d a ño en la z o na señalada. ¿Cuál d e las siguientes
a firmaciones es correcta?

RE: L a a rteria q ue irriga esta zo na tamb ien irriga el area 4 1 d e


B roadman .

2 5 . ¿ La irrigació n d e la paratiroides su perior izqu ierda está a cargo d e?

R E : arteria tiroidea su perior izquierda/ a rteria t iroidea in ferior izquierda.

2 6 . ¿ De d ó nde p rovienen las fibras a mielínicas q ue atraviesan el lóbulo p osterior d e la g lánd ula
h ip ó fisis?

R E : In fundíbulo

2 7 . ¿ Cuáles p ertenecen a la d ivisión d el n ú cleo ventral p osterior (VP)?

R E : Ventral p o steromedial y ventral p osterolateral.

2 8 . Pa ciente d e 5 3 a n os h a experimentado u n estado de vigilia a u n p eriodo b reve d e sueño, el


d o ctor a d ia gnosticar n a rcolepsia. S eñale la estructura anatómica y la h o rmona q ue se
en cuentra afectada.

R E : H ipotálamo, zo na lateral y o xerina.

2 9 . ¿ Con respecto a la u b icación d el gru po medial d el tálamo, cual en unciado es el correcto?


R E : localización med ial a la lámina med ular interna

3 0 . Pa ciente d e 2 0 anos acu de a emergencias u n viernes a las 3:00 am, sufrió u na caída alrededor de
u n p iso d e a ltura d e la vivien da d o n de festejaba, a l exponerlo a exámenes de imagenología
(t o mografía), en lo s resultados se ve co mprometido el p iso y p a redes d el tercer ventrículo.
¿ C uál d e las sigu ientes estructuras n erviosas estarían comprometidas?

R E : H ipotálamo, epitálamo y tálamo .

3 1 . L a s fibras simpáticas p o sganglionares q ue inervan la parótida se o riginan en:

R E : ganglio cervical superior

3 2 . R itmo circadiano:
ME JOR VELOCIDAD DE R EACCION- 1 5:30

ME JOR C OORDINACIÓN- 1 4:30

ME JOR E FICACIA C ARDIOVASCULAR Y FUERZA MUSCULAR- 1 7:00

3 3 . L a s terminaciones n erviosas d el comp onente a utónomo están activadas p o r:

R E : E stiramiento.

3 4 . ¿ N úcleo talámico q u e t iene co mo fu nció n la in hibició n y la mo dulación d e la t ransmisión


t a lamo cortical?
R E : n úcleo reticular.

3 5 . C ual d e lo s siguientes g rupos arteriales sería el respon sab le d e p ro ducir isquemia masiva de lo s
p a res craneales motores contralaterales si se infarta?
R E : Grup o anterolateral.

3 6 . A lguien d esea a prender a t o car guitarra. A l co mienzo es incapaz d e t ocar correctamente las
n o tas d e la canción, p ero con el t iempo va mejoran do las n otas d e la canción y lo hace de
ma nera a utónoma. ¿Qué es lo q ue le p ermite a l g uitarrista ad quirir este n ivel d e maestría con la
g u itarra?
R E : la p otenciación a largo p lazo.

3 7 . ¿ La lesión d e la raíz S 1 q ue reflejo afectaría?

R E : aq uiliano .

3 8 .cual d e las siguientes raíces p roduce d eb ilidad en la flexión d e la cadera si se lesiona?

RE : L2

3 8 . Medio d e fijación d e la g lándu la t iroides:

R E : vasos p aratiroideos, vaina p aratiroidea.

3 9 . R itmo circadiano:
C OMIENZO D E LA S ECRECION D E MELATONINA- 2 100

TE MPERATURA CORPORAL MAS A LTA- 19:00

MA YOR E LEVACION DE LA PRES ION S ANGUINEA-18:30

4 0 . R eceptores q ue reaccionan a nte la d eformación d el tejid o como estímulo:

RE: Merkel y Meissner

4 1 . ¿ Dónde se u bican los ganglios simpáticos en la región p élvica?

RE: Anterior al sacro

4 2 . R ecorrido d e las venas cerebrales externas (VCS up):

RE: Entre 8 y 12-superficie lateral del hemisferio-atraviesa la aracnoides-recorre entre aracnoides y


duramadre 2cm – lagunas venosas frente al seno sagital superior.

4 3 . E n relació n al comp lejo n uclear vestibular to das son correctas excepto:

RE: el núcleo medial también forma una proyección unilateral que desciende para coordinar
movimientos posturales de los miembros superiores.

4 4 . U n ir:

SENSITIVA GENERAL: PORCION ROSTRAL TRACTO SOLITARIO

SENSITIVA VISCERAL: PORCION INTERMEDIA DEL TRACTO SOLITARIO

MOTOR BRAQUIAL: NUCLEO AMBIGUO

FIBRAS SENSITIVAS GENERAL: NUCLEO ESPINAL DEL TRIGEMINO.

4 5 . E n relació n co n la d ivisió n fu n cio nal de n úcleos t a lámicos cu a l d e lo s sig uientes n o es


co rrespond iente a l grupo mu ltimodal?
R E : dorsomedial

4 6 . Pa ciente femenino d e 3 2 anos se p resenta a la emergencia con u n cuadro d e dificultad para el


h a b la y mo vimiento ., b ajo este cu a dro el intern o d e la g u ardia d ecide interconsulta a
n eu rología, se d ecide emplear u n estu dio imagenológico y con este se le d iagnostica un infarto de
la PICA izquierda. A firmación correcta:

RE: las PICA no irrigan a las pirámides bulbares.

4 7 . E n lo s d ermatomas T5-T8 q ue vísceras encontramos?

RE: Hígado

4 8 . Fo rman p arte d el sistema límbico:

Correctas: hipocampo, corteza del cingulo, área 32 y 24 de Broadmann, fórnix, núcleo accumbens

Incorrecto: Locus niger


4 9 . ¿ las p royeccio nes amigdalocorticales a q ue p arte d e la corteza específicamente llegan?

RE: somatosensorial y lóbulo temporal.

5 0 . A rteria q ue b ordea los p edúnculos cerebrales, a si como la p ineal, velo medular superior,
en tre o tros:

RE: arteria cerebelosa superior

5 1 . L a su perficie in fero lateral y med ial d el ló bu lo t emporal, a sí co mo las su perficies laterales y


med ial d el lóbulo o ccipital están a cargo d e:
• Arteria cerebral posterior ramas
corticales • Arteria cerebral posterior
ramas centrales • Arteria cerebral
posterior ramas coroideas • Arteria
pontina
• Arteria laberíntica
5 2 . Pa ciente d e 5 5 a n os d esempleado in g eniero in d ustrial co n h á b itos d e t a baquismo
y a lcoholismo, h ace u nos meses esta muy mentiroso, se d esorienta y confunde d e vez en
cu an do; el examen p resenta amnesia p ara ciertos momentos d e la vida recientemente
o cu rridos. L ugar d e la lesión:

RE: tálamo (cuerpos mamilares)

5 3 . Paciente femenin o d e 3 2 a ñ os a cude a la con sulta refirien do esterilidad, hipogonadismo,


a n amnesis se refería a nosmia, E n la R MN cerebral se confirmó u na h ipoplasia severa de ambos
b u lb os o lfatorios, escriba el síndrome q u e p resente el p aciente:

RE: Kallman

5 4 . E l d o lor referido a pendicular in icialmente se da en el epigastrio d ebid o


a : RE: Relacionado al pancreas
5 5 . Paciente d e 7 2 años q ue acude p o r d olores d el o jo d erecho, con visión d ob le se le realiza RMN
cerebral en la cual se aprecian lesiones d el sen o caverno so que se muestra a continuación junto
a la foto d el paciente selecciona el d iagn ostico p robable:

RE: Sindrome Tosola Hunt


5 6 . U N IR:
N ú cleo intermedio d e Cajal: con desarrollo en los segmentos medulares cervicales
superiores N ú cleo comisural p o sterior: Laminas V,Vl,Vll entre C2 y S2
N ú cleo ventro lateral: a lo largo de toda la medula ingresando al bulbo

Lesiones simpáticas
• El tronco simpático puede resultar lesionado en el cuello por heridas de bala o por arma blanca.
• Las lesiones por tracción de la primera raíz torácica del plexo braquial pueden afectar a los nervios
simpáticos destinados al ganglio estrellado.
• Estas situaciones pueden producir un síndrome de Horner de tipo preganglionar (v. más adelante).
• Las lesiones de la médula espinal o de la cola de caballo pueden ocasionar la disrupción del control
simpático de la vejiga.

Síndrome de Horner
Consiste en:
• contracción de la pupila (miosis)
• ligera caída del párpado (ptosis)
• enoftalmos
• dilatación de las arteriolas cutáneas
• pérdida de la sudoración (anhidrosis)

Todos estos síntomas resultan de una interrupción de la inervación simpática de la cabeza y del cuello.
Las causas patológicas incluyen las lesiones del tallo cerebral o de la parte cervical de la médula espinal,
capaces de interrumpir los fascículos reticuloespinales que descienden desde el hipotálamo hacia el centro
simpático de la columna lateral del primer segmento torácico en la médula espinal. Estas lesiones pueden
incluir la esclerosis múltiple y la siringomielia.
La tracción del ganglio estrellado debida a una costilla cervical o a la afectación del mismo por una lesión
metastásica puede llegar a interrumpir la parte periférica de la vía simpática.

1. Síndrome de Horner Central (primera neurona) → hiperestesia contralateral del cuerpo y la pérdida
de la sudoración de todo el hemicuerpo.
2. Síndrome de Horner Preganglionar (segunda neurona) → Incluyen la pérdida de la sudoración
limitada a la cara y al cuello, así como presencia de rubor o palidez de la cara y el cuello.
3. Síndrome de Horner Posganglionar → Dolor facial o signos de enfermedd del oído, la nariz o la
garganta.

Pupila de Argyll Robertson


• La pupila de Argyll Robertson se caracteriza por: pupila pequeña, de tamaño fijo y que no reacciona
a la luz, pero que se contrae como respuesta a la acomodación.
• Suele estar ocasionada por una lesión neurosifilítica que interrumpe las fibras que tienen un
trayecto desde el núcleo pretectal hasta el
núcleo parasimpático (Edinger-Westphal)
del nervio oculomotor en ambos lados.
• El hecho de que la pupila se contraiga con la
acomodación implica que las conexiones
entre el núcleo parasimpático y el músculo
constrictor de la pupila del iris se encuentran
intactas.

Síndrome de la pupila tónica de Adie


• Presenta un reflejo fotomotor ausente o disminuido
• Contracción lenta o retardada ante la visión cercana
• Dilatación lenta o retardada en la oscuridad.
• Este síndrome benigno, que probablemente sea el resultado de una alteración en la inervación
parasimpática del músculo constrictor de la pupila.
• El síndrome de Adie puede confirmarse estudiando la hipersensibilidad a los agentes colinérgicos.
Estos fármacos colinérgicos no originan contracción pupilar en la midriasis ocasionada por la lesión
del oculomotor o en la midriasis relacionada con fármacos
Síndrome de Frey
• Sigue a las heridas penetrantes de la glándula parótida.
• Durante el proceso de cicatrización, las fibras secretomotoras parasimpáticas posganglionares que
tienen un trayecto por el nervio auriculotemporal pueden crecer y unirse al cabo distal del nervio
auricular mayor, que inerva las glándulas sudoríparas de la piel facial suprayacente.
• Por este mecanismo, un estímulo dirigido a la producción de saliva, en vez de ello, ocasiona
sudoración

La lesión del nervio facial puede ir seguida de un síndrome similar. Durante el proceso de regeneración, las
fibras parasimpáticas, habitualmente destinadas a las glándulas submandibulares y sublinguales, se desvían
a la glándula lagrimal. Ello produce el lagrimeo de los ojos en asociación con salivación, las llamadas
lágrimas de cocodrilo.

Enfermedad de Hirschsprung
La enfermedad de Hirschsprung (megacolon) es una enfermedad congénita en la que existe una ausencia
de desarrollo del plexo mientérico (plexo de Auerbach) en la parte distal del colon. El segmento implicado
del colon carece de células ganglionares parasimpáticas y no existe peristaltismo. Ello impide la progresión
de las heces, y la parte proximal del colon se distiende enormemente.

Caco Clínico Límbico / Carpenter


Lesiones en el epitelio olfatorio → Degeneración en capas específicas de la capa glomerular.
D esgarramiento de los filamentos olfatorios → Fracturas en la lámina cribiforme del hueso etmoides o una
hemorragia en los lóbulos frontales.
Meningitis o un absceso del lóbulo frontal → Compromete los nervios olfatorios
Anosmia unilateral → Diagnóstica las neoplasias intracraneales, en especial meningiomas de la espina del
esfenoides o del surco olfatorio.
Tumor hipofisiario → Afectan el bulbo y la cintilla olfatoria (solo cuando se extiende hasta la silla turca)
Alucinaciones Olfatorias → Lesiones en la circunvolución del parahipocampo, uncus o áreas adyacentes
C risis uncinadas → Sensaciones olfatorias sssadables y pueden proceder a una convulsión generalizada.
Epilepsia psicomotora → Lesiones localizadas en el hipocampo y la estimulación local de esta estructura en
gastos conscientes. Episodio Convulsivo →
Lesión bilateral del hipocampo → Profundo deterioro de la memoria de acontecimientos recientes y con
trastornos relativamente leves de la conducta. La memoria de los acontecimientos remotos no suele estar
alterada. Aunque las funciones intelectuales generales pueden permanecer en un nivel bastante elevado.
Estos pacientes muestran una incapacidad para aprender nuevos hechos y destrezas. Aun cuando el fórnix
contiene la mayor parte de fibras eferentes de la formación del hipocampo, la evidencia acerca de que la
interrupción de estas fibras produce pérdida de la memoria es escasa. Los cuerpos mamilares, de manera
semejante al fórnix, parecen estar implicados en la memoria, pero los datos experimentales no apoyan esta
hipótesis.
lesión en la sustancia blanca desconecta las conexiones de red entre la corteza visual y las áreas de
procesamiento del lenguaje, un paciente puede perder la capacidad de leer.

. Las áreas de Wernicke y Broca se comunican entre sí a través de varias conexiones difere ntes, la más
conocida de las cuales es una vía de materia blanca subcortical llamada fascículo arqueado ( vea la Figura
19.2A). Además, numerosas conexiones polisinápticas transmiten información desde el área de Wernicke
a lo largo de la corteza perisilviana intermedia para llegar al área de Broca.

CAUSAS DE AFASIA

Relacionado con Wernicke y Broca


1. Circunvolución temporal superior → Wernicke
2. Áreas 44 y 45 → Broca
3. Formulación y planificación del lenguaje → Broca
4. Circunvolución frontal inferior → Broca
5. Área 22 → Wernicke
6. Léxico → Wernicke
7. Sintaxis → Broca
8. Comprensión del lenguaje → Wernicke

Sx Ataxia sin agrafia


• Pueden escribir normalmente. No pueden leer su propia escritura.
• Pueden nombrar las palabras que deletrean en voz alta.
• Anomia de color
• NO hay afasia significativa

Lesión en la corteza occipital dominante (generalmente izquierda) que se extiende hasta la parte posterior
del cuerpo callosum, a menudo un infarto de PCA. → impide el procesamiento de la información visual del
hemicampo derecho, incluido el material escrito. Por tanto, suele haber una:
• Hemianopsia derecha

Mientras tanto, la información sobre el hemicampo izquierdo que ha alcanzado la corteza occipital derecha
no puede cruzar a las áreas del lenguaje debido a la lesión en el cuerpo calloso posterior.

Sx Ataxia con agrafia


Lesión del lóbulo parietal inferior dominante. En la región de la circunvolución angular.
• La afasia a veces está ausente
• Puede consistir solo de disnomía y parafasias leves.
Sx de Gerstmann
• Agrafia
• Acalculia (habilidades de cálculo aritmético deterioradas)
• Desorientación de derecha e izquierda
• Agnosia de los dedos (incapacidad de nombrar o identificar dedos individuales)
Cuando los cuatro componentes están presentes en ausencia de un estado de confusión global u otro
trastorno difuso, este síndrome se está localizando fuertemente en el lóbulo parietal inferior dominante,
en la región de la circunvolución angular.

Lesiones localizadas: No afectan a los recuerdos antiguos, aunque éstos se acaban


perdiendo, junto con otras capacidades mentales. cuando se presenta una demencia
avanzada causada por una degeneración grave y extensa de la corteza cerebral.

C asos clínicos BARR


Amnesia
• Destrucción de ambos cuerpos mamilares.
• Se interrumpe el principal circuito del sistema límbica FUERA de la formación hipocampica.
• Memoria resulta alterada.
• Lesión bilateral en los núcleos mediodorsales del tálamo, lo que interrumpe las fibras
mamilotalámicas.
Amnesia Grave
• La transección quirúrgica bilateral del fornix, que se lleva a cabo para intentar limitar la propagación
de descargas epilépticas o durante la extirpación de tumores de la región del tercer ventrículo.

Amnesia global transitoria


• La pérdida de función del hipocampo puede ser debida a una oclusión arterial que cause un infarto
en la formación hipocámpica de un hemisferio que vaya seguido más adelante por un infarto
parecido en el otro hemisferio. Más habitualmente, un hipocampo intacto se ve privado de oxígeno
sólo durante un breve período de tiempo, tras el cual el paciente de forma repentina deja de ser
consciente de los acontecimientos que han tenido lugar durante las horas inmediatamente
precedentes, a la vez que es temporalmente incapaz de formar nuevos recuerdos.

Amnesia Anterograda
• Caracterizada por el deterioro de la consolidación de nuevos recuerdos declarativos, es una
consecuencia habitual de daños más graves en la cabeza.

Alzheimer
• Lesión en CA1 o Sector de Sommer
• Perdida de neuronas colinérgicas de la sustancia innominada del procensefalo basal.
• Se proyectan por el hipocampo y por toda la corteza cerebral.
• Incapacidad de formar nuevos recuerdos.

Psicosis de Korsakoff
• Lesión en el diencéfalo (tálamo (amnesia) y cuerpos mamilares)
• Trastornos metabólicos, alcoholismo.
• El paciente introduce acontecimientos que recuerda de un pasado remoto en historias elocuentes
pero descaradamente falsas, para intentar compensar la ausencia de recuerdos más recientes.
Cuando los hipocampos o los circuitos de Papez dejan de ser funcionales. se retienen los
C onmoción
recuerdos de acontecimientos antiguos porque éstos ya han quedado establecidos,
• Perdida de conciencia
posiblemente en forma de cambios macromoleculares por toda la corteza cerebral.

LESIONES BILATERALES EN EL SER HUAMANO


Kluver Bucy
Extirpación de los dos lóbulos temporales.
Provoca en ocasiones un apetito voraz, un incremento de la actividad sexual (a veces pervertida) y un
aplanamiento afectivo. Agnosia visual, también se pueden dar después de un traumatismo craneal, una
infección vírica del cerebro y, en algunos pacientes con la enfermedad de Alzheimer.
Epilepsia del lóbulo temporal
• La epilepsia es una afección en la que una excitación sináptica anormal causa una propagación
incontrolada de potenciales de acción por el cerebro.
• Iniciarse con síntomas sensitivos o una sensación subjetiva de extrañeza conocida como aura.
• La naturaleza del aura puede proporcionar una pista sobre la localización del foco epileptógeno
donde se inicia la actividad anormal.
• Estimulación de las neuronas motoras.
Epilepsia jacksoniana
Se origina en un foco situado en la corteza motora primaria.
El petit mal
• tipo de epilepsia infante
• Se caracterizada por episodios frecuentes de pérdida de la consciencia, de no más de un segundo
de duración, que se conocen como crisis de ausencia.
• Puede originarse en un foco del tálamo.

NOTA:
C abeza comprimida durante el nacimiento
• La localización más frecuente del foco epileptógeno es la superficie medial del lóbulo temporal,
que puede verse dañada por la cercana tienda del cerebelo.
• Las neuronas que se encuentran cerca de la cicatriz resultante constituyen el foco, que suele
localizarse en la amígdala, en el extremo anterior del hipocampo o en el área entorrinal.

En muchos casos, la actividad no se expande por todo el cerebro, y el diagnóstico puede pasarse por alto
debido a la ausencia de convulsiones

Proceso de ataque de epilepsia


1. Un ataque suele empezar con la alucinación de un olor desagradable pero no identificare causado
por la estimulación de la corteza del uncus y los núcleos corticomediales de la amígdala.
2. El aura a menudo incluye un deja vu. que es una sensación antinatural de familiaridad con los
alrededores y las circunstancias que se atribuye a la actividad de la formación hipocámpica, la
amígdala y la corteza de asociación sensitiva del lóbulo temporal.
3. A medida que el ataque avanza, aparecen sensaciones de miedo y ansiedad (estimulación de los
núcleos centrales y basolaterales de la amígdala) y manifestaciones vegetativas como sudoración,
taquicardia (frecuencia cardíaca rápida) y sensaciones abdominales peculiares (estimulación de la
amígdala, la corteza de la ínsula, el hipotálamo y las neuronas simpáticas preganglionares).
4. Excepcionalmente, se presentan un lenguaje y un comportamiento irracionales que el paciente
luego no recuerda.

ESTADOS DE ANSIEDAD
• La amígdala tiene una actividad inapropiada.
• Pueden presentar ataques de pánico, de una actividad excesiva del sistema nervioso simpático o
un estado generalizado dominado por sensaciones subjetivas de preocupación que se presenta
con manifestaciones motoras, corno tensión muscular e inquietud.

D epresión
En diversos trastornos psiquiátricos, mucho sufrimiento es debido a la depresión, que es un estado anormal
bastante diferente de la tristeza que todo el mundo puede experimentar en las circunstancias apropiadas.
Los fármacos que alivian la depresión intensifican tas acciones sinápticas de la noradrenalina y de la
serotonina, ya sea bloqueando la recaptación de aminas en las terminaciones presinápticas (antidepresivos
tricíclicos como la amitriptilina y la imipramina), ya sea inhibiendo la monoaminooxidasa, una enzima que
cataliza la degradación oxidativa de la noradrenalina y la serotonina. La mayor parte de neuronas que usan
aminas como transmisores se localiza en el tronco encefálico. Sus axones altamente ramificados finalizan
en la sustancia gris del prosencéfalo, incluidas todas las partes del sistema límbico.
Esquizofrenia
El proceso de pensar está profundamente alterado, con delirios, alucinaciones auditivas, incapacidad para
asociar ideas y una disminución de la expresión de las emociones.
Mediciones anatómicas detalladas demuestran que la formación hipocámpica, la amígdala y la
circunvolución parahipocámpica son más pequeñas de lo normal en los cerebros de los pacientes
esquizofrénicos, posiblemente debido a un crecimiento anormal de estas partes del cerebro.
Los agentes antipsicóticos van a ejercen una acción antagónica a la de la dopamina, que es el principal
neurotransmisor de las neuronas del área tegmental ventral que se proyectan a la amígdala, el núcleo
accumbens, la formación hipocámpica y la corteza prefrontal.; también bloquean receptores de
noradrenalina y serotonina. Los antipsicóticos del grupo de las dibenzodiazepinas. representadas por la
clozapina, ejercen una acción antagónica a la de la noradrenalina y la serotonina más fuerte que la de la
serotonina, Pueden causar parkinsonismo como efecto secundario. Un tratamiento prolongado puede
provocar también un trastorno del movimiento llamado discinesia tardía, que se caracteriza por
movimientos coreiformes de la lengua y los labios. A diferencia del parkinsonismo debido a efectos
secundarios, la D ISCINESIA TARDÍA frecuentemente persiste después de que el fármaco se haya retirado

MACRO T

¿En relación a los núcleos límbicos y de proyección inespecífica cuál de los siguientes núcleos
tiene como Aferencia las vías lentas del dolor? R// NCM

Empareja los contenidos según corresponda

Fibras simpáticas postganglionares esfínter interno anal PLEXO HIPÓGASTRICO


Tronco vagal anterior IZ QUIERDO
Fibras simpáticas preganglionares de la glándula suprarrenal NERVIO ESPLÁNICO
MAYOR
Esfínter vesical PLEXO HIPÓGASTRICO
Selecciona a que nivel encontramos los 3 ganglios simpáticos cervicales R// T1-T3 NIML
¿Las siguientes características a que fibras corresponde?

Corre por la parte medial de la capsula interna para ingresar a la estría medular R// Fibras
pa lidohabenulares

Caso clínico
Paciente masculino de 44 años, se presenta a consulta neurológica refiriendo que desde hace
una semana y sin ninguna razón aparte, ha perdido toda la sensibilidad el lado derecho del
cuerpo, refiere también tener poca sensibilidad en su rostro y que la comida no le está
sentando tan bien como de costumbre, bajo este cuadro el neurólogo a cargo del caso decide
ordenar la RM cerebral con la cual se diagnostica un infarto talámico, utilizando sus
conocimiento de neuroanatomía, responda donde está la lesión R// Tálamo izquierdo, nucleo
VPL y VPM

En relación con la división funcional de núcleos de la micos cual de los siguientes no corresponde
al grupo multimodal:D orsomedial

Lactante que es traído a la guardia en brazos de madre, al llegar el niño está irritable la madre
entrega al doctor una epicrisis en la que dice que el paciente ingresó de la sala de pediatría hace
un mes con el diagnóstico de enfermedad de Hirschprung. Haciendo uso de sus conocimientos
responda, que parte específica del tubo digestivo se encuentra enormemente distendida en El
Niño : Porción proximal del intestino grueso
Empareja los contenidos según corresponda.

Fibras simpáticas preganglionares de la glándula suprarrenal: nervio esplácnico mayor


Fibras simpáticas postganglionares esfínter interno anal: plexo hipogástrico
Tronco vagar anterior: izquierdo
Esfínter vesical: plexo hipogástrico

En los dermatoma T5-T8 Que vísceras encontramos: Hígado

Las proyecciones de cual de los siguientes núcleos se dirige fundamentalmente a la parte


posterior de la circunvolución del cuerpo calloso y al lóbulo parietal de la corteza
prelímbica:NCL
Señale la respuesta incorrecta Seleccione una

a. La arteria carótida interna da una Collateral para irrigar la adenohipófisis b. Un infarto


de la arteria cerebral media causaría sordera cortical
c. Un infarto de la arteria cerebral anterior nos podría causar un déficit motor en los
miembros inferiores
d. La batería cerebral posterior es rama de la arteria basilar

e. La arteria radicular de Adamkiewitzc Encontrará comúnmente en zonas lumbares de


lado izquierdo

Paciente masculino de 35 años de edad sin antecedentes enfermedad, acude a consulta por
deterioro progresivo de la memoria reciente y ligeramente de la pasada desde hace un año. No
refiere trastornos del sueño ni eventos estresantes. Dice tener una vida sana con buena
alimentación y actividad, sólo refiere que hace dos años nadando en el mar casi se ahoga y tuvo
que ser asistido con respiración boca a boca. por 10 minutos. Al examen físico neurológico
General se encuentra dentro de los parámetros normales, en los exámenes de laboratorio lo
único destacable es una hemoglobina rozando el límite inferior. Qué área del cerebro está
afectada: Área C A1 del hipocampo
Las proyecciones amígdala corticales a qué parte de la corteza específicamente llegan :
Somatosensorial y lóbulo temporal.

Paciente femenino de 32 años se presenta a la emergencia con un cuadro de dificultad para el


habla y movimiento, bajo este cuadro el interno de la guardería decide inter consulta a
neurología, se decide emplear un estudio imagino lógico y con éste se le diagnostica un infarto
de la PICA Izquierda, usando sus conocimientos de neuroanatomía responda cual de las
siguientes afirmaciones es correcta

a. Éste infarto nos podría causar síndrome de Dejerine

b. La s PICA no irriga las pirámides bulbares

c. El área de irrigación de la PICA se delimita a la porción postero inferior del

cerebelo

d. La PICA Esta rama directa de la arteria espinal posterior

Éste infarto nos podría causar una hemianopsia bitemporal La inervación autónoma del
páncreas es a cargo de:Plexo Celiaco

¿En relación con los núcleos límbicos y de proyección inespecífica cuál de los siguientes núcleos
tiene como Aferencias las vías lentas del dolor? NC M

¿Las proyecciones de cuál de los siguientes núcleos se dirige fundamentalmente a la parte


posterior de la circunvolución del cuerpo calloso y al lóbulo parietal a la corteza
perilímbica?Seleccione una:b. NLD

Paciente masculino de 44 años , se presenta a consulta neurológica refiriendo que desde hace
una semana y sin ninguna razón aparente ha perdido toda la sensibilidad el lado derecho del
cuerpo , refiere también tener poca sensibilidad en su rostro y que la comida no le está sentando
tan bien como de costumbre , bajo este cuadro el neurólogo a cargo del caso decide ordenar
una RM cerebral con la cual se diagnostica un infarto talámico , utilizando sus conocimientos de
neuroanatomía , responda dónde está la lesión. Tálamo Izquierdo, Núcleos VPL y VPM

Las fibras postganglionares simpáticas llegan a la glándula parótida por: Nervio


a uriculotemporal

¿Cuál de las siguientes vías constituyen una vía aferente y eferente del Hipotálamo? b. Haz
proscencefalico medial

El dolor referido apendicular inicialmente se da en el epigastrio debido a: Est imulación de las


fibras C viscerales que llegan al plexo celíaco y de ahí a segmentos toracolumbares.
¿Cuál de las siguientes estructuras se localiza en la parte ventral del tercer ventrículo en las
inmediaciones del receso infundibular extendiéndose a la eminencia media? N. Arciforme

¿En relación con la división funcional de núcleos talámicos cuál de los siguientes no corresponde
al grupo multimodal?D orsomedial

Pediátrico de 7 años que acude referido por presentar cambios de la voz que se hace mas grave
de la voz además refiere la mama esta con aparición de bellos en la región genital y la barba.
¿Según sus conocimientos donde está localizada la lesión del paciente? EN CASO DE RESPONDER
MAS DE UNA TOPOGRAFIA SE PUNTUARAA CON O PUNTOS Respuesta: Destrucción de la pineal
¿Cuál de los siguientes núcleos tiene como Aferencia el brazo del colículo inferior? NGM

¿Cuál de los siguientes núcleos tiene como Aferencia Lemnisco medial? VPL

En los dermatomas T5-T8 que viseras encontramos: Hígado

Paciente femenino de 32 años acude a la consulta refiriendo esterilidad, hipogonadismo


hipogonadotropo, amenorrea primaria. En la anamnesis se refería anosmia. En la RMN
cerebral se confirmó una hipoplasia severa de ambos bulbos olfatorios. Escriba el síndrome
que presenta el paciente. (solo una palabra) EN CASO DE RESPONDER MAS DE UN SINDROME
SE PUNTUARÁ CON O PUNTOS Respuesta: Kallman

¿Cuál de los siguientes núcleos tiene como Aferencia el área gustativa pontina? NVPM

Empareje los contenidos según corresponda


Agente que irrita la actividad autónoma parasimpática:melaconina
Neurotransmisor en la mayoría de las terminaciones posganglionares
simpáticas:Noradrenalina
Agente que irrita la actividad autónoma simpática: fenilefrina
Agente bloqueante sobre los receptores de las células efectoras
parasimpáticas:ATROPINA
Agente que bloquea los receptores de las células efectoras simpáticas: Propranolol

¿Cuál de las siguientes estructuras se localiza en la parte ventral del tercer ventrículo enlas
inmediaciones del receso infundibular extendiéndose a la eminencia media?
Seleccione una:N. ArciformeN

Selecciona la inervación simpática de las glándulas sudoríparas de la cabeza:Tl-T3 NIML

Lactante que es traído a la guardia en brazos de la madre, al llegar el niño está irritable de la
madre le entrega al Dr. una epicrisis en la que dice que el paciente egresó de la sala de pediatría
hace un mes con el diagnóstico de Enfermedad de Hirschsprung. Haciendo uso de sus
conocimientos responda, ¿Qué parte específica del tubo digestivo se encuentra enormemente
distendida en el niño? Respuesta: Parte proximal del colon

¿CUAL DE LAS SIGUIENTES NO ES UNA CARACTERÍSTICA COMÚN ENTRE LA ATETOSIS, LA


COREA Y EL BALISMO? Se manifiestan con marcada rigidez
¿Cuál de los siguientes enunciados es incorrecto?

A. La s fibras estriadonigras se originan en una población de neuronas no espinosas


diferente a las estriadopalidales, pero comarten los mismos t ransmisores
B. Las fibras estriadopalidales inmunorreactivas a encefálica se localizan en GPL

C. Las fibras eferentes del globo pálido se dividen en 4 haces

D. las grandes neuronas del globo pálido pertenecen a una sola población neuronal

E. Las fibras subtálamo paliadles ejercen efecto excitador sobre el globo palido

Paciente de 90 años que acude por dolores de cabeza de mayor intensidad en la noche, se le
realiza una angiografía cerebral que reporta aneurisma de gran tamaño en la unión de la
comunicante y la cerebral posterior. Cuando lo examinan entre otros se encuentra dilatación
de la pupila ipsilateral con pérdida del reflejo fotomotor¿Qué fibras autónomas están
afectadas en nuestro por esta lesión? Respuesta: fibras parasimpáticas preganglionares que
viajan por la periferia del nervio

La inervacion secretomotora de la mucosa nasal es a cargo de:nucleo salivatorip superior

Paciente femenino de 32 años, se presenta a la emergencia con un cuadro de dificultad para el


habla y movimiento, bajo este cuadro el interno de la guardia decide interconsulta a neurología,
se decide emplear un estudio imagenológico y con este se le diagnostica un infarto de la PICA
izquierda, usando sus conocimientos de neuroanatomía, responda cuál de las siguientes
afirmaciones es correcta.

Seleccione una:

a.- la PICA es rama directa de la arteria espinal posterior

B .- Las PICA no irriga a las pirámides bulbares.

C.- Este infarto nos podría causar una hemianopsia bitemporal.

D.- El área de irrigación de la PICA se delimita a la porción posterioinferior del cerebelo. E.-
Este infarto nos podría causar un síndrome de dejerine.

Paciente masculino de 35 años de edad sin antecedentes de la enfermedad, acude a consulta


por deterioro progresivo de la memoria reciente y ligeramente de la pasada desde hace 1 año,
no refiere trastornos del sueño no eventos estresantes dice tener una vida sana con buena
alimentación y actividad, solo refiere que hace dos años nadando en el mar casi se ahoga y tuvo
que ser asistido con resporacion boca a boca, por 10 minutos. Al examen físico neurológico y
general se encuentra dentro de los parámetros normales, en los exámenes de laboratorio lo
único destacable es una hemoglobina rozando el interior inferior. ¿Qué área del cerebro está
afectada? Respuesta: Sector CA1 hipocampo.

Paciente masculino de 25 años previamente sano que presenta un cuadro de 7 meses de


evolución caracterizado por taquicardia, hipertensión arterial, debilidad muscular proximal,
estrías cutáneas abdominales, fascie de luna llena, joroba en la espalda, visión borrosa,
alteraciones neurológicas, osteomusculares, cutáneas y auditivas. Se le realizan pruebas
diagnosticas que orientan hacia una posible causa hipofisiaria y se decide realizar una
resonancia magnética cerebral que revela una masa de 6 mm en adenohipófisis que deforma
ligeramente la estructura de la glándula hipofisiaria: Enfermedad de Cushing

¿ Qué estructura del SN g enera como respuesta a su estimulación la liberación de g randes


ca ntidades de ACTH y hormona gonodotrófica con respuesta lactógena?

Señale la respuesta incorrecta Selecciones una:

A.- La arteria cerebral posterior es rama de la arteria basilar

B.- La arteria radicular de Adamkiewitzc se encontrará comúnmente en zonas lumbares del


lado izquierdo.

C.- La arteria carótida interna da una colateral para irrigar la adenohipófisis.

D .- Un infarto de la a rteria cerebral media causaría sordera cortical.

E.- Un infarto de la arteria cerebral anterior nos podría causar un déficit motor en los
miembros inferiores.

Paciente masculino de 55 años que presenta una trombosis de la carótida interna, producto de
un trauma de la región anterior del cuello, como consecuencia de una golpiza, al examen físico
el paciente se encuentra con una PA de 140/95, glucemia de 92 y FC de 120, al examen
neurológico se constató obnubilación, hemiplejia izquierda, y un estrabismo de carácter
convergente en el ojo derecho, con estos datos, en qué lugar del recorrido de la arteria carótida
interna se encuentra la lesión. Seleccione una: En su porción cavernosa

¿Las proyecciones amigdalacorticales a que parte de la corteza específicamente llegan?


Respuesta:

Paciente masculino de 45 años, acude a la emergencia refiriendo un dolor de cabeza y falleció


súbitamente, en la autopsia se observa un daño en la zona señalada del encéfalo, usando su
conocimiento de neuroanatomia. Responda ¿Cuál de las siguientes afirmaciones es correcta?
Respuesta: La arteria que irriga a esta zona también irriga al á rea 41 de brodman

¿Las proyecciones de cuál de los siguientes núcleos se dirige fundamentalmente a la parte


posterior de la circunvolución del cuerpo calloso y al lóbulo parietal a la corteza perilímbica?
Respuesta: NLD (núcleo lateral dorsal)
La inervación secretomotora de la mucosa nasal es a cargo de: Núcleo salivatorio superior
¿En relación a los núcleos límbicos y de proyección inespecífica cuál de los siguientes núcleos
tiene como aferencias las vías lentas del dolor?
Respuesta: NCM (núcleo centromediano)

Hombre de 47 años con antecedentes de diabetes e hipertensión arterial, además, dentro de su


familia, existieron casos de fallecimiento por cardiopatías y un caso aislado de colesterolemia.
Meses atrás sufrió un accidente de tránsito que lo llevó a un cuadro de hemiplejía braquiocrural
izquierda, con parálisis facial periférica derecha por lo que fue intervenido quirúrgicamente y
fue dado de alta una semana después de la práctica quirúrgica al no presentar sintomatología
alguna. Luego de tres meses, sufrió una caída que provocó una herida debajo de la oreja, al cabo
de dos semanas, acude a realizarse chequeos de rutina, sin embargo, al pasar por la sala de
comidas del hospital comenzó a llorar en el lado derecho de la cara y manifesto que, al leer la
prensa en horario matutino, las hojas acababan con varias gotas encima. Qué estructura se
encuentra afecta en el paciente? Respuesta: Lesión de la s fibras pa rasimpáticas
posganglionares del nervio a uriculotemporal - a uricular mayor y facial derechos
La irrigación de la paratiroides superior izquierda está a cargo de: Arteria tiroidea superior
iz quierda

Paciente femenino de 32 años, se presenta con un cuadro de dificultad para el habla y


movimiento, bajo este cuadro, el interno de guardia decide interconsulta a neurología, se decide
emplear un estudio imagenológico y con este se le diagnostica un infarto de la PICA izquierda,
usando sus conocimientos de neuroanatomía, cuál de las siguientes afirmaciones es correcta.
Respuesta: La PICA no irriga las pirámides bulbares.

Paciente masculino de 35 años de edad sin antecedentes de enfermedad, acude a consulta por
deterioro progresivo de la memoria reciente y ligeramente de la pasada desde hace 1 año, no
refiere trastornos del sueño ni eventos estresantes, dice tener una vida sana con buena
alimentación y actividad, solo refiere que hace dos años nadando en el mar casi se ahoga y tuvo
que ser asistido con respiración boca a boca, por 10 minutos. Al examen físico neurológico y
general se encuentra dentro de los parámetros normales, en los exámenes de laboratorio lo
único destacable es una hemoglobina rozando el límite inferior. Qué área del cerebro se
encuentra afectada? Respuesta: Sector CA1 - hipocampo

La inervación autónoma del páncreas está a cargo de: Respuesta: plexo celíaco

La glándula tiroides está situado en una vaina fascial que pertenece al sistema de las fascias del
cuello. Esta vaina está constituida de la siguiente manera : Anteriormente por la hoja
profunda de la lámina pretraqueal de la fascia cervical, posteriormente por la vaina visceral y sus
extensiones.
¿Cuál de los siguientes vasos debería estar infartado para afectar VA, VL, DM,
AV?Tuberotalamico

¿Las siguientes características a que fibra corresponde? Corre por la parte medial de la capsula
interna para ingresar a la estría medular: Fibras palidohabenulares

Agente bloqueante sobre los receptores de las células efectoras parasimpáticas - Atropina.

agente que bloquea los receptores de las células efectoras simpáticas - Propranolol

Agente que irrita la actividad autónoma simpática - fenilefrina.


Neurotransmisor en la mayoría de las terminaciones posg anglionares simpáticas -
Noradrenalina.
Agente que irrita la actividad autónoma parasimpática – metacolina

cual de los siguientes vasos debería estar infartado para afectar VA, VL, DM, AV. t uberotalamico

enunciado incorrecto: las fibras estrionigras se originan en una población de neuronas no


espinosas diferentes a la estriadopalidales, pero comparten los mismos neurotransmisor es.
adolescente de 17 años que acude por presentar evidente retraso de la pubertad, no presenta
caracteres sexuales secundarios y los genitales tienen aspecto infantil, además de ello su voz es
muy aguda, por lo que sufre de bullying en la escuela. D ONDE ES LA LESIÓN: Glándula pineal.

En los dermatomas T5-T9 que viscera encontramos: Piloro

En relacion con los nucleos limbicos y de proyeccion inespecifica,cual de los siguientes nucleos
tiene como aferencias el fasciculo mamilotalamico: NA

Cual de las siguientes no es una caracteristica comun entre la atetosis , la corea y el balismo:
Se ma nifiestan con marcada rigidez

Las siguientes características a que fibras corresponde?

Corre por la parte medial de la cápsula interna para ingresar a la estria medular: Fibras
pa lidohabenulares
¿Cuál de las siguientes estructuras se localiza en la parte ventral del tercer ventrículo en las
inmediaciones del receso infundibular extendiéndose a la eminencia media? R// N. Arciforme

Paciente de 90 años que acude por dolores de cabeza de mayor intensidad en la noche, se le
realiza una angiografía cerebral que reporta aneurisma de gran tamaño en la unión de la
comunicante y la cerebral posterior. Cuando lo examinan entre otros se encuentra dilatación de
la pupila ipsilateral con pérdida del reflejo fotomotor. ¿Qué fibras autónomas están afectadas
en nuestro por esta lesión? R// Fibras parasimpáticas preganglionares del nervio oculomotor.

¿En relación con los núcleos límbicos y de proyección inespecífica cuál de los siguientes
núcleos tiene como Aferencias las vías lentas del dolor? NCM(Núcleo centromediano)

¿Cuál de los siguientes núcleos tiene como Eferencia la corteza premotora? NVA(Núcleo
ventral a nterior)

Selecciona la inervacion simpatica de las glándulas sudoríparas de la cabeza T1- T3 NIML

¿Cuál de los siguientes núcleos tiene como aferencia el área gustativa pontina? NVPM (Núcleo
ventral posteromedial)

Las fibras postganglionares simpáticas llegan a la glándula parótida por: Nervio


a uriculotemporal

¿Cuál de los siguientes núcleos tiene como aferencia el Lemnisco medial? VPL (Núcleo ventral
posterolateral)

¿Cuál de los siguientes enunciados es incorrecto?


Respuesta: Las fibras estriadonigras se originan en una población de neuronas no espinosas
diferente a la estriadopalidales, pero comparten los mismos neurotransmisoress

¿Cuál de los siguientes núcleos tiene como aferencia el brazo del colículo inferior? Respuesta:
NGM

Lactante que es traído a la guardia en brazos de la madre al llegar el niño está irritable la madre
le entrega al doctor. Una epicrisis en la que dice que el paciente egresó de la sala de pediatría
hace un mes con el diagnóstico de enfermedad de Hirschprung. Haciendo uso de su
conocimiento responda qué parte específica del tubo digestivo se encuentra enormemente
distendida en el niño? Respuesta: La parte proximal del colon

Las proyecciones de cuál de los siguientes núcleos se dirige fundamentalmente a la parte


posterior de la circunvolución del cuerpo calloso y el lóbulo parietal a la corteza perilímbica?
Respuesta: NLD

En relación con los núcleos límbicos y de proyección inespecífica cuál de los siguientes núcleos
tiene como aferencia el fascículo mamilotalámico? Respuesta: NA

Paciente femenino de 32 años acude a la consulta refiriendo esterilidad , hipogonadismo


hipogonadotropo, amenorrea primaria. En la anamnesis se refería anosmia.En la RMN cerebral
se confirmó una hipoplasia severa de ambos bulbos olfatorios. Respuesta: Síndrome de
KALLMAN

Pediátrico de 7 años que acude referido por presentar cambios en la voz que se hace más grave
de la voz además refiere la mama esta con aparición de vellos en la región genital y la barba
¿Según sus conocimientos anatómicos donde esta localizado la lesión del paciente? R//Glándula
pineal

las proyecciones amigdalocorticales a que parte de la corteza específicamente llega?


somatosensorial y lobulo temporal

Señale la respuesta incorrecta

la A. carotida interna da una colateral para irrigar


la adenohipofisis
Un infarto de A. cerebral media causaría sordera cortical
la A. cerebral posterior es rama de la A. basilar
la A. radicular de ADAMKIEWITZC se encontrara en zonas lumbares de lado izq.
Un infarto de la A. cerebral ant. nos podría causar un déficit motor en los
miembros inferiores

Paciente masculino de 35 años de edad sin antecedentes de enfermedad, acude a consulta por
deterioro progresivo de la memoria reciente y ligeramente de la pasada desde hace 1 a ño, no
refiere trastornos del sueñño ni evento estresantes dice tener una vida sana con buena
alimentación y actividad, solo refiere que hace 2 años nadando en el mar casi se ahoga y tuvo
que ser asistido con respiración boca a boca, por 10 minutos. al examen fisico neurologico y
general se encuentra dentro de los parametros normales, en los examenes de laboratorio lo
unico destacable es una hemoglobina rozando el limite inferior. que area del cerebro esta
afectada? C A1, Hipocampo
EN RELACION CON EL COMPLEJO AMIGDALINO TODOS SON CORRECTOS EXCEPTO: Las fibras
que se originan en la mitad rostral del hipotalamo pasan a todos los nucleos amigdalinos

la inervacion autonoma del pancreas es a cargo de Plexo celiaco

paciente masculino de 45 años acude a la emergencia refiriendo un dolor de cabeza y fallece


subitamente, en la autopsia se observa un daño en la zona señalada del encéfalo, usando su
conocimiento en neuroanatomia, responda la que es correcta: la a rteria que irriga a esta zona
t a mbién irriga al á rea 41 de brodman

Un paciente llega a urgencias con una herida corto punzante a nivel de T10, d acuerdo a sus
conocimientos anatómicos indique que nervio se encuentra lesionado:NERVIO ESPLÁCNICO
MENOR

La oclusión de la arteria cerebral posterior genera que manifestación clínica:HEMIANOPSIA


HOMÓNIMA CONTRALATERAL (SIN AAFECTACION A LA MACULA)

Cual o cuales pertenecen a la división del núcleo ventral posterior (VP):VENTRAL


POSTEROLATERAL, VENTRAL POSTEROMEDIAL

La vena cerebral interna es formada por la union de:VENA TALAMOESTRIADA Y VENA


C OROIDEA

Relaciona los conocimientos del ritmo circadiano:

21H00: COMIENZO DE LA SECRECION DE MELATONINA


18H30: MAYOR ELEVACION DE LA PRESION SANGUINEA
19H00: TEMPERATURA CORPORAL MAS ALTA

Cual de las siguientes características no pertenecen al síndrome de Horner: MID RIASIS

¿Selecciona cual de los siguients recorridos es correcto para la carrotida interna desde que
atraviesa el canal carotideo hasta que se divide en cerebral media y cerebral anterior? C ANAL
C AROTIDEO, SIFON CAROTIDEO, SENO VENOSO, GIRO HACIA ARRIBA, APOFISIS CLINOIDE
ANTERIOR, ESPACIO SUBARACNOIDEO, PERFORA DURAMADRE Y ARACNOIDES, PASA DEBAJO DEL
NERVIO OPTICO, GIRA HACIA ARRIBA LATERAAL AL QUIASMA OPTICO, SE DIVIDE DE ACM Y ACA

Relaciona los conocimientos anatómicos, según corresponda

TRONCO VENOSO LP: V.POSTEROMEDIAL V. POSTEROLATERAL


RAMAS CORTICALES DE ACM DE MAYOR TAMAÑO: AREA ANGULAR
A.FRONTOPOLAR: A. C ALLOSOMARGINAL
Paciente con trastornos en la fisiología del sueño posterior a una lesión isquémica del tejido
cerebral, se examina y se miden valores de melatonina bajos en sangre. ¿Qué vasos sanguíneos
estaría ocluido?ARTERIA CENTRAL POTEROLATERAL

Tenemos a alguien que desea aprender a tocar guitarra. Al comienzo es incapaz de tocar
correctamente las notas de la canción, pero con el tiempo va mejorando y se ve capaz de tocar
sin tener que pensar que cuerdas tocar, de manera autónoma. ¿Qué es lo que permite al
guitarrista adquirir este nivel de maestria con la guitarra? LA POTENCACION A LARGO PLAZO

Que enfermedad esta relacionada con las manifestaciones clínicas siguientes: ateraciones de la
sudoración – a lteraciones pupilares – hipotensión postural – edema periférico :DIABETES
MIELLITUS
¿Qué nucleos están encargados de la respuesta de reclutamieto? NUCLEO VENTRAL ANTERIOR Y
NUCLEOS INTRALAMINARES

Cual de los siguientes elementos anatómicos constituye el limite anterior del tercer venticulo:

LAMINA TERMINAL

¿Dónde se ubica el paraganglio cardiaco? CORONARIA IZQUIERDA


¿Cuál de las siguientes características se le atribuye a la zona incierta? ESTIMULA LA SED

Paciente masculino de 52 años que presenta las siguientes manifestaciones clínicas tras haber
sufrido un accidente cerebrovascular. Sx de Horner ipsilateral, ataia, paraisis de paladar y la
cuerda vocal ipsilaterales, perdida de sensaciones térmicas y nocioceptivas ipsilateral y del lado
contralateral del cuerpo. La RMN cerebral es la siguiente. Utilizando sus conocimientos de
neuroanatomía que síndrome presenta el paciente y que arteria esta afectada?SINDROME
WALLENBERG Y ARTERIA CEREBELOSA POSTEROINFERIOR

Paciente es llevado a urgencias por presentar un accidente cerebrovascular, y en la evalucacion


presente lo siguiente: parálisis del hipogloso ipsilatera, hemiplejia contra lateral, perdida de la
propiocepción y vibración contralateral. Según sus conocimientos en neuroanatomía cual o
cuales de los siguientes enunciados son correctos: SE ENCUENTRA AFECTADA LA ARTERIA
ESPINAL ANTERIOR, SE PRESENTA SX B ULBAR MEDIAL, ESTAN AFECTADAS LAS PIRAMIDES
B ULBARES.
¿Qué grupo talámico esta ubicado debajo de la cara dorsal de la parte más rostral del tálamo?

GRUPO NUCLEAR ANTERIOR

El grupo posterolateral de las arterias centrales del encéfalo van a irrigar:

TALLAMO (PARTES POSTERIORES, INCLUIDOS LOS NUCLEOS GENICULADOS) Y MESENCEFALO


(TECTUM Y PARTE LATERAL DEL PEDUNCULO C EREBRAL)

Al existir una oclusión en la arteria cerebral media, ¿Qué manifestaciones presenta el paciente?

HEMIPLEJIA CONTRALATERAL ACENTUADA EN LAS EXTREMIDADES SUPERIORES Y EN LA CARA,


AFASCIA ACENTUADA SI SE C OMPROMETE EL HEMISFERIO D OMINANTE, PERDIDA
C ONTRALATERAL DE LA SENSIBILIDAD, DE LA POSICION Y EL TACTO D ISCRIMINATIVO

Paciente llega a la sala de emergencias por una contusión fuerte en la cabeza causada por un
accidente de tráfico. Después de hacerse una resonancia magnética, se puede observar la
destrucción total de ambos cuerpos mamilares. ¿Que complicación tendrá el paciente?
AMNESIA
Esfínter de la pupila tiene inervasion parasimpática procedente de que núcleo: NUCLEO D E
ED INGER-WESTPHAL

¿Cuáles de los siguientes vasos irriga la parte superior del tegmento protuberancial del
pedúnculo cerebeloso superior y el colículo inferior del mesencéfalo? RAMAS PROXIMALES DE LA
ARTERIA CEREBELOSA SUPERIOR

La arteria recurrente de Haubner es rama de: ARTERIA C EREBRAL ANTERIOR

Relaciona los conocimientos del ritmo circadiano:

04H30: TEMPERATURA CORPORAL MAS BAJA


02H00: SUEÑO MAS PROFUNDO
00H00: SUPRIME LA SENSACION DE SED
¿Qué estructura nerviosa regula las actividades rítmicas del cerebro y el sistema endocrino?
NUCLEO SUPRAQUIASMATICO

¿Qué nervio se forma por la unión de las fibras postganglionares de la glandula lagrimal y nervio
maxilar? NERVIO CIGOMATICO

¿Cuáles son los medios de fijación de la glandula tiroides? VASOS PARATIROIDEOS, VENA
PARATIROIDEA

¿De donde provienen las fibra amielínicas que atraviesan el lóbulo posterior de la glandula
hipófisis? INFUNDIBULO
Relaciona los conocimientos del dolor referido:

ESTOMAGO: HOMBRO DERECHO


RIÑON: NEUROGLIA INTERCOSTAL DE LAS 12 COSTILLA HASTA EL OMBLIGO
PULMONES: TRAPECIO DERECHO
ESOFAGO: ENTRE LAS ESCAPULAS
Respecto al triangulo de Silvio, cual o cuales serían los enunciados correctos:

SE FORMA EN LA REGION INSUALAR, LA RAMA POSTERIOR DDE ACM FORMA EL PUNTO DE


SILVIO CUANDO EMERGE POR LA C ISURA LATERAL

Sobre los ganglios linfáticos correspondientes a la glandula suprarrenal derecha cual es correcto:
LOS ANTERIORES EN NUMERO DE 4 A 5
¿Cuál no es un limite del cuadrilatero de la suprarrenal?ARTERIA SUPRARRENAL SUPERIOR

¿Qué arteria irriga el lobulillo paracentral?ARTERIA CEREBRAL ANTERIOR

¿la porción supramesocolica del páncreas porque órgano está cubierta?HIGADO

¿Cuáles de los siguiente grupos arteriales seria el responsable de producir isquemia masiva de
los pares craneales motores contralaterales si se infarta? GRUPO ANTEROLATERAL
El hipotálamo es un centro anatomico muy importante en el ciclo de gonadotropina, seleccione
cual de los siguientes enunciados es una condición indispensable para que cumpla este
ciclo:INTEGRIDAD DE LA ADENOHIPOFISIS

37. ¿Cuál de los siguientes orígenes de fibras simpáticas corresponde a la inervación del
cuello?T2

¿Cuál de los siguientes no forma parte de los nucleos que contienen la sustancia inominada? N.
PRESEPTAL

Relaciona los elementos neuroanatómicos

PLEXO LUMBAR Y MESENTERICO INFERIOR: COLON DESCENDENTTE Y RECTO


GANGIO SIMPATICO CERVICALL INFERIOR Y TORAACICO SUP: T1-T5
PLEXO CELIACO Y MESENTERICO SUPERIOR: T5-L1
GLANDULAS SUDORIPARA DE LA CABEZA T1-T3
GANGLIO OTRICO: GLANDULA PAROTIDA

Relaciona los elementos neuroanatómicos según corresponda:

PULVINAR: EFERENCIA (AMIGDALA)


VA: PLANIFICACION MOTORA Y C OMPORTAMIENTO COMPLEJO
NUCLEOS DEL GRUPO LATERAL: FORMACION DE RESPUESTAS C ONDUCTUALES
C OMPLEJAS
NUCLEOS DEL GRUPO ANTERIOR: (AFERENCIA) CUERPO MAMILAR

¿Cuál de los siguientes no forma parte de los componentes del circuito de Papez? AREA
TEGMENTAL VENTRAL
¿Dónde se ubican los ganglios simpáticos en la región pélvica?ANTERIOR AL SACRO

Hacia donde se dirigen las fibras aferentes del seno carotideo :NUCLEO SOLITARIO

Paciente es diagnosticado con un comportamiento pasivo, con inestabilidad emocional,


ansiedad, alto interés en la comida e hipersexualidad. No se observan trastornos de la memoria.
¿cuall o cuales de las siguientes puede haber causado un estado asi en el pacient: LA
D ESTRUCCION UNILATERAL O BILATERALDEL CUERPO AMIGDALINO, EXTIRPACION DE AMBOS
LOBULOS TEMPORALES
Dolor somatico apendicular en que dermatomas se localiza:T12 Y L1

¿el infarto de cuales de los siguientes grupos de arterias afectaría la via piramidal?GRUPO
POSTEROLATERAL, POSTEROMEDIAL Y ANTEROLATERAL

¿Qué estructura forman la confluencia de ambos troncos simpáticos? GRUPO IMPAR

¿Cuál de los siguientes grupos de arterias centrales seria el responsable del desarrollo de
ceguera y sordera simultánea si se infartara, todo ello por una lesión directa en una estructura
de relevo de ambas vías? GRUPO POSTEROLATERAL

Un paciente masculino se somete a un tratamiento experimental contra la epilepsia, en este


tratamiento se le extirparon ambos lobulos temporales. Como consecuencia secndaria de esta
operación ¿Qué complicación cree usted que tendrá este paciente? SINDROME DE KLUVER-
B UCY
¿Dónde vierte su secreción la glandula hipófisis? C APILARES SANGUINEOS

Relaciona los conocimientos neuroanatómicos (vascular):

V BASAL: C INTILLA OPTICA


VCM PROFUNDA: SUPERFICIE ANTERIOR DEL ENCEFALO
VCM SUPERFICIAL: VENA DE LABBÉ
V TERMINAL: C ISURA CEREBRAL TRANSVERSA

Los filetes nerviosos del lóbulo anterior de la glandula hipófisis a que sistema pertenecen:
VEGETATIVO

Con respecto a la glandula pineal ¿Cuál nervio se encarga del suministro nervioso aferente del
ganglio cervical superior? NERVIO C ORONARIO

Cual o cuales de los siguientes pertenecen a los síntomas de la enfermedad de RAYNAUD:


C IANOSIS DE LOS DEDOS, D OLOR AGUDO
Paciente masculino de 45 años, acude a la emergencia refiriendo un dolor de cabeza y fallece
súbitamente, en la autopsia se observa un daño en la zona señalada de encéfalo, usando su
conocimiento de neuroanatomía, responda cual de las siguientes afirmaciones es correcta:LA
ARTERIA QUE IRRIGA ESTA ZONA TAMBIEN IRRIGA AL AREA 41 D E BRODMAN

Paciente masculino de 35 años de edad sin antecedentes de enfermedad, acude a consulta por
deterioro progresivo de la memoria reciente y ligeramente de la pasada desde hace 1 año, no
refiere trastornos del sueño ni eventos estresantes dice tener una vida sana con buena
alimentación y actividad, solo refiere que hace dos años nadando en el mar casi se ahoga y tuvo
que ser asistido con respiración boca a boca, por 10 min. Al examen físico neurologico y general
se encentra dentro de los parámetros normales, en los exámenes de laboratorio lo único
destacable es una hemoglobina rozando el limite inferior. ¿Qué área del cerebro está afectada?:

¿las proyecciones amigdalocorticales a que parte de la corteza específicamente


llegan?PREFONTRAL

Paciente masculino de 55 años que presenta una trombosis de la carótida interna, producto de
un trauma de la región anterior del cuello, como consecuencia de una golpiza, al examen físico
del paciente se encuentra con PA de 140/95, glucemia de 92 y FC de 120, al examen neurológico
se constató obnubilación, hemiplejia izquierda, y estrabismo de carácter convergente en el ojo
derecho, con esos datos, en que lugar del recorrido de la carotida interna se encuentra la
lesión:EN SU PORCION CAVERNOSA
En relación a las venas tiroideas superiores, todas son incorrectas menos : D RENAN ENEL
TRONCO TIROLINGO FACIAL Y D IRECTAMENTE EN LA VENA YUGULAR INTERNA
¿Qué estructura del SN genera como respuesta a su estimulación la liberación de grandes
cantidades de ACTH y hormona gonadotropa con respuesta lactógena?

Inervacion autónoma del pancreas es a cargo de? PLEXO CELIACO

Paciente femenino de 32 años, se presenta a la emergencia con un cuadro de dificultas para el habla
y movimiento, bajo este cuadro el interno de la guardia decide interconsulta a neurología, se decide
emplear un estudio imagenológico y con este se diagnostica un infarto de la PICA izquierda,
usando sus conocimientos de neuroanatomía, responda cual de las siguientes afirmaciones es
correcta.LAS PICA NO IRRIGA A LAS PIRÁMIDES BULBARES

Paciente masculino de 25 años previamente sano que presenta un cuadro de 7 meses de


evolución caracterizado por taquicardia, hipertensión arterial, debilidad muscular proximal, estrías
cutáneas abdominales, fascie de luna llena, joroba en la espalda, visión borrosa, alteraciones
neurológicas, osteomusculares, cutáneas y auditivas. Se le realizan pruebas
diagnosticas que orientan hacia una posible causa hipofisiaria y se decide realizar una resonancia
magnética cerebral que releva una masa de 6mm en adenohipófisis que deforma ligeramente la
estructura de la glándula hipofisiaria. Mencione patología que presenta el
paciente:ENFERMEDAD DE CUSHING

La glandula tiroides esta situada en una vaina fascial que pertenece al sistema de las fascias del
cuello. Esta vaina esta constituida de la siguiente manera: ANTERIORES POR LA HOJA
PROFUNDA DE LA LAMINA PRETRAQUEAL DE LA FASCIA CERVICAAL, POSTERIORMENTE POR LA
VAINA VISCERAL Y SUS EXTENSIONES
¿Cuál de los siguientes enunciados es incorrecto?

LAS FIB RAS ESTRIADONIGRAS SE ORIGINAN EN UNA POBLACION D E NEURONAS NO


ESPINOSAS D IFERENTE A LA ESTRIOPALIDALES, PERO C OMPARTEN LOS MISMOS
NEUROTRANSMISORES

En relación con los nucleos límbicos y de proyección cual de los siguientes nucleos tiene como
aferencias el fascículo mamilotalámico: NA

La irrigación de la partiroides superior izquiera esta a cargo de: A.t iroidea superior izquierda

Adolescente 17 años que acude por presentar evidente retraso de la pubertad, no presenta
caracteres sexuales secundarios y los genitales tienen aspecto infantil, además de ella su vez es muy
aguda, por lo que está sufriendo bullying en la escuela. Según sus conocimientos escriba en que parte
del SN esta lesión: pineal (pinealoma parenquimatoso)
miércoles, 17 de febrero de
Comenzado el
2021, 21:00
Estado Finalizado
miércoles, 17 de febrero de
Finalizado en
2021, 21:13
Tiempo empleado 13 minutos
Cali!cación Sin cali!car aún

Pregunta 1 Finalizado

Puntúa 0,00 sobre 1,00

Marcar pregunta

Con respecto a la irrigación del páncreas,


mencione de donde proviene la arteria
pancreatoduodenal inferior.

Seleccione una:

a. Directamente de la aorta abdominal

b. Arteria gastroduodenal

c. Ramas de la arteria suprarrenal superior


Ramas de la arteria suprarrenal superior

Ramas de la arteria suprarrenal superior

Ramas de la arteria suprarrenal superior

d. Arteria esplénica

e. Arteria mesentérica superior

La respuesta correcta es: Arteria mesentérica


superior

Pregunta 2 Finalizado

Puntúa como 1,00 Marcar pregunta

Una mujer de 25 años consultó a su médico


porque experimentaba episodios de cambio de
color acompañado de dolor en los dedos cuarto y
quinto de ambas manos. Al principio sus dedos se
volvían blancos por la exposición al frío y luego
adquirían un color azul oscuro. El cambio de color
se limitaba a la mitad distal de cada dedo y se
acompañaba de un dolor agudo. El único
tratamiento que le aliviaba el dolor consistía en
mantener las manos cerca de una estufa caliente
o entrar en una habitación cálida. La paciente le
comentó al médico que había notado que sus
dedos estaban húmedos de sudor durante
algunos de los episodios. Utilice sus
conocimientos de neuroanatomía y formule el
diagnóstico.

La paciente tienen una falla de pigmentación en como


respuesta sensorial por la temperatura.

Pregunta 3 Finalizado

Puntúa 0,00 sobre 1,00

Marcar pregunta

Las !bras poscomisurales de la estría terminal


terminan en?

Seleccione una:

a. Núcleo aumbens

b. GPi

c. Área preóptica medial

d. Área pretectal

e. Corteza frontal

La respuesta correcta es: Área preóptica medial

Pregunta 4 Finalizado

Puntúa 0,00 sobre 1,00

Marcar pregunta

Cual de las siguientes a!rmaciones es correcta:

Seleccione una:

a. La arteria cerebral media derecha va a irrigar


en la mayoría de personas , al área 22 , 44 y 45 de
brodman

b. La arteria radicular de adamkiewicz surge en


la mayoría de ocasiones en zonas lumbares altas
del lado derecho

c. La arteria radicular de adamkiewicz surge en


la mayoría de ocasiones en zonas lumbares altas
del lado izquierdo

d. La arteria de heubner es rama de la cerebral


media

e. La arteria cerebral posterior es la primera


rama de la arteria basilar

La respuesta correcta es: La arteria radicular de


adamkiewicz surge en la mayoría de ocasiones
en zonas lumbares altas del lado izquierdo

Pregunta 5 Finalizado

Puntúa 0,00 sobre 1,00

Marcar pregunta

Escoja la alternativa correcta

Seleccione una:

a. La arteria comunicante posterior es una rama


de la arteria cerebral media.

b. La arteria oftálmica es una rama de la arteria


cerebral media.

c. Las arterias pontinas son ramas de la arteria


carótida interna.

d. La arteria espinal posterior procede de la


arteria vertebral.

La respuesta correcta es: La arteria espinal


posterior procede de la arteria vertebral.

Pregunta 6 Finalizado

Puntúa 1,00 sobre 1,00

Marcar pregunta

Cual de las siguientes estructuras es reconocida


por su propiedad de presentar calci!caciones?

Seleccione una:

a. Adenohipó!sis

b. Glándula pineal

c. Neurohipó!sis

d. Bulbo olfatorio

e. Hipó!sis

La respuesta correcta es: Glándula pineal

Pregunta 7 Finalizado

Puntúa como 1,00 Marcar pregunta

Que arteria se localiza proximal o distal a la


arteria comunicante anterior, recorre un trayecto
de dirección caudal y lateral e ingresa al espacio
perforado anterior, este vaso irriga la parte
anteromedial de la cabeza del núcleo caudado, las
partes adyacentes de la cápsula interna y el
putamen, y partes de los núcleos septales.

DE ESCRIBIR MÁS DE UNA ESTRUCTURA


VASCULAR SE ANULARÁ LA RESPUESTA.

Arteria cerebral media

Pregunta 8 Finalizado

Puntúa 1,00 sobre 1,00

Marcar pregunta

Las siguientes estructuras forman parte del


sistema límbico, excepto:

Seleccione una:

a. Hipocampo

b. Área 32, 24 de Brodmann

c. Locus niger

d. Corteza del cíngulo

e. Núcleo accumbens

La respuesta correcta es: Locus niger

Pregunta 9 Finalizado

Puntúa 0,00 sobre 1,00

Marcar pregunta

En relación al Asa motora, seleccione cual de las


siguientes secuencias corresponde a la vía
directa.

Seleccione una:

a. Corteza cerebral....Putamen...........Núcleo
pedúnculopontino.....Tálamo....Corteza

b. Corteza cerebral....Putamen......GPi
MNc.........Núcleo
pedúnculotalámico.....Tálamo....Corteza

c. Corteza cerebral........GPi MNc.........Núcleo


pedúnculopontino.....Tálamo....Corteza

d. Corteza cerebral....Putamen......GPi
MNr.........Núcleo pedúnculopontino.........Corteza

e. Corteza cerebral....Putamen......GPi
MNr.........Núcleo
pedúnculopontino.....Tálamo....Corteza

La respuesta correcta es: Corteza


cerebral....Putamen......GPi MNr.........Núcleo
pedúnculopontino.....Tálamo....Corteza

Pregunta 10 Finalizado

Puntúa 0,00 sobre 1,00

Marcar pregunta

Cual de los siguientes núcleos talámicos cumple


una función descrita de la siguiente manera:
“Respuestas asociadas al comportamiento . que
implican decisiones basadas en la predicción y en
los incentivos”.

Seleccione una:

a. Núcleo reticular del tálamo

b. Núcleo mediodorsal del tálamo

c. Núcleos intralaminares del tálamo

d. Núcleo dorsomedial del tálamo

e. Pulvinar

La respuesta correcta es: Núcleo mediodorsal del


tálamo

Finalizar revisión

* Navegación por el cuestionario

1 2 3 4 5 6 7 8 9

10

Mostrar una página cada vez

Finalizar revisión
miércoles, 17 de febrero de
Comenzado el
2021, 21:00
Estado Finalizado
miércoles, 17 de febrero de
Finalizado en
2021, 21:13
Tiempo empleado 13 minutos
Cali!cación Sin cali!car aún

Pregunta 1 Finalizado

Puntúa 0,00 sobre 1,00

Marcar pregunta

Con respecto a la irrigación del páncreas,


mencione de donde proviene la arteria
pancreatoduodenal inferior.

Seleccione una:

a. Directamente de la aorta abdominal

b. Arteria gastroduodenal

c. Ramas de la arteria suprarrenal superior


Ramas de la arteria suprarrenal superior

Ramas de la arteria suprarrenal superior

Ramas de la arteria suprarrenal superior

d. Arteria esplénica

e. Arteria mesentérica superior

La respuesta correcta es: Arteria mesentérica


superior

Pregunta 2 Finalizado

Puntúa como 1,00 Marcar pregunta

Una mujer de 25 años consultó a su médico


porque experimentaba episodios de cambio de
color acompañado de dolor en los dedos cuarto y
quinto de ambas manos. Al principio sus dedos se
volvían blancos por la exposición al frío y luego
adquirían un color azul oscuro. El cambio de color
se limitaba a la mitad distal de cada dedo y se
acompañaba de un dolor agudo. El único
tratamiento que le aliviaba el dolor consistía en
mantener las manos cerca de una estufa caliente
o entrar en una habitación cálida. La paciente le
comentó al médico que había notado que sus
dedos estaban húmedos de sudor durante
algunos de los episodios. Utilice sus
conocimientos de neuroanatomía y formule el
diagnóstico.

La paciente tienen una falla de pigmentación en como


respuesta sensorial por la temperatura.

Pregunta 3 Finalizado

Puntúa 0,00 sobre 1,00

Marcar pregunta

Las !bras poscomisurales de la estría terminal


terminan en?

Seleccione una:

a. Núcleo aumbens

b. GPi

c. Área preóptica medial

d. Área pretectal

e. Corteza frontal

La respuesta correcta es: Área preóptica medial

Pregunta 4 Finalizado

Puntúa 0,00 sobre 1,00

Marcar pregunta

Cual de las siguientes a!rmaciones es correcta:

Seleccione una:

a. La arteria cerebral media derecha va a irrigar


en la mayoría de personas , al área 22 , 44 y 45 de
brodman

b. La arteria radicular de adamkiewicz surge en


la mayoría de ocasiones en zonas lumbares altas
del lado derecho

c. La arteria radicular de adamkiewicz surge en


la mayoría de ocasiones en zonas lumbares altas
del lado izquierdo

d. La arteria de heubner es rama de la cerebral


media

e. La arteria cerebral posterior es la primera


rama de la arteria basilar

La respuesta correcta es: La arteria radicular de


adamkiewicz surge en la mayoría de ocasiones
en zonas lumbares altas del lado izquierdo

Pregunta 5 Finalizado

Puntúa 0,00 sobre 1,00

Marcar pregunta

Escoja la alternativa correcta

Seleccione una:

a. La arteria comunicante posterior es una rama


de la arteria cerebral media.

b. La arteria oftálmica es una rama de la arteria


cerebral media.

c. Las arterias pontinas son ramas de la arteria


carótida interna.

d. La arteria espinal posterior procede de la


arteria vertebral.

La respuesta correcta es: La arteria espinal


posterior procede de la arteria vertebral.

Pregunta 6 Finalizado

Puntúa 1,00 sobre 1,00

Marcar pregunta

Cual de las siguientes estructuras es reconocida


por su propiedad de presentar calci!caciones?

Seleccione una:

a. Adenohipó!sis

b. Glándula pineal

c. Neurohipó!sis

d. Bulbo olfatorio

e. Hipó!sis

La respuesta correcta es: Glándula pineal

Pregunta 7 Finalizado

Puntúa como 1,00 Marcar pregunta

Que arteria se localiza proximal o distal a la


arteria comunicante anterior, recorre un trayecto
de dirección caudal y lateral e ingresa al espacio
perforado anterior, este vaso irriga la parte
anteromedial de la cabeza del núcleo caudado, las
partes adyacentes de la cápsula interna y el
putamen, y partes de los núcleos septales.

DE ESCRIBIR MÁS DE UNA ESTRUCTURA


VASCULAR SE ANULARÁ LA RESPUESTA.

Arteria cerebral media

Pregunta 8 Finalizado

Puntúa 1,00 sobre 1,00

Marcar pregunta

Las siguientes estructuras forman parte del


sistema límbico, excepto:

Seleccione una:

a. Hipocampo

b. Área 32, 24 de Brodmann

c. Locus niger

d. Corteza del cíngulo

e. Núcleo accumbens

La respuesta correcta es: Locus niger

Pregunta 9 Finalizado

Puntúa 0,00 sobre 1,00

Marcar pregunta

En relación al Asa motora, seleccione cual de las


siguientes secuencias corresponde a la vía
directa.

Seleccione una:

a. Corteza cerebral....Putamen...........Núcleo
pedúnculopontino.....Tálamo....Corteza

b. Corteza cerebral....Putamen......GPi
MNc.........Núcleo
pedúnculotalámico.....Tálamo....Corteza

c. Corteza cerebral........GPi MNc.........Núcleo


pedúnculopontino.....Tálamo....Corteza

d. Corteza cerebral....Putamen......GPi
MNr.........Núcleo pedúnculopontino.........Corteza

e. Corteza cerebral....Putamen......GPi
MNr.........Núcleo
pedúnculopontino.....Tálamo....Corteza

La respuesta correcta es: Corteza


cerebral....Putamen......GPi MNr.........Núcleo
pedúnculopontino.....Tálamo....Corteza

Pregunta 10 Finalizado

Puntúa 0,00 sobre 1,00

Marcar pregunta

Cual de los siguientes núcleos talámicos cumple


una función descrita de la siguiente manera:
“Respuestas asociadas al comportamiento . que
implican decisiones basadas en la predicción y en
los incentivos”.

Seleccione una:

a. Núcleo reticular del tálamo

b. Núcleo mediodorsal del tálamo

c. Núcleos intralaminares del tálamo

d. Núcleo dorsomedial del tálamo

e. Pulvinar

La respuesta correcta es: Núcleo mediodorsal del


tálamo

Finalizar revisión

* Navegación por el cuestionario

1 2 3 4 5 6 7 8 9

10

Mostrar una página cada vez

Finalizar revisión
miércoles, 17 de febrero de
Comenzado el
2021, 21:04
Estado Finalizado
miércoles, 17 de febrero de
Finalizado en
2021, 21:14
Tiempo empleado 9 minutos 59 segundos
Cali!cación Sin cali!car aún

Pregunta 1 Finalizado

Puntúa como 1,00 Marcar pregunta

Una mujer de 25 años consultó a su médico


porque experimentaba episodios de cambio de
color acompañado de dolor en los dedos cuarto y
quinto de ambas manos. Al principio sus dedos se
volvían blancos por la exposición al frío y luego
adquirían un color azul oscuro. El cambio de color
se limitaba a la mitad distal de cada dedo y se
acompañaba de un dolor agudo. El único
tratamiento que le aliviaba el dolor consistía en
mantener las manos cerca de una estufa caliente
o entrar en una habitación cálida. La paciente le
comentó al médico que había notado que sus
dedos estaban húmedos de sudor durante
algunos de los episodios. Utilice sus
conocimientos de neuroanatomía y formule el
diagnóstico.

Daño en los núcleos posteriores lateral del tamalo

Pregunta 2 Finalizado

Puntúa 1,00 sobre 1,00

Marcar pregunta

Recibe sus principales aferencias de dos sitios: el


segmento palidal lateral y la corteza motora las
proyecciones corticales probablemente
representen colaterales de !bras corticófugas
destinadas a otras sustancias.

Seleccione una:

a. Núcleo subtalámico

b. Núcleo dorsal del rafe

c. Núcleo pedunculoprotuberancial

d. Cuerpo estriado

e. Locus niger

La respuesta correcta es: Núcleo subtalámico

Pregunta 3 Finalizado

Puntúa 0,00 sobre 1,00

Marcar pregunta

Las siguientes estructuras forman parte del


sistema límbico, excepto:

Seleccione una:

a. Hipocampo

b. Núcleo accumbens

c. Corteza del cíngulo

d. Área 32, 24 de Brodmann

e. Locus niger

La respuesta correcta es: Locus niger

Pregunta 4 Finalizado

Puntúa 1,00 sobre 1,00

Marcar pregunta

En relación al Asa Límbica la respuesta correcta


es:

Seleccione una:

a. Corteza cerebral.......Estriado ventral.....Pálido


Ventral.......MNr

b. Corteza cerebral.......Estriado ventral.....Pálido


Dorsal.......DMmc

c. Corteza cerebral.......Estriado Dorsal.....Pálido


Dorsal.......DMmc

d. Corteza cerebral.......Estriado ventral.....Pálido


Ventral.......DMmc

e. Corteza cerebral.......Estriado Dorsal.....Pálido


Ventral.......DMmc

La respuesta correcta es: Corteza


cerebral.......Estriado ventral.....Pálido
Ventral.......DMmc

Pregunta 5 Finalizado

Puntúa 1,00 sobre 1,00

Marcar pregunta

En relación al Asa cognoscitiva, seleccione cual de


las siguientes secuencias corresponde a la vía

Seleccione una:

a. Corteza cerebral.....Núcleo
caudado.......GPe.......Tálamo.....Corteza

b. Corteza cerebral.....Núcleo caudado.......GPi


MNr.......Tálamo.....Corteza

c. Corteza cerebral.....Núcleo caudado.......GPe


MNr.......Tálamo.....Corteza

d. Corteza cerebral.....Núcleo Putamen.......GPi


MNr.......Tálamo.....Corteza

e. Corteza cerebral.....Núcleo caudado.......GPe


MNc.......Tálamo.....Corteza

La respuesta correcta es: Corteza


cerebral.....Núcleo caudado.......GPi
MNr.......Tálamo.....Corteza

Pregunta 6 Finalizado

Puntúa 0,00 sobre 1,00

Marcar pregunta

Quien le da su inervación autónoma a la glándulas


sublinguales

Seleccione una:

a. VIII par , nervio cuerda del tímpano , !bras


parasimpáticas

b. VII par , nervio lingual , !bras parasimpáticas

c. V par , nervio mandibular , !bras simpáticas

d. IX par , Nervio cuerda del tímpano , !bras


parasimpáticas

e. V par , Nervio lingual , !bras simpáticas

La respuesta correcta es: VII par , nervio lingual ,


!bras parasimpáticas

Pregunta 7 Finalizado

Puntúa 0,00 sobre 1,00

Marcar pregunta

Que neurotransmisor utilizan los núcleos Ventral


lateral, medial y posterior del Tálamo para
conectarse con la porción sensoriomotora del
Neoestriado?

Seleccione una:

a. Glutamato

b. Dopamina

c. Sustancia P

d. Encefalina

e. Endor!na

La respuesta correcta es: Glutamato

Pregunta 8 Finalizado

Puntúa 0,00 sobre 1,00

Marcar pregunta

Escoja la alternativa correcta

Seleccione una:

a. La arteria oftálmica es una rama de la arteria


cerebral media.

b. La arteria espinal posterior procede de la


arteria vertebral.

c. Las arterias pontinas son ramas de la arteria


carótida interna.

d. La arteria comunicante posterior es una rama


de la arteria cerebral media.

La respuesta correcta es: La arteria espinal


posterior procede de la arteria vertebral.

Pregunta 9 Finalizado

Puntúa como 1,00 Marcar pregunta

Región que constituye la sustancia gris


periventricular de la parte más rostral del tercer
ventrículo, el núcleo correspondiente a esta
región circunda las paredes del tercer ventrículo.

DE ESCRIBIR MÁS DE UNA ESTRUCTURA


NUCLEAR, SE ANULARÁ LA RESPUESTA.

V núcleo

Pregunta 10 Sin contestar

Puntúa como 1,00 Marcar pregunta

Cual de los siguientes núcleos se encuentra en la


adherencia intertalámico?

Seleccione una:

a. Núcleo dorsofascicular

b. Núcleo Reuniens

c. Núcleo parafascicular

d. Núcleo anteroventral

e. Núcleo Reticular

La respuesta correcta es: Núcleo Reuniens

Finalizar revisión

* Navegación por el cuestionario

1 2 3 4 5 6 7 8 9

10

Mostrar una página cada vez

Finalizar revisión
:
PREGUNTAS EXAMEN NEUROANATOMIA PRACTICA

1. Paciente masculino de 40 años que ingreso a emergencia por accidente en moto, se encuentra
desorientado, con debilidad de brazo y pierna izquierda. Se realiza tomografía. ¿Que observa?

Hematoma epidural
2. En el siguiente grafico el numero 2 (el cuadro rojo inferior) corresponde a:

NUCLEO FASTIGIO
3. Paciente que presenta diplopía y estrabismo convergente el par craneal más probablemente
afectado sería:

C
4. Mujer de 34 años se presenta con la anormalidad mostrada en la foto. En la imagen derecha:
Cual de las siguientes estructuras señaladas con literales es la más probablemente lesionada?

C
5. Señale las estructuras que estarían con mayor probabilidad incluidas en el área resaltada:

Núcleo del III par craneal, lemnisco medial


6. Indique la estructura señalada con el punto rojo

Coliculo inferior
7. EL MUSCULO OBLICUO INFERIOR ES INERVADO POR:
NERVIO MOTOR OCULAR COMUN
8. Un paciente presenta ataxia, pérdida bilateral de las sensaciones de tacto fino y presión, ¿a
cuál imagen corresponde la lesión sombreada con negro?

C
9. Hombre 24 años muere posterior accidente automovilistico. La autopsia se muestra del lado
derecho y en el lado izquierdo se muestra una pieza anatómica normal. Teniendo en cuenta la
lesión mostrada en el lado derecho, lo más probable es que es que esta lesión causara:

Anisocoria por midrisis arreactiva


10. Paciente posterior a accidente automovilístico se evidencia en RM se evidencia fractura
vertebral. la sintomatología del paciente estaría predominantemente caracterizada por:

Paraparesia, anestesia en silla de montar e incontinencia urinaria


11. Hombre de 27 años, es traido al departamento de emergencia, posterior a un accidente en
motocicleta. Tiene una herida maxillofacial. Su condicion es estabilizada, y reparacion
quirurgica es realizada. Mientras que se recupera, el paciente desarrolla dificultad para
masticar su comida. En el examen fisico, se le pide que abra su boca, y sucede lo que ese
muestra en la foto inferior. El nervio mas probablemente lesionado, emerge a través de cual
de los siguientes foramenes mostrados en la tomografia?

A
12. Niño de 4 años es traído al consultorio por su madre. El examen físico revela la anormalidad
mostrada en la foto. Cual de las siguientes localizaciones es la más probablemente afectada?

C
13. Hombre de 35 años acude a la oficina debido a dolor en la lengua. Por 2 semanas ha tenido
fiebre, experimentado mialgias y artralgias. No tiene problemas medicos. Trabaja como un
conductor de uber. Tuvo sexo con un extraño hace aproximadamente un mes atras. El examen
fisico revela un exantema en todo su torax y linfoadenopatia cervical. Una ulcera es
visualizada en el surco medio de la lengua y esta 2 centrimetros anterior al foramen cecum. Se
obtiene una prueba para HIV. La sensacion dolorosa causada por la ulcera es mas
probablemente percibida por cual de los siguientes nervios?

D
14. Mujer de 27 años acude a la oficina a causa de varias semanas de dolor y sensacion de
“abombamiento” en el oido izquierdo. Tambien describe la sensacion de craquido mandibular
mientras que come. La paciente tiene un historial de dolores de cabeza episodicos y dolor
facial izquierdo por varios años que son peores cuando mueve la mandibula. Ademas su novia
le ha dicho que rechina sus dientes cuando duerme. Durante el examen fisico es incapaz de
abrir su boca totalmente y desarrolla dolor con movimientos pasivos de la mandibula. El canal
auditivo externo y membrana timpanica son normales. Pruebas de Weber y Rinne son
normales. Se sospecha una disfuncion del Musculo pterigoideo. Cual de las siguientes
opciones es la mas probable de estar lesionada y causar estas manifestaciones?

A
15. Hombre de 24 años sufre una lesion neurologica. Tiene un historial de Esclerosis Multiple. La
resonancia magnetica revela un area de mielinizacion que lesiona un nervio cranial. El examen
fisico muestra los hallazgos en el video. Cual de los siguientes es el nervio cranial mas
probablemente lesionado?

Óptico derecho
16. Niña de 4 años se presenta a su consulta con la examinacion mostrada en el video. Cual de los
siguientes nervios esta lesionado?

Troclear izquierdo
17. Hombre de 23 años con historial de paralisis que se resuelven espontaneamente acude con los
hallazgos mostrados en la foto inferior. Lo más probable es que la localizacion de la lesion se
encuentre en cual de las siguientes opciones?

Tegmentum bilateralmente
18. Hombre de 55 años acude al departamenteo de emergencia a causa de cefaela y dificultad
para a vision de inicio subito. Rapidamente queda inconciente camino al hospital. Tiene un
historial de hipertesion arterial. Sus medicamentos incluyen aspirina y lisonopril. Su tension
arterial es 150/90 mm Hg y su pulso es 90 latidos por minute. Una tomografia de cerebro
revela una hemorragia aguda en el lobulo temporal mas compresion anterior del lobulo
temporal anterior medial en contra del margen libre del tentorio del cerebelo. Cual de los
siguientes nervios craneales es el mas probable de estar comprometido en este paciente?

B
19. Hombre de 70 años con historial de cancer de pulmon acude a la oficina debido a disfonia y
disfagia. No tiene dificultad para la vision ni para la audicion. En el examen fisico, se revela lo
mostrado en el video. Su hombro izqueirdo carece de fuerza. La imagen de torax revela una
lesion con apariencia tumoral en el pulmon y varias lesiones osteoliticas en las costillas de
apariencia metastasica. Una lesion comprometiendo cual de las siguientes estructuras
anatomicas es la mas probable de causar los signos neurologicos del paciente?

Foramen yugular
20. Hombre de 34 años, inicia tratamiento con quimioterapia para linfoma Hodgkin. Luego de eso,
experiementa vomitos severos y require fluidos intravenosos. Cual de las siguientes
localizaciones es la responsable de estos sintomas?

D
21. Mujer de 38 años con diagnostico de cancer de mama fue sometida mastectomia y a
linfadectomia axilar izquierda. Posteriormente durante el examen fisico se observa la foto
mostrada en la parte inferior izquierda. Este paciente más probablemente se lesiono cual de
los siguientes nervios?

V
22. Mujer de 22 años acude a la emergencia luego de lesionarse su hombro derecho durante un
juego de baloncesto. Estaba tratanto de bloquear un lanzamiento cuando abduccio y roto su
brazo externamente de forma forzada contra un jugador del equipo contrario. El paciente
cayo inmediatamente con dolor severo sosteniendo su brazo. En el examen fisico, hay una
obvia asimetria en su hombro derecho en comparacion con el izquierdo, y ella esta
sosteniendo su brazo derecho con leve abduccion y rotacion externa. El examen fisico revela
debilidad de abduccion del hombro también perdida de la sensibilidad sobre un parche en el
hombro lateralmente, una radiografia muestra una dislocacion anterior de la cabeza del
humero. Este paciente más probablemente se lesiono cual de los siguientes nervios?

M
23. Hombre de 23 años es traido a la emergencia luego de sufrir una puñalada en una pelea
callejera. El paciente se siente mareado. Su tension arterial es 100/60mmHg y su pulso es
115/min. El examen fisico revela una hemorragia profusa debido a heridas penetrantes en el
cuello y el area del hombro izquierdo. Fluidos intravenosos, y transfusion sanguinea es llevada
a cabo. La exploracion quirurgica urgente es realizada, y el vaso sanguineo lesionado es
reparado. Adicional exploracion muestra la completa seccion del nervio indicado con una
flecha roja en la imagen inferior. Cual de las siguientes acciones es la mas probable de estar
debilitada como resultado de la lesion de este paciente?

Extensión de la muñeca
24. Hombre de 24 años sufre una fractura en su tibia derecha en un accidente automovilistico. Su
pierna derecha es inmovilizada con un yeso y requiere muletas para deambular. Luego de 2
semanas, acude a la clinica con debilidad y adormecimiento de su extremidad superior
derecha. No tiene dolor de cuello o dolor en el brazo. En el examen neurologico, no tiene
fuerza en la extension de la muñeca con ausencia del reflejo tricipital. Lesión a cual de los
siguientes nervios es el mas probablemente responsible de los sintomas de este paciente?

N
25. Mujer de 45 años es evaluada por adormecimiento y hormigueo de su mano derecha que
empezo 2 meses atrás y ocasionalmente la despierta durante la noche. Los sintomas del
paciente alivian intermitentemente al sacudir su mano. Al examen fisico, hay dismunicion de
la sensacion de los tres primeros dedos de su mano derecho con preservacion de la sensacion
a lo largo de la eminencia tenar. La fuerza muscular esta intacta. Flexion de la palma a nivel de
la muñeca reproduce los sintomas en su mano derecha. El paciente mas probablemente sufrio
una lesion en cual de los siguienes nervios?

O
26. Hombre de 12 años es traído a la emergencia por su madre debido a torpeza en su mano
derecha. Hace varias horas trato de subir un arbol pero perdio su balance y cayo desde el
arbol. El trato de parar la caida tratando de sostenerse de una rama del arbol con su brazo
derecho. Cayo seguramente en sus pies sin sufrir trauma o perdida de la conciencia. En el
examen fisico, tiene dificultad para mover los dedos de su mano derecha. Cual de las
siguientes estructuras es la más probablemente lesionada en este paciente?

E
27. Hombre 65 años acude al medico por debilidad y adormecimiento de ambas piernas.
Recientemente fue dado de alta del hospital luego de pasar varias semanas en la unidad de
cuidados intensivos bajo sedación. En el examen fisico es incapaz de evertir los pies. Ademas
tiene analgesia en el dorso de ambos pies y en la parte lateral de los tobillos. Cuando se le
pide que camine se observa lo visto en el video. Cual de los siguientes es el mas
probablemente lesionado?

Peroneo común
28. Hombre de 64 años acude al departamento de emergencia con severo, lacinante, dolor de
espalda que se irradia hasta las piernas. Se ha caído frecuentemente durante estas ultimas 2
semanas. Durante la ultima semana, tiene dolor y dificultad para miccionar que ha ido
aumentando ademas de una analgesia del area mostrada en la imagen inferior. Fue
diagnosticado con cancer de prostata hace un año y tratado con radioterapia. El examen fisico
revela una severa debilidad de extremidades inferiores que es mas pronunciada del lado
derecho. Los reflejos rotulianos y aquileanos estan ausentes en las extremidades. Cual es la
localizacion más probablemente responsable de la lesion?

Raices espinales nerviosas


29. Hombre de 53 años es evaluado por molestias en su mano derecha. Hace un mes empezo a
desarrollar la sensacion de “punzadas de agujas” en su mano derecho que eran peores en la
noche. No tiene edema, ni rubicundez, o aumento de la temperatura localmente en la mano.
No tiene historial de trauma en cuello en la extremidad superior derecha. El paciente ha
fumado un paquete de cigarrillos al dia por 20 años. Ha trabajado como contador durante 30
años. En el examen fisico, hay una perdida de la sensibilidad en el 5 to dedo ademas de
debilidad para la aduccion de la muñeca. Este paciente más probablemente se lesiono cual de
los siguientes nervios?
P
30. Hombre de 25 años acude a la oficina con debilidad de su brazo derecho. El paciente se
lesiono su hombro derecho hace varios meses luego de un accidente en motocicleta y ha
tenido dificultades elevando su brazo desde ahí. En el examen fisico, tiene analgesia en una
pequeña area en la parte lateral superior derecha de su brazo y atrofia prominente del
deltoides con una fuerza 3/5 en la abducción del hombro. Los reflejos y empuñadura derechas
son normales bilateralmente. Este paciente más probablemente se lesiono cual de los
siguientes nervios?

M
31. Mujer de 90 años se presenta con una paralisis de hemicuerpo izquierdo de predominio distal
con mayor severidad en la mano, con signo de Hoffman a la exploración física. Cual es el sitio
mas probable de la lesión?

A
32. Mujer de 69 años se presenta con la anormalidad presentada en el vídeo. Cual es la otra
caracteristica acompaña más probablemente a este evento?
Debilidad
33. Hombre de 31 años sufre un trauma craneo encefalico. Se levanta rapidamente, y luego de 3
horas cae inconciente. Muere 2 horas después, la imagen obtenida al ingreso al hospital se
encuentra del lado derecho. Según la imagen en el lado izquierdo, Cual es el sitio más
probable de lesión?

B
34. Mujer de 20 años, se presenta con incapacidad para la marcha correcta. Cuando cierra sus ojos
no puede percibir cuando tocan su pierna derecha. Según la imagen inferior. Cual es la
localización más probable de su lesión?

1
35. Mujer de 99 años es admitida debido a inabilidad para la marcha, incontinencia y compromiso
cognitivo. La imagen obtenida al ingreso al hospital se encuentra del lado derecho. Según la
imagen en el lado izquierdo, Cual es el sitio más probable de lesión?

P
36. Hombre 52 años, falleció a causa de un tumor maligno en encefalo. Este tumor causo aumento
de presión endocraneana. La autopsia revelo una lesión en el mesencefalo que se muestra en
la pieza anatómica en la imagen inferior. Cual es la causa mas probable de estas lesiones
encerradas en la línea roja?
Hernia uncal
37. Hombre de 65 años con hipertensión arterial y consumo de 3 cajetillas de tabaco al día.
Desarrollo subitamente dolor de cabeza. Cae inconciente. Muere 9 horas después, la imagen
obtenida al ingreso al hospital se encuentra del lado derecho. Según la imagen en el lado
izquierdo, Cual es el sitio más probable de lesión?

Q
38. Un hombre se presenta a su emergencia luego de un trauma craneo encefálico con la
manifestación mostrada en la foto, en la imagen derecha. Cual de las siguientes estructuras
señaladas con literales es la más probablemente lesionada?

E
39. Mujer de 69 años se presenta con la anormalidad mostrada en la imagen. Cual es el sitio más
probable de la lesión?

Cordón lateral izquierdo


40. Hombre de 29 años sufre una lesión en su médula espinal luego de una caída desde un
segundo piso. Realiza rehabilitación pero aún no puede caminar normalmente. Durante el
examen físico realiza extensión del primer dedo del pie ante el estímulo plantar. Cual es la
localización más probable de su lesión?

2
41. Hombre 85 años acude por paralisis del hemicuerpo izquierdo. Cual es la localización más
probable de la lesión causante de este evento?

D
42. Un hombre de 63 años acude con paralisis del lado derecho del cuerpo. En la ilustración
anatómica mostrada en la imagen inferior. Cual de los literales señalados sería la localización
más probable de la lesión?

A
43. Un hombre se presenta con la paralisis mostrada en la foto. En la imagen derecha: Cual de las
siguientes estructuras señaladas con literales es la más probablemente lesionada?
C
44. Mujer de 75 años se presenta con inicio subito de dificultad para realizar calculos, confusion
de derecha con izquierda, dificultad para escribir e incapacidad para reconocer los nombres de
los dedos. Segun la siguiente imagen cual es la localizacion mas probable de su lesion?

J
45. Mujer de 65 años se presente con inicio subito para la compresion del lenguaje. Cual de las
siguientes localizaciones es la mas probable de ser lesionada?

G
46. Hombre de 80 años se presente con inicio subito para el habla en la produccion del lenguaje
pero mantiene la compresion del lenguaje. Cual de las siguientes localizaciones es la mas
probable de ser lesionada?

A
47. Un hombre de 78 años acude con una lesión en la vía mostrada en la imagen inferior. Cual de
las siguientes funciones es la mas probablemente afectada?

Motora voluntaria
48. Una mujer de 65 años acude con una lesión en la vía de color VERDE, mostrada en la imagen
inferior. Cual de las siguientes funciones es la mas probablemente afectada?

Propioceptiva
49. Una mujer de 73 años acude con una lesión en la vía de color VERDE, mostrada en la imagen
inferior. Cual de las siguientes funciones es la mas probablemente afectada?

Termoalgesia
50. En la siguiente imagen señale la estructura señalada con un punto rojo.

RE:pilar anterior del fórnix


1. Hombre de 34 años se reúne con su médico después de experimentar extraños movimientos
espasmódicos en las extremidades y la cara, incontrolables. Además de cambios de
comportamiento. Tiene cambios de humor. Dice que su padre mostró síntomas similares
cuando tenía 40 años y su abuelo 49 años. En el examen se observa un movimiento irregular,
breve y sin propósito de las manos. Es diagnosticado con enfermedad de Huntington. ¿Cuál de
los siguientes localizaciones se observaría cambios lesionales con mayor probabilidad en este
paciente?

RE: B
2. Lesión a nivel de la corteza marcada de rojo, el paciente presentaría:

RE:Ceguera cortical
3. Hombre de 89 años es llevado al médico por sus dos hijos que están preocupados por los
cambios en su comportamiento durante el año pasado. Hace comentarios sexualmente
sugerentes a mujeres y se masturba en público. También se le vio recientemente tratando de
meterse los bloques de juguete de sus nietos en la boca. La memoria está intacta y el examen
neurológico es normal. ¿Cuál de las siguientes localizaciones es con mayor probabilidad
lesionada?

E
4. Mujer de 34 años presenta un trastorno caracterizado por conductas exploratorias orales o
táctiles (tocamientos o succiones socialmente inapropiadas); hipersexualidad, bulimia,
trastornos de la memoria, emociones planas, prosopagnosia. Seleccione la letra que
corresponde a la estructura más probablemente lesionada en este caso.

D
5. La desviación de la mirada conjugada en dirección de la lesión se produce por daño a nivel de
la corteza oculomotora, representada en la siguiente área, (seleccione el literal correcto):

A
6. Un hombre de 48 años con hipertensión grave no controlada desarrolla cefalea y vómitos. Dos
horas más tarde, tiene debilidad en el lado izquierdo seguida rápidamente de pérdida de la
consciencia. Muere poco después de su ingreso al servicio de urgencias. Se muestra una
sección de su cerebro como se ve en la autopsia. ¿El mecanismo más probable de la lesión que
se muestra es la hemorragia de cuál de los siguientes?
Re: arterias lenticulo estriadas
7. En la siguiente tomografía simple cerebral se evidencia área de isquemia en parénquima
cerebral, correspondiente a la irrigación de la siguiente arteria:

RE: arteria recurrente de Heubner


8. Un hombre de 80 años tiene una marcha inestable, dificultad para sonreír y un temblor rítmico
de su mano derecha en reposo. Su trastorno se caracteriza por una marcada disminución de la
disponibilidad de un determinado neurotransmisor. ¿Los receptores post-sinápticos para el
neurotransmisor deficiente se encuentran en qué estructura de los núcleos basales?

A
9. LA ESTRUCTURA SEÑALADA CORRESPONDE A:
Re: núcleo subtalámico
10. SEÑALE EL ENUNCIADO INCORRECTO CON RESPECTO A LA ESTRUCTURA SEÑALADA:

a. CORRESPONDE AL TRÍGONO HABENULAR


b. SU LÍMITE INTERNO ES LA PARTE POSTERIOR E INTERNA DEL TÁLAMO Correcta
c. EN ESTÁ ÁREA SE ENCUENTRAN LOS NÚCLEOS HABENULARES
d. POR FUERA Y DETRÁS ESTÁ LA GLÁNDULA PINEAL

12. RE: epífisis


13. DE LA ESTRUCTURA RESALTADA EN ROJO CUÁL DE LAS SIGUIENTES ESTRUCTURAS
CONSTITUYE EL LÍMITE INFERIOR EN SU PARTE POSTERIOR:

RE: espacio interpeduncular

14. RE: su limite externo es la capsula interna.


15. SEÑALE EL ENUNCIADO CORRECTO CON RESPECTO AL NÚCLEO RESALTADO EN COLOR
VERDE:

RE: núcleo preóptico


16. EL NÚCLEO SEÑALADA CON LA FLECHA ROJO, SU FUNCIÓN ES:
RE:tono emocional
17.
Re: TALAMO
18. A LÁMINA TERMINAL CORRESPONDE AL NÚMERO:

RE:2
19. SEÑALE LO CORRECTO CON RESPECTO AL ÁREA DENTRO DEL CÍRCULO AZUL:

RE:CENTRO DE CONTROL AUTONOMO


El elemento señalado con asterisco corresponde a:
a- Ninguna de las anteriores

b- Seno recto
c- Anastomótica mayor
d- Vena de labbe
e- Vena de galeno
La arteria recurrencial de heubner es rama de:

a. 3
b. 6
c. 1
d. 5
e. 4
f. 2
Una sola respuesta. Seleccione la estructura indicada por la flecha negra.

a. Colículo facial
b. Área vestibular
c. Plexo coroideo
d. Núcleo endorestiforme
e. Cuerpo yuxtarestiforme

¿El signo de Babinski forma parte del?


a. Síndrome de motoneurona inferior
b. Síndrome de motoneurona superior
c. Síndrome de Parinaud
d. Ninguna de las anteriores
e. Síndrome de Wallenberg
Luego de un altercado domestico un hombre de 53 años sufre un golpe en su cabeza. Cuando
es encontrado, se encuentra oliendo a alcohol, sin poder mover el ojo izquierdo hacia la parte
lateral. ¿Cuál de las siguientes estructuras, al lesionarse explicaría los hallazgos?

a. B
b. A
c. C
d. F
e. E
En la tumoración señalada en la imagen, por su ubicación anatómica qué estructura puede
estar afectada:

a. Seno sagital inferior


b. Nervio neumogástrico
c. XII par
d. Área motora primaria
e. Quiasma óptico
Señale el enunciado correcto con respecto al área resaltada en rojo

a. Ninguna opción es correcta


b. Es la fosa interpeduncular y emerge el nervio troclear
c. Corresponde a pedúnculos cerebelosos superiores y emerge el nervio oculomotor
d. Es la fosa interpeduncular y emerge el motor ocular común
e. Corresponde a los pedúnculos cerebrales y emerge el nervio troclear.
En la siguiente imagen de ventrículos cerebrales, lo señalado corresponde a:

a. Agujero interventricular
b. Receso supraóptico
c. Receso pineal
d. Acueducto de Silvio
e. Receso infundibular
A nivel del surco resaltado en rojo emerge

a. Nervio trigémino
b. No emerge ningún nervio
c. Nervio facial
d. Nervio neumogástrico
e. Nervio glosofaríngeo
INDIQUE QUE ARTERIA PUEDE ESTAR OCLUIDA SI EL PACIENTE PRESENTA: HEMIPARESIA,
HIPOESTESIA CONTRALATERAL A PREDOMINIO SUPERIOR, HEMIANOPSIA HOMÓNIMA
CONTRALATERAL, ANOSOGNOSIA

Seleccione una:
a. 3
b. 2
c. 4
d. 1 Correcta
PACIENTE CON MOVIMIENTOS BRUSCOS INCOORDINADOS DE BRAZO DERECHO, LA
EXTREMIDAD SUPERIOR DERECHA SE PROYECTA BRUSCAMENTE DE FORMA VIGOROSA. CUÁL
ESTRUCTURA ESTÁ AFECTADA?
Seleccione una:
a. NÚCLEO DEL TÁLAMO IPSILATERAL
b. NÚCLEO SUBTALÁMICO DERECHO
c. NÚCLEO SUBTALÁMICO IZQUIERDO Correcta
d. CIRCUNVOLUCIÓN PREFRONTAL ASCENDENTE
SEÑALE EL ENUNCIADO CORRECTO CON RESPECTO A LA ARTERIA SEÑALADA

Seleccione una:
a. IRRIGA EL TÁLAMO Correcta
b. SU OBSTRUCCIÓN SE RELACIONA CON APATÍA Y CAMBIOS DE PERSONALIDAD
c. IRRIGA EL ÁREA MOTORA PRINCIPAL CORRESPONDIENTE A LAS EXTREMIDADES INFERIORES
d. IRRIGA LA CORTEZA CORRESPONDIENTE AL ÁREA AUDITIVA
EN LA ENFERMEDAD DE PARKINSON CÚAL DE LAS ESTRUCTURAS SEÑALADAS ESTÁ AFECTADA

Seleccione una:
a. TODAS
b. A
c. C
d. B
SEÑALE EL ENUNCIADO CORRECTO CON RESPECTO A LA ESTRUCTURA SEÑALADA:
Seleccione una:
a. FORMA PARTE DEL NÚCLEO LENTICULAR Correcta
DEL NÚCLEO BASAL SEÑALADO INDIQUE LO CORRECTO EN EN LO REFERENTE A SUS
CONEXIONES

Seleccione una:
a. RECIBE FIBRAS DEL ESTRIADO Correcta

RE: rama de la arteria basilar

Flecha naranja: seno longitudinal inferior


SEÑALE LA LESIÓN SI EXISTE UN PROCESO OBSTRUCTIVO DE LA ARTERIA SEÑALADA

Seleccione una:
a. AFASIA DE EXPRESIÓN
b. ESTEROTAXIA CONTRALATERAL
c. DESVIACIÓN DE LA MIRADA HACIA EL MISMO LADO DE LA LESIÓN
d. TODAS LAS ANTERIORES

RE: seno petroso superior


Hombre de 39 anos acude con dolor de cabeza de 8 meses de evolución. Durante la
exploración neurológica revela los hallazgos mostrado en las fotos. El compromiso estructural
de cual de las siguientes estructuras es la mas probable de causar estos hallazgos?

RE: H
Mujer de 36 anos acude al departamento de emergencia por dolor en su ojo izquierdo y visión
doble intermitente. Hace aproximadamente 3 anos comenzó a experimentar cefalea frontal y
dolor retroocular izquierdo. Un ano y medio ante de la presentación actual empezó a
desarrollar caída del parpado y midriasis del lado izquierdo. Ademas noto que ocasionalmente
su ojo izquierdo se desviaba hacia la izquierda, causando visión doble (la cual era peor cuando
miraba hacia la derecha). El examen físico se revela en la foto inferior izquierda. Según la
localización mas probable de la lesión. Cual de las siguientes estructuras es la mas
probablemente afectada?

RE:G
Hombre de 71 anos acude a la consulta por perdida del gusto. Esta preocupado por la
posibilidad de haber contraído COVID-19 posterior a una reunión navideña hace una semana.
Actualmente presenta dolor cervical derecho, vértigo, ataxia, adormecimiento facial derecho y
corporal izquierdo además de ronquera. El paciente sufre hipertensión arterial, fuma 4
cajetillas de tabaco por dia y acude regularmente a consulta con su quiropráctico. El examen
físico se muestra en la foto. Ante la localización mas probable de la lesión. Cual de las
siguientes estructuras es la mas probablemente afectada?

RE: E
Mujer de 27 años acude a la oficina a causa de varias semanas de dolor y sensacion de
“abombamiento” en el oido izquierdo. Tambien describe la sensacion de craquido mandibular
mientras que come. La paciente tiene un historial de dolores de cabeza episodicos y dolor
facial izquierdo por varios años que son peores cuando mueve la mandibula. Ademas su novia
le ha dicho que rechina sus dientes cuando duerme. Durante el examen fisico es incapaz de
abrir su boca totalmente y desarrolla dolor con movimientos pasivos de la mandibula. El canal
auditivo externo y membrana timpanica son normales. Pruebas de Weber y Rinne son
normales. Se sospecha una disfuncion del Musculo pterigoideo. Cual de las siguientes
opciones es la mas probable de estar lesionada y causar estas manifestaciones?

RE:A
Hombre de 70 años con hipertensión arterial acude con cuadro clínico de 3 meses de
evolución caracterizado por amnesia, destellos luminosos, y diplopía. Lo mas característico de
estos eventos fueron la recurrencia e intermitencia. El examen físico revelo reducción leve de
la atención, anisocoria por midriasis derecha, limitación de la mirada hacia arriba, reducción
de la convergencia ocular derecha acompañado de blefaroptosis derecha. Debido a estos
hallazgos fue redireccionada a la emergencia, pero al llegar su examen físico era normal. Fue
admitida para estudio, y 24 horas después el examen físico revelo incapacidad para responder
a comandos verbales, cuadriplejia e intactos movimientos oculares verticales. Ante la
localización mas probable de la lesión. Cual de las siguientes estructuras es la mas probable de
estar afectada?

RE:B
Mujer de 67 años se presenta con inicio subito que afecto la parte derecha de su cuerpo. Los
resultados del examen fisico sugieren oclusion de una arteria. La resonancia mostrada en la
parte inferior confirman la localizacion de la lesion. Basado en la localizacion, cual de los
siguientes nucleos del talamo es el mas probable ser el mayormente afectado?

RE: Ventral posterolateral


Una resonancia de cerebro de una mujer de 69 años revela una lesion en la region del talamo
indicado por la zona de color rojo. Oclusion de cual de las siguientes vasos es la mas probable
de causa esta lesion?

Re: arteria posterior medial


Un hombre de 56 años acude a su médico por dificultad para iniciar el movimiento que ha ido
empeorando en los últimos meses. Durante un examen neurológico, su marcha consiste en
pasos cortos y rápidos durante los cuales sus pies no abandonan el suelo. Y tiene temblor en
reposo ¿Cuál de las siguientes ubicaciones en el cerebro del paciente es más probable que
este lesionada?

RE:B
Hombre de 85 años acude al departamento de emergencia a causa de una perdida subita de
propiocepcion y analgesia en el hemicuerpo derecho. La tomografia revela una lesion en el
talamo muy circumscrita. Cual de las areas indicadas en la imagen inferior es la localizacion
mas probable de la lesion?
RE: C
Una mujer de 68 años sufre la aparición repentina de trastornos leves del movimiento. La
resonancia magnética a su regreso reveló una lesión hemorrágica en el área irrigada por las
arterias estriadas laterales. ¿Cuál de las áreas indicadas en la imagen siguiente representa la
ubicación más probable de esta lesión?

RE:B

1. El área resaltada en rojo forma parte funcional de:

El sistema límbico
2. El elemento señalado con asterisco corresponde a:

Vena de Labbé
3. Cual de las siguientes estructuras esta identificada entre las dos flechas en la imagen
inferior?

Formación hipocampal
4. Un paciente de 65 años de edad, con antecedes de hipertensión y diabetes en llevado a la
emergencia. El paciente no puede expresar lo que le sucede, además de que cursa con
paralisis del brazo y la pierna derecha. Según la imagen tomografica cuál es la arteria
afectada?

Arteria cerebral media izquierda


5. El área resaltada en rojo está relacionada principalmente con:

Recepción de estímulos internos y externos


6. EN LA TUMORACIÓN SEÑALADA EN LA IMAGEN, POR SU UBICACIÓN ANATÓMICA QUÉ
ESTRUCTURA PUEDE ESTAR AFECTADA:

Quiasma óptico
7. Una resonancia magnética de cerebro, de un hombre de 53 años revela un infarto lacunar
en el area del tálamo que selectivamente se proyecta a la región cortical senaladas por las
flechas blandas en la imagen inferior. Cual de los siguientes núcleos talámicos representa
la localización más probable de esta lesión?

Anterior
8. Hombre de 23 años es traído al departamento de emergencia luego de un accidente de
motocicleta. La tomografía de cerebro revela la fractura de multiples huesos y hemorragia
bilateral en las areas mostradas en la parte inferior con rojo. Según la lesión en esta
persona. Cuál de las siguientes opciones es la más probable de experimentar como
manifestación?

Apatía, hiperoralidad e hipersexualidad


9. En el polígono arterial de Willis la ARTERIA marcada con el número 5 es rama de:

ARTERIA CAROTIDA INTERNA


10. Una lesión en el área resaltada en rojo dará como resultado:
Dificultad para hablar con fluidez
11. Hombre 86 años, es incapaz de mover sus 4 extremidades, esta despierto y es capaz de
responder preguntas a través de un pestañeo que significaba “sí” y dos pestañeos que
significaba “no”. Hay movimientos verticales de sus ojos, pero no horizontales. No hay
perdida ni propioceptiva ni dolorosa. ¿Cuál es la localización más probable de la lesión
causante de este cuadro?

A
12. Mujer 40 años acude por percibir aromas desagradables e indescriptibles, nausea, y una
sensación ominosa de miedo estos cuadros son de origen súbito. Luego de lo cual se siente
cansada y queda dormida. ¿Cuál es la localización más probable de la lesión causante de
este cuadro?

D
13. En la imagen de cerebelo, el número 2 representa al núcleo:

Del Techo
14. Indique cual es la estructura señalada con el punto verde:
Cuerpos mamilares
15. Señale la respuesta correcta:

Hemorragia subaracnoidea
16. En el siguiente corte anatómico de disección se ha removido el neopallium para aislar qué
circuito funcional?

Circuito de Papez

17.

18.
19.

20.

21.
22. IDENTIFIQUE EL AREA RESALTADA EN ROJO:

Piramides
23. La linea amarilla corresponde a:
SURCO PROTUBERANCIAL SUPERIOR
24. El área resaltada en rojo corresponde a:

NINGUNA DE LAS OPCIONES


25. El área resalta con rojo corresponde a:

CARA ANTEROLATERAL DEL PUENTE


26. IDENTIFIQUE LA ESTRUCTURA SEÑALADA:

PISO IV VENTRICULO
27. ESTÁ IMAGEN CORRESPODE A:

Mesencéfalo
28. EL NÚCLEO REPRESENTADO EN ESTA IMAGEN CORRESPONDE A LOS NÚCLEOS:

IX,X, XI
29. PERTENECE A LA ESTRUCTURA EN VERDE:

NÚCLEO SENSITIVO PRINCIPAL DEL TRIGÉMINO


30. LA LESIÓN DEL NÚCLEO DENTRO DEL CIRCULO PRODUCE:

MIDRIASIS
31. LA LESIÓN DEL DEL NÚCLEO DENTRO DEL CÍRCULO OCASIONA
DISGEUSIA
32. A NIVEL DEL SURCO PREOLIVAR EMERGE:

GLOSOFARINGEO
33. DEL ÁREA RESALTADA EN VERDE PODEMOS ENCONTRAR:

NÚCLEOS DEL FACIAL, SENSITIVO DEL TRIGÉMINO VESTIBULAR


34. De la siguiente imagen seleccione:Elemento característico del nivel anatómico:

Fibras arqueadas internas en formación


35. Mujer de 21 años presenta fasciculaciones en el párpado derecho que han progresado en
un año afectando a la mejilla y el labio ipsilaterales más pérdida auditiva ipsilateral desde
la infancia. Acompañado de analgesia de hemicara derecha. El paciente fallece a causa de
una enfermedad en su tallo encefálico, y se realiza la autopsia de su tallo encefálico ¿Cuál
de las siguientes estructuras, al lesionarse explicaría los hallazgos en este paciente?

A
36. La estructura señalada en rojo corresponde a fibras:

EFERENTES SOMATICOS GENERALES


37. Si ocurre una lesión a nivel del área resaltada en rojo se lesionaría:

LA VIA CORTICOESPINAL
38. Señale el enunciado correcto con respecto al tracto señalado:

Corresponde al fascículo cuneiforme y lleva información de tacto discriminativo, vibración y


posición de los miembros superiores.
39. Hombre de 73 años acude por inicio súbito de debilidad de hemicuerpo izquierdo y visión
doble. El examen neurológico revela hemiplejía izquierda más caída palpebral derecha con
desviación del globo ocular hacia el lado externo. Según los hallazgos encontrados en el
examen físico. ¿Cuál es el sitio más probable de lesión entre las estructuras demarcadas
dentro de los círculos en el corte de anatomía macroscópica?

D
40. EL ÁREA RESALTADA EN ROJO CORRESPONDE A:
FIBRA COMISURAL
41. EL ÁREA RESALTADA EN ROJO CORRESPONDE A:

COLUMNA ANTERIOR DEL FÓRNIX


42. EL ÁREA RESALTADA EN ROJO CORRESPONDE A:

CÁPSULA EXTERNA
43. SEÑALE EL ENUNCIADO CORRECTO CON RESPECTO AL ÁREA DENTRO DEL CÍRCULO AZUL:

SU LÍMITE EXTERNO ES LA CÁPSULA INTERNA


44. NDIQUE QUE ARTERIA PUEDE ESTAR OCLUIDA SI EL PACIENTE PRESENTA: HEMIPARESIA,
HIPOESTESIA CONTRALATERAL A PREDOMINIO SUPERIOR, HEMIANOPSIA HOMÓNIMA
CONTRALATERAL, ANOSOGNOSIA:

1
45. SEÑALE LA LESIÓN SI EXISTE UN PROCESO OBSTRUCTIVO DE LA ARTERIA SEÑALADA:

AFASIA DE EXPRESIÓN
46. En la siguiente tomografia simple cerebral se evidencia área de isquemia en parénquima
cerebral, correspondiente a la irrigación de la siguiente arteria:

Arteria recurrente de Heubner


47. Un hombre de 48 años con hipertensión grave no controlada desarrolla cefalea y vómitos.
Dos horas más tarde, tiene debilidad en el lado izquierdo seguida rápidamente de pérdida
de la consciencia. Muere poco después de su ingreso al servicio de urgencias. Se muestra
una sección de su cerebro como se ve en la autopsia. ¿El mecanismo más probable de la
lesión que se muestra es la hemorragia de cuál de los siguientes?

Arterias lenticulo estriadas


48. Mujer de 67 años acude por inicio súbito de anormalidad en su marcha. El examen
neurológico revela. Anisocoria por miosis derecha, resequedad de la cara derecha, caída
del parpado derecho, analgesia de la cara derecha, analgesia del hemicuerpo izquierdo,
perdida del gusto, y ataxia derecha. ¿Compromiso de la cual de las siguientes estructuras
señaladas dentro de los ovalos explica las manifestaciones de esta paciente?

C
49. Hombre de 65 años acude al departamento de emergencia, debido a inicio súbito de
dificultad para el habla y debilidad de un lado del hemicuerpo. El examen neurológico
revela multiples anormalidades. Una neuroimagen de resonancia magnética revela una
lesión a nivel del tallo encefálico. Según la imagen asociada, ¿Cuál de los siguientes
hallazgos es son los mas probables en este paciente?

Debilidad de cuadrante inferior de cara derecha, incoordinación derecha, hemiplejia


derecha y signo de babinski derecho
50. Un paciente presenta varios años de dolor progresivo "similar a un relampago" en las
extremidades inferiores. El examen revela una profunda pérdida de la sensación de
vibración / propiocepción de la extremidad inferior con sensación de dolor / temperatura
preservada y fuerza preservada en las extremidades inferiores. Hay arreflexia en las
extremidades inferiores. Las extremidades superiores son normales. Se le pide que junte
los pies, y se pare firme (tal como la imagen) primero con los ojos abiertos (lado izquierdo
de la imagen) luego con los ojos cerrados (lado derecho de la imagen) ¿Lesión de la cual de
las siguientes estructuras explica más probablemente las manifestaciones presentadas en
esta paciente?

columnas posteriores
51. Un hombre de 76 años, desarrollo perdida súbita de varias funciones de su cuerpo. Fue
intubado, en su examen neurológico era incapaz de mover sus 4 extremidades, estaba
despierto y era capaz de responder preguntas a través de un pestañeo que significaba “sí”
y dos pestañeos que significaba “no”. Hay movimientos verticales de sus ojos, pero no
horizontales. No hay perdida ni propioceptiva ni dolorosa. Según los hallazgos
encontrados en el examen físico. ¿Cuál es el sitio más probable de lesión entre las
estructuras demarcadas dentro de los círculos en el corte de anatomía macroscópica?
A
52. EN EL ASTA ANTERIOR DE LA MEDULA ESPINAL:

EN SU ESTRUCTURA EXISTEN MOTONEURONAS


53. SEÑALE EL ANUNCIADO CORRECTO CON RESPECTO AL TRACTO SEÑALADO:

LLEVA INFORMACION DE TACTO DISCRITIVA, VIBRACION Y POSICION DE LOS MIENBROS DE


LOS MIEMBROS SUPERIORES, CONOCIDDO COMO CUNEIFORME
54. SEÑALE LO CORRECTO CON RESPECTO AL AREA RESALTADO EN VERDE:

CONTIENEN UN GRAN NUMERO DE AXONES ASCENDENTES


55. El área resaltada en rojo corresponde a:

Cara posterior del mesencéfalo, colículo Inferior


56. La estructura señalada en rojo corresponde a fibras:

Eferentes somáticas generales


57. Mujer de 75 años manifiesta temblor en su mano derecha, manifiesta que se siente más
“lenta” en los últimos meses. Tiene rigidez generalizada. Es diagnosticada con una
enfermedad neurodegenerativa que disminuye las cantidades de dopamina a nivel del
sistema nervioso central y lesiona una parte en el tallo encefálico ¿Compromiso de la cual
de las siguientes estructuras señaladas explica las manifestaciones presentadas en esta
paciente?

5
58. Señale el enunciado correcto con respecto al tracto señalado:

Corresponde al fascículo cuneiforme y lleva información de tacto discriminativo, vibración y


posición de los miembros superiores.
59. Señale el enunciado correcto con respecto al área resaltada en rojo:

Es la fosa interpeduncular y emerge el motor ocular común


60. Mujer de 21 años presenta fasciculaciones en el párpado derecho que han progresado en
un año afectando a la mejilla y el labio ipsilaterales más pérdida auditiva ipsilateral desde
la infancia. Acompañado de analgesia de hemicara derecha. El paciente fallece a causa de
una enfermedad en su tallo encefálico, y se realiza la autopsia de su tallo encefálico ¿Cuál
de las siguientes estructuras, al lesionarse explicaría los hallazgos en este paciente?

A
61. Hombre de 73 años acude por inicio súbito de debilidad de hemicuerpo izquierdo y visión
doble. El examen neurológico revela hemiplejía izquierda más caída palpebral derecha con
desviación del globo ocular hacia el lado externo. Según los hallazgos encontrados en el
examen físico. ¿Cuál es el sitio más probable de lesión entre las estructuras demarcadas
dentro de los círculos en el corte de anatomía macroscópica?

D
62. Indique la estructura señalada con la flecha:

Cola del núcleo caudado


63. En la imagen de talamo, la estrella negra señala:
Complejo nuclear anterior
64. Hombre de 65anos con diabetes mellitus tipo 2. Se presenta a emergencias con ptosis
palpebral izquierda y desviación del ojo izquierdo hacia afuera. Sus reflejos pupilares son
normales. ¿Cuál es la estructura mas probable de la lesionada causante de este cuadro?

A
65. En venas del encéfalo, lo señalado con la estrella negra es:

Vena anastomótica inferior


66. La estimulación del área señalada con la flecha roja produce:

Contracción de los músculos esqueléticos


67. Hombre de 78 anos, con hipertensión y fibrilación auricular. Acude por vértigo, nistagmo,
analgesia cara izquierda, hemianalgesia derecha. ¿cuál es la localización mas probable de
la lesion?
B
68. Cual es la más probable:

Cuna
69. Mujer de 59 años desarrolla súbitamente movimientos violentos incontrolables del
hemicuerpo derecho. El medico de emergencia le dice que se llama hemibalismo. ¿Cual es
el sitio mas probable de la lesión?

D
70. Área resaltada en rojo:

Fórceps mayor del cuerpo calloso


71. Elemento anatómico marcado es:
Núcleo rojo
72. En relación a los ventrículos cerebrales, la estructura número 2:

Agujero interventricular
73. Numero 3:

Seno petroso superior


74. Numero 5:

Glándula pineal
75. Numero 3:
Arteria cerebelosa antero inferior
76. La estructura señalada es:

Seno cavernoso
77. Estructura señalado en rojo:

Hipocampo
78. SEÑALE LO CORRECTO CON RESPECTO AL ÁREA DENTRO DEL CÍRCULO AZUL:

Pertenece al sistema nervioso autónomo


79. Amarillo:

Seno longitudinal superior


80. Estructura señalada:
Vena tálamo estriada
81. LA ESTRUCTURA SEÑALADA CON LA FLECHA NARANJA FIGURA (a). ES:

Seno longitudinal inferior


82. Estructura en el corte:

Pirámides bulbares
83. Estructura señalada:

Cabeza núcleo caudado


84. Con respecto a la arteria señalada:
Se origina el seno carotideo
85. Si ocurre una lesión a este nivel el paciente presentara sintomatología:

Motora
86. Área resaltada en rojo:

Espinotalámico

También podría gustarte